Sei sulla pagina 1di 286

ISSUE TOPICS

To understand the most important characteristics of a society, one must study its major cities.
Write a response in which you discuss the extent to which you agree or disagree with the
statement and explain your reasoning for the position you take. In developing and supporting
your position, you should consider ways in which the statement might or might not hold true and
explain how these considerations shape your position.

The argument that to understand the most important characteristics of a society, one must study
its major cities is a narrow-minded approach to truly understand a society. While it is logical to
study cities as these have the highest concentration of population, such an approach ignores the
small towns, and rural areas that also make up the society.

For a country, such as India, where more than 70 per cent of people live in rural areas, studying
just the cities will give one an incomplete picture of how the society is. Major cities offer the
most visible characteristics of a society, modern or historical, real or fantastic. They are usually
the political, financial and social capitals of a society. When we are trying to get a superficial
understanding of a society, it is the city which provides us a view of the fast-paced, ever-
changing political, financial, technological or social spheres, all of which are situated in major
cities. However, to gain a deeper understanding of a society or rather, the relatively permanent
aspects of a society, we must also investigate into minor cities, towns, villages.

The political, financial and social aspects of a society are certainly important. But one cannot
ignore the other important facets namely tradition, history and culture. Most major cities are
cosmopolitan in nature, that is, they have people of different societies living together. Thus, the
cultural, and traditional aspects of a society somewhat gets diluted. On the other hand, in small
towns and villages, there is almost no globalization and the traditions and cultures of the society
are relatively preserved.

In conclusion, while the major cities of a society do provide valuable insights into the political,
economic and social aspects of a society, one must also study its smaller towns and villages for
more information and understanding of all its traditions and culture.

Educational institutions have a responsibility to dissuade students from pursuing fields of study
in which they are unlikely to succeed.
Write a response in which you discuss the extent to which you agree or disagree with the claim.
In developing and supporting your position, be sure to address the most compelling reasons
and/or examples that could be used to challenge your position
Agree.
1. Colleges have the most knowledge of actual job outlook, job trends, successful students. 
    a. the average student will not know this. has limited view of world.
    b. Colleges should inform students.
    c. Colleges should take care of students.
2. They should prevent students from spending unnecessary time and money in a field they
cannot succeed in.
    a. example) pre-med students who spend years trying to get into med school,    only to be
rejected by the colleges that advised them during admissions.
    b. shouldn't just take tuition, they should help open up career prospects. 

Disagree. they shouldn't dissuade.


1. Colleges don't have a way of measuring a student's potential.
   a. students have hidden potential. michael jackson's high school coach fired him from the
basketball team.
2. The definition of success is different with every person.
   a. sucess means money to some. financial stability
   b. success means a creative life to others
   c. success means being someone who can serve others

Stance. Educational institutions should dissaude student from studying fields he won't likely to
succeed in. 
      Education is a broad topic, one that means career preparation for some, and means
exploration of new ideas and opening of the mind to others. Choosing the right education is
important to success, making the education industry one of the largest in the country, and also
one of the areas that have continuously grown even throughout times of recent times of
recession. It is a given that educational institutions should provide their students with means to
success. They use their resources to train and to education students to reach their highest
potentials. So if an education institution, through its high level of experience with various types
of students and with its knowledge of professional fields, finds that a student is unsuited to his
pursuited field of study, then it should dissuade the student from that particular field. 
         Of course, some might argue that educational instituions have no right to do persuade or
dissuade students about their futures. After all, the idea of success is different to each person.
Success can mean financial stability, a creative life, or the ability to be of service. What right do
institutions have to decide success for a student? And how exactly can educational institutions
know how to judge a student? Michael Jordan's high school coach considered him a horrible
basketball player and failed him during tryouts. Students have hidden potential and no judge is
perfect.
         Yet looking at the first argument, though the idea of success is different to each person, a
basic tenet of success is satisfaction of a certain level of financial freedom and of contributing to
society. If an educational institute sees that a student might have trouble fulfilling this basic
need, then it should inform the student of other choices he has in his careeer. Also, though not all
judges are perfect in their perceptions of their subjects, if a student has passion and the drive to
succeed in his given field, then no amount of criticism will stop him. Michael Jordan would have
become the number one basketball player whether or not his coach dissuaded him. In fact,
perhaps because of this dissuasion he might have tried harder. If someone truly wants to do
something, then he won't be affected by dissuasion from others. If, on the other hand, a person is
timid about his future and uncertain, then a little guidance might be of great help.
       An important reason why educational instituions should help their students decide their
courses of study is that they have the most knowledge of the real world of job prospects and
cases of successful students. They have access to research, to past students, and to companies
and organizations throughout the committee. They have a more accurate view of the real world
of professional jobs. The student, on the other hand, though not in all cases, will most likely be
less knowledgeable of the current world than the educational institution. The student will
probably be younger and therefore have less experience with careers and jobs, more likely to
make decisions on faulty assumptions or on misevaluations of himself. Many students prepare
for medical school by taking pre-med during their undergraduate years, their basis for
preparation being vague dreams of being doctors or pressure from parents. However, if a
student's capabilities are not toward the medical sciences, they should be afforded the resources
to decipher this, if not actual counseling as to better career choices. 
Educational institutions should also prevent students from spending unnecessary time and money
in a field they cannot succeed in. Costs of college are increasing every year, with graduate school
becoming the norm. If students are investing so much of their time and money into an
educational institute, that institute has a responsibility not to just provide them with a diploma
but also to prepare them for their careers. There are pre-med students who spend years trying to
get into med school, only to be rejected by the colleges that advised them during admissions.
There are also colleges that are considered diploma mills by knowledgeable professionals, who
accept students based on their ability to pay tuition, and who roll out diplomas and graduate
students without taking responsibility for their futures.
Because educational institutions have the most resources, connections, and knowledge, and
because their students are investing important time and money into them, they must take
responsibility toward their students by making sure they enter fields they can succeed in. If a
student's characteristics prove he cannot be successful in his chosen field, the institution must
provide him with the realisitic reasons why he is not suited and show him a better course of time.

Scandals are useful because they focus our attention on problems in ways that no speaker or
reformer ever could.
Write a response in which you discuss the extent to which you agree or disagree with the claim.
In developing and supporting your position, be sure to address the most compelling reasons
and/or examples that could be used to challenge your position.
Scandals are useful because they focus our attention on problems in ways that no speaker or
reformer ever could.
Write a response in which you discuss the extent to which you agree or disagree with the
claim. In developing and supporting your position, be sure to address the most compelling
reasons and/or examples that could be used to challenge your position.
The Indian Government faced the largest telecom scandal when the news of its awarding the 2G
spectrum to private companies without legal auctions got exposed, leaving the Government
vulnerable to bear the brunt of extreme criticism, from both the national and the international
media.

The beleaguered Government had also illegally allocated some of the country's largest coal
mines to public sectors and other private companies, which when unveiled, resulted in the
resignation of several political bigwigs working in coalition with the people involved in the scam
and de-allocation of several coal mines.

These are events that stir the entire nation, agitate the mass and very often lead to the no-
confidence of the Government. Scandals, as we term them, exhume some of the deepest secrets
of any nation, be it of the Government or at the individual level.

Scandals are widespread mammoth occurrences, publicized extensively, that defames the
involved institution. They are classified broadly as political, social, educational, sports and
religious scandals. They spread like wildfires, across entire nations within a short span, fuelled
by the media, resulting in mass movements and criticisms.

The scandals offer ways to expose the dire problems and their consequences vehemently than it
can be done through public speakers or reformers. What speakers can achieve is to agitate the
mass, generally limited within certain geographical boundaries and political constraints, but
scandals spread like wildfires, engulfing the entire nations in flames, charring it to dust and
spewing out the dark smoke of political blasphemy. They bring to light the closely guarded,
illegal activities a nation's deeply involved in, the various scams worth millions of dollars of
people's money and life, and, unmasks the Satan that hides behind the 'social reformers' who are
accused of the scams. They tend to focus our attention to problems that we normally,
nonchalantly and blatantly avoid. The paparazzi, the media, the press, the internet and quite often
the whistle-blowers too, cumulatively fuel this effort.
International scandals, involving some of the renowned personalities and organizations, such as
the Olympics scandals, where eminent sports persons were found to resort to drugs, the US
president having an amorous affair with a White House office staff, the Pope of the Vatican City
demonized for outraging the modesty of young seminarians and fathering a child, or Wikileaks
exposing the brutality the Afghan prisoners faced at the hands of the US army, call for
worldwide action and demand purgation of the colloquial system. These problems cannot be
sufficiently addressed by any reformer as very often they may face grave consequences from
people in power that eventually subdues or threatens them. Scandals brazenly expose the nudity
of the system, which cannot be veiled with the rags of lies.

News of such wrong-doings proliferates through both scandals and rumors. But what
differentiates a scandal from a rumor is that the latter changes its hues and forms as it passes
down through various channels whereas scandals exponentially evolve, exposing more scandals
associated with the previous ones, forming a mountain out of a molehill.

But scandals are not always true. Rumors that cumulate into scandals may often become
disorganized and wane away and deviate from their specific course. It may also result in uncalled
violent consequences between people or nations. It is at this juncture we need a humble reformer,
who would guide the agitation with profound leadership qualities, which would help the cause
achieve its motive and mitigate the problems.

Scandals, thus, cannot be restrained within any specific boundary. Scandals spread their roots
and secure themselves firmly to the ground only to shake the foundation of the entire setup and
demand reforms. They tend to expose the institution behind the purdah, purify the system and
purge out evils. In a grand and effective way, unlike any other method, it draws people's focus on
issues which require immediate attention.

Claim: Governments must ensure that their major cities receive the financial support they need in
order to thrive.
Reason: It is primarily in cities that a nation's cultural traditions are preserved and generated.
Write a response in which you discuss the extent to which you agree or disagree with the claim
and the reason on which that claim is based.

Admittedly, a nation’s cultural traditions should be preserved and it is in some of its major cities
that the nation’s traditions are generated. For example, one could have a deep understanding of
traditional Chinese culture by visiting Beijing and seeing the Forbidden City, the Summer Palace
and the Temple of Heaven. Also, one could become acquainted with Indian Culture by visiting
New Delhi and seeing the Taj Mahal. However, this doesn’t mean that governments should give
their major cities the financial support just because these cities need such support to thrive.
First, major cities have their own ways to get financial support. Since many famous companies
have built their headquarters in the big cities, these cities’ government could collect large tax
revenue from big companies. For example, the headquarter of the E-commerce giant Alibaba is
in Hangzhou and the headquarter of the Chinese largest search engine company Baidu is in
Beijing. Thus, each year, these big companies would pay millions of dollars of tax to the city
government. Also, the big cities would always attract big foreign investments. Recently the
famous Walt Disney Company have just invested several billions of dollars to open the Shanghai
Disneyland theme park and this would undoubtedly help to boost the growth of Shanghai’s
economy. Finally, the tourist industry in major cities would always be very prosperous. Every
year, millions of tourists come to Beijing to see the Forbidden City or to Shanghai to see the
Oriental Pearl Tower, which would generate millions of revenues to these cities’ government.
Therefore, there is no need for the government to ensure that the major cities should receive the
financial support they need because these major cities have various ways to get financial support,
such as tax revenues, foreign investments, and tourist industries.

Second, due to the globalization, many major cities all over the world are becoming increasingly
similar to one another. They all share the same characteristics, such as skyscrapers, wide streets
and modern facilities. Thus, people might not experience the nation’s cultural traditions in big
cities due to the similarity among them.

On the contrary, it is in some small towns and rural areas that the nation’s cultural traditions are
preserved. As a result, government should give these regions the full financial support to
preserve these cultural traditions. For example, minorities living in the remote Yunnan, Guangxi
and Guizhou province of China have unique traditions and customs that one cannot experience in
major cities. A minority called Dai in Yunnan province has a traditional water-sprinkling festival
that people pour water onto each other to celebrate the lunar new year. Also, such small towns
and rural villages are relatively less-developed than are big cities so they need more financial
support than do big cities. In China, there exists huge inequality between urban cities and rural
areas. People living in urban cities like Beijing and Shanghai would receive much better health
care service and educational resources than people living in rural areas. Almost all Chinese best
universities and best hospitals are located in big cities like Beijing and Shanghai. By contrast,
people living in rural areas and remote villages have limited access to education and medical
resources. And these underprivileged people could not move freely to the big cities because of
the Hukou household registration system. Thus, small towns and rural villages usually could not
attract big investments and don’t have large tax revenues. The government should give more
financial support to the small towns and rural villages to help these areas to thrive, so that the
cultural traditions in those areas could be preserved.

To sum up, there is no need for the government to ensure that the major cities should receive the
financial support they need because these major cities have various ways to get financial support,
such as tax revenues, foreign investments, and tourist industries. By contrast, the government
should give more financial support to the small towns and rural villages to help these areas to
thrive, so that the cultural traditions in those areas could be preserved.

Some people believe that government funding of the arts is necessary to ensure that the arts can
flourish and be available to all people. Others believe that government funding of the arts
threatens the integrity of the arts.
Write a response in which you discuss which view more closely aligns with your own position
and explain your reasoning for the position you take. In developing and supporting your
position, you should address both of the views presented.
The term "arts" refers to a broad category of musical, physical, and visual-spatial practices, but
in essence it contains the ideas of self-expression, creativity, and freedom. The arts connect
cultures when people do not speak the same language, with visual images transcending the
boundaries of language. Even within people who speak the language, the arts provide a channel
for communicating ideas, emotions, and values in a way that words alone cannot. The arts then,
broad as the term is, are an important part of any culture. Therefore some people believe that the
government should take an active role in funding the arts so that they can flourish and be
available to all people. However, other believe that government funding threaten the very
essence of art. My opinion is that though government funding is beneficial rather than harmful to
the the field of art.
The most beneficial part of government funding would be in art education. Because children
contain the future's vision, they need to have an education that develops them not only mentally,
but also in their creativity and their ability to connect with others. The arts are the most
affordable way to bring children in touch with this ability. The most ideal method would be to
provide each child with a wise mentor to help him become enlightened, but as this is not feasible
due to the shortage of such ready-to-work mentors and the costs of such a program. The arts,
however, allow children to have a medium that helps them develop their skills and learn such
things as dedication, practice, craftsmanship, and to experience the pride of presenting their
creations. Government funding in this case would help children gain access to art programs.
Currently, a rigorous education in the arts is taught only in exclusive private schools, and even
public schools have cut the visual arts, physical education, and music from their programs
completely due to lack of funding. Government funding of the arts would help bring these
programs to a greater number of students instead of to a select few.
For people who completely disagree with the notion of government funding, they might argue
that art in the past has not been financed by government but by private aristocracy. They might
argue that government funding is not necessary - after all, the great pieces of Michaelangelo
were by commission and not funded by the government. They might also argue that government
funding would limit creativity and self-expression, as the government would have an agenda.
However, though many of the famous paintings in history were funded by either individuals or
organizations, the act of funding did not limit the quality, creativity, or self-expression of the
artists. The Mona Lisa by Leonardo da Vinci did not have any less brilliance or genius due to the
fact that it was a commissioned project. Whether this funding is from a private individual or
from the government makes little difference. The paintings on the ceilings of the Sistine Chapel
were commissioned by the Chuch, which in those times had as much, or even more, authority
than our current government. However, though the work was funded, and though there was a
very pointed agenda of Christian theology, the work itself is still no less a great artwork than if it
were created for the purpose of self-expression.
      The arts has tied in with its name the ideas of creativity and self-expression, and the notion of
government funding, which carries agenda and politics, doesn't tie well with these ideas.
However, funding of art works in the past have not diminished their value, and more importantly,
government funding is greatly needed in the field of art. The area that needs the most support is
art education of students, as this creates the greatest long term benefits for the public. 

_______________________________________________________________
Other Examples

 KUMAR VIKRAMJEE

Arts depict the characteristics of a civilization and society, so, the preservation of all the art
forms is of utmost importance. Government can help in preserving the integrity of arts by
funding and subsidizing art related works.
Arts are mostly non-profitable endeavors. Proper financial support to arts will ensure that, artist
are able to develop their artistic skills and they get a proper platform for exhibiting their skills
without any hardships. 
In developing and under-developed countries, the scarcity of financial support hinders the
proliferation of arts. The artists are forced to choose alternative career options in which they can
afford a standard of living. Such a scenario has a detrimental effect on the arts of a civilization. If
the government provides an adequate financial support and encouragement, then the younger
generations shall be able to find career options in arts. 
In most of the countries, museums and art galleries are funded by the government because they
are subjects of national importance, likewise a particular skill set which develops an unique art
forms such as plays, paintings, or music is of national importance and must be preserved. 
On the other hand, the government funding of the arts may threaten the integrity of arts, if the
funding is prejudiced to some particular fields or subjects of arts. In Soviet Union era,
government encouraged only those areas of arts  which aligned to their political motives. This
biased support led to ignorance of other areas of arts which did not developed in that era. In
some countries, a particular forms of drawing or music are banned. So, if the government
funding is biased towards different subjects of arts, then the overall development of the arts gets
compromised, as the creativity of the artist is limited to produce arts with specific themes. 
It's the duty of the government to be unbiased towards all forms of arts and contribute to the
overall development of arts. 
Summarizing the topic, government funding of arts is beneficial for the proliferation of arts, as it
helps in preserving as well as developing arts. But, it may also affect the development, if
government funding is concentrated to particular subjects , rather than overall all development of
arts.

 http://theunauthentic.wordpress.com/2011/10/06/gre-argument-practice/

Funding for the arts can be a controversial topic when nations now are competing to see which
one has the highest math scores, the most technology, and even the best athletes. The arts are an
extension of culture however, and should receive the same sort of funding that so many other
programs receive. That is not to say that they should be fully funded by the government, but in
order to flourish and reach all people, some government support should be available.

It is the start up in any business that requires the most capital and the biggest risk. In the same
way, new programs require the most at the beginning, but if they are given proper support, they
can support themselves with only some contribution needed for maintance. Athletics budgets all
over the country vary greatly, but what the best ones (that reach the most people) have in
common are not just their ample budgets but their high participation of fans. This participation
leads to revenue for the program which it can then turn around and re-invest in itself. In much
the same way, the arts need funding to gain exposure so that they have the opportunity to find
those same supporters and participants.

Even the most talented people need things to accomplish their goals. This can be especially true
in the arts. Music requires instruments, art requires supplies, plays require props. While it is
possible to do all these things while using borrowed instruments and minimal supplies and props,
for a program to truly flourish it needs more than the bare minimum. Through government
support these programs are able to provide more for their students and they can reach a wider
audience. Once they do that, they are then able to reach out for support on their own and require
much less of the program.

No program should rely solely on the government. The arts are capable of supporting
themselves, but it is much harder than with athletics. While a football team can count on ticket
sales throughout their season, most bands hold one to two concerts a year. They may not even
charge admission for these concerts. Government support, once the program is established, can
help to supplement what they are not able to make themselves. During high school, our marching
band members were required to participate in fundraisers, which were held throughout the year.
These fundraisers were used to cover the cost of travel for the band, but were not even enough
for that. Without money they recieved from the school system and government programs, they
would not have been able to compete, despite being one of the best in the country.
Claim: In any field — business, politics, education, government — those in power should step
down after five years.
Reason: The surest path to success for any enterprise is revitalization through new leadership.
Write a response in which you discuss the extent to which you agree or disagree with the claim
and the reason on which that claim is based.

The author in making his claim has neglected few points which might go against him. The
frequent changes in leadership don’t allow the leaders to develop a full understanding of the
system’s values and to set a rapport with his people. If a leader is successful in achieving
organization’s goals then leadership change is unnecessary. But the author is quite true in the
matter of innovativeness. New leader brings innovativeness in ideas with him. He works with
great vigour to fulfil his duties. Hence leadership changes are good as far as the organization is
getting good benefits out of it.

If an existing leader is doing well, then it’s not necessary that he should step down after five
years. As far as he is achieving the organization’s or system’s goals he should be allowed to
continue after five years. E.g. a school principal, if he is successful in bringing good discipline in
addition to excellent academic and extracurricular performance in the students he must be
allowed to continue his service even after the five year terms.

If a leader has served his organization for a long time then he must be having a complete
understanding of the system’s core functional values, missions and goals. This understanding
will help him in taking decisions which will be most beneficial for the organization. On the
contrary a new leader requires some time to completely understand its organizations principles,
beliefs and goals. Therefore his decisions may not be coinciding with organization’s needs. If he
has spent half of the entire service period of five years in understanding system’s values then the
time left for converting these understandings in to actions will be very less.

A leader can never succeed in achieving his goals without the support of his organization. He has
to win the confidence of his people. To set a rapport with his people he need to spend a quite
long time in the system. E.g. selection of narendra modi as BJP’s Prime Minister Candidate
didn’t happen overnight. It took 15 years to him to set a rapport with his fellow members. People
just cannot accept readily a newly elected leader no matter how competent he is. Hence in a short
period of five years it will be quite difficult for any newly elected leader to establish an accord
with his people and hit success.
The author’s view is quite appropriate in the view of innovativeness in the ideas and decision
making. A new leader thinks differently than his predecessor. He may find the solutions to the
existing problems of the system in a different and more efficient way. E.g. the newly elected
governor of RBI brought some changes in the import export regulations which proved highly
successful in taming the CAD of the nation, which his predecessor failed to achieve. Hence to
look into problems with different angles leadership changes are mandatory.

A newly elected leader is driven by spirit of obligations. Hence he works with tremendous vigour
and dedication. This vigour and determination might be missing in the leaders who have spent
several years in the organization. The ennui of the job generally causes a decrease in the
dedication and interest for the job. This can hamper the decision making ability of the leaders
which will ultimately reflect in organizations poor performance. Hence to invigorate the system
with a fresh breeze of ideas leadership changes are bound to be made.

The author in making his claim has neglected few points which might go against him. The
frequent changes in leadership don’t allow the leaders to develop a full understanding of the
system’s values and to set a rapport with his people. If a leader is successful in achieving
organization’s goals then leadership change is unnecessary. But the author is quite true in the
matter of innovativeness. New leader brings innovativeness in ideas with him. He works with
great vigour to fulfil his duties. Hence leadership changes are good as far as the organization is
getting good benefits out of it
In any field of endeavor, it is impossible to make a significant contribution without first being
strongly influenced by past achievements within that field.
Write a response in which you discuss the extent to which you agree or disagree with the
statement and explain your reasoning for the position you take. In developing and supporting
your position, you should consider ways in which the statement might or might not hold true and
explain how these considerations shape your position.

By reviewing the past history of humans' life, it will be revealed that they have shown significant
performance which has led to make some "impossible" situations to "possible" ones; moreover,
human races has been always trying to use the previous experiences of their ancestors to achive
progresses in "some" field of endeavor; in fact, the humanity has a kind of limitation in these
experiences; hence, just can use of "some" of them not "all".

In the statement has been mentioned that it is "impossible" for the mankind gain improvement in
"any field" of endeavor which is in a contradict with some fields of study; for instance, the
areospace technology and knowledge almost a new technology which might have an age around
fifty years. Before the first journey of human to the moon, there were not any previous
achivement in this field and, even this technology would never come to the mind of our
ancestors; but after that NASA could land its first spacecraft on the moon, the new science of
areospace was established. Or as another example, the discovery of relativity ratio by Einestain
almost sixty years ago, it was the first time that the knowledge of nuclear power was founded
which before that period, a little number of people, just a few scientists including Einstain were
involved in this field. In fact, in some fields of study that are known as contemporary sciences
such as areospace and nuclear power, there is no previous achivement.

In addition, in the statement has been mentioned that in "all field" of endeavor, it is impossible
that reach to achivement; however, as it was mentioned in the previous paragraph, just in few
fields such as aerospace, nuclear power and Nano cannot use of previous achivements. On the
contrary, there is a positive side that human could use of it such as airplains, trains, cars and etc.
Wright's brothers were constructed the first type of plains which has no motor but, they could fly
with it. After passing times, this invention was improved and progressed, but the fundamentals
and basic rules are the same. This improvement has flourished as far as we perceive a vast type
of plains; Airbusses, Jests, Booings and etc. are such these examples. In the case of trains, we
also can see such this enhancement; improving from a simple steam train to a high speed and
sophisticated train. Hence, humanity has been able to use of same achivements by previous
people as an instructor and director.

As a summary, the humanity has shown that is capable to change "impossible" parts of the life to
"possible" facets while there is no previous experience about it; on the other hand, he/she could
utilize previous experience to progress some field of study.
Nations should pass laws to preserve any remaining wilderness areas in their natural state, even
if these areas could be developed for economic gain.
Write a response in which you discuss your views on the policy and explain your reasoning for
the position you take. In developing and supporting your position, you should consider the
possible consequences of implementing the policy and explain how these consequences shape
your position.

The actions of the humankind during the past few decades, has by far made this time the worst
period for our environment. We have polluted the air, the water and the soil, hunted countless
animals, destroyed the ecosystem, and made wildlife impossible in many areas of the world. In
short, we have ruined the natures hierarchy, and we have done it for one reason: we thought the
progress in economy and industry is worth the sacrifice. But there was a point when some people
saw that this is a malfunctioning cycle, and needs to be readjusted somewhere - nations should
start paying attention to the environment, and since fixing the ruined jungles and forrests and
other natural landscapes will probably take hundreds of years, we should take extra care of the
remaining parts.

Having all this in mind, the statement above is making a good point. Many people, from
environmental activists to ordinary people who just care about the nature and wild life, would
undoubtedly defend this statement and agree that governments should pass laws to preserve the
remaining wilderness by any means possible. They would argue that the earth, the mother nature,
has nourished us beyound our expectations, while in return we have torn it apart. Since no other
organization would have the power to do so, it would logically be the goverments’ responsibility
to pass laws to preserve the remaining wilderness, before they are occupied and destroyed by
factories or incorporations seeking profit.

But looking deeper down the statement, there are many things that should be defined before
passing such rules. For example, there are for sure countries in africa wich can not provide
enough food for their people, and have acres of “wilderness areas” preserved, because people are
unable to use them properly. in this case, there should be groups of experts studying the area, to
define the fine line between the areas that should be preserved, and the areas that could be put to
use in order to provide people’s basic needs. The other case, are the national parks, which
contrary to public beliefs do have economic gain. So not all economic gains should be
considered against preserving the nature.

So, although I believe that nature should be preserved from those who plan to destroy it for
economic gain, I think there are expectations to add to the above statement. While the nature
should be preserved “natural”, we should understand that the people of “today” should be
respected as much as the people of “tomorrow”. Environmental mottos and quotes sound very
pleasing when they preach about keeping the nature for the future generations, but they largely
forget that we are not providing enough food for the present population. Let’s preserve the
natural wilderness areas, but let’s do it correctly.

People's behavior is largely determined by forces not of their own making.


Write a response in which you discuss the extent to which you agree or disagree with the
statement and explain your reasoning for the position you take. In developing and supporting
your position, you should consider ways in which the statement might or might not hold true and
explain how these considerations shape your position.

The term "behavior" is not quite easy to describe. There exists a plethora of definitions for it
according to individual perspectives. But, in general "behavior" means how a person acts or
reacts according to the external or internal forces. Human beings being the most intelligent
species, have liberty of choice. They think, decide and execute things. People do behave in a way
they are brought up. Many external factors such as society, environment, school, peers, families
may contribute to their thinking, but it is highly unjustified to say that external factors are solely
responsible for the behavior of people.

We as human learn, hear and absorb things but eventually it is us who decides what to do. We
have free will and we can only account ourselves for our actions. However, there are outside
factors that contributes to human behavior. External factors always had impact on humans.
History shows many a great or flawed men were a product of these influences. But, eventually it
depends on the individual about how and what to do. There are laws, rules and regulations in the
society but it depends on the individual whether to abide by those rules or not. For instance, in a
particular country there may be a punishment for throwing garbage on the road. A person
travelling to that country might not throw garbage being extra cautious about the rules there. But,
when he goes to the other country where there are no such rules, he might. In this case, their
sense of individual responsibility will come into play. External forces might push people, make
them think, realize what needs to be done. In the end it is completely dependent on us to make
the decisions. We are our decision-makers and nothing can make us do what we don't want to.
There are rules, laws and outer forces that continuously reminds us against doing bad or illegal.
But, there are people who break laws, which is evident by the crime rate. So, what are the effects
of external forces on such people? They have decided not to let the external forces determine
their behavior. Students are continuously told to study well for their bright future. There are
students who will let the external forces push them and study well. On the other hand, there are
students who will not let the external forces influence them and act according to their will. A
thief will steal in spite of the laws in the society. Many argue that their act resulted from poverty,
circumstances etc, but it is eventually the thief who makes the decision in spite of the options. A
person can only be responsible for his behavior. External forces can play a part, but human
beings are the outcome of their own actions.
However, the above statement is not entirely without support, it runs contrary to common sense
and everyday human actions. So, I firmly believe that our action springs from our free will
accords with common sense and everyday experience.
Governments should offer a free university education to any student who has been admitted to a
university but who cannot afford the tuition.
Write a response in which you discuss your views on the policy and explain your reasoning for
the position you take. In developing and supporting your position, you should consider the
possible consequences of implementing the policy and explain how these consequences shape
your position.
One of the most important and complex phases of life that, in addition to enhance knowledge,
shapes a human being, contributes for an all-round development and growth, associates him with
the highly functioning world, is college. College education goes beyond books, moulding the
person, both physically and mentally, shaping and focusing on his career. With more and more
students embracing higher education, the state stands to benefit economically, prosper culturally
and in terms of new leaders from today’s youth force. Nevertheless, do every student who enrols
in higher education come from economically firm backgrounds? Given the rising cost of
education and vast majority of the population belonging from middle class families, how is it
possible that this basic question of relaxing the tuition fees has escaped the minds of policy
makers? Thus, every Government should strive to make university education free for students
who have been enrolled, but cannot pay the tuition fees.

To be educated is our right, and to educate is our duty. It is this fundamental law which now has
been monetized and been turned into a money laundering scheme- an opportunity for tax
collectors and investors to extort money from students and professionals alike. With this gradual
trend, we oversee how important education today is to a country and to its people. It is the
educated force, who creates all the difference, and not merely money- as without education and
proper knowledge, development will lose its trail and money will become futile. Hence, a
Government, as its fundamental duty towards its citizen, should encourage university education,
without burdening the students, especially the needy ones, with the liability of tuition fees.

Exempting the students from tuition costs of university education will have advantages manifold.
There will be an inevitable increase in the number of students opting for higher education-
especially those with merits but not with enough funding. This will provide the State a
continuous stream of well professional engineers, doctors, theorists, artists, thinkers, writers and
revolutionists, among the many. With sound education, these people can positively contribute
towards the society – in fields of advancement of technology, health research, increase in
aesthetic values, literary works, towards deeper understanding of the past and future, and
bringing in changes. Students, thus can be encouraged to pursue higher education for positively
contributing towards research and development, or academics. Hence, a nation stands to benefit,
in all aspects, raising the potential of people and itself in general, thus welcoming a “smarter
economy”.
On the contrary, education comes at a great price. University education costs soar due to
inclusion of newer labs, advanced technology, securing patents, getting new projects and
research subjects. Without charging the students the tuition fees, the cost to state will rise to an
unexpected value. Private universities, unlike having a support from Governments for most
public universities, will not be able to remunerate the faculties. Quality of education will droop,
and the overall benefits will observe a gradual plunge. Policy makers, before passing the bills of
free university education, should therefore, examine the situation with greater care and deeper
contemplation.

With an increase in number of students applying for redemption from tuition fees, a proper
background examination deems to be obligatory for the universities. Consideration should be
done strictly on the basis of merit and low family income groups. Tie – ups with university
students to serve and contribute, maybe in terms of academics, research or money, to their alma
mater after they have flourished and established themselves, will no doubt help to regain the cost
to university per students. Meanwhile, grants and funds for research can be channelized from
students placed in big industries and research groups.

Implementing the bills for free higher education for the low income background students,
undoubtedly, will foster merit and greater encouragement towards pursuing masters, PhDs, or
even post-doctoral researches, must however be done after careful scrutiny, as it involves an
enormous amount of money, investment of energy, years of research, and an ultimate aim for the
versatile development of the nation in the long run, which if achieved, will escalate the nation’s
respect among peers. After-all, education is the sole instrument for evolution and progress.

Universities should require every student to take a variety of courses outside the student's field of
study.
Write a response in which you discuss the extent to which you agree or disagree with the claim.
In developing and supporting your position, be sure to address the most compelling reasons
and/or examples that could be used to challenge your position.

Learning is a lifetime opportunity and students have to make the best use of their learning period.
Even if a student decides to do a particular major, his course selection must include a variety of
courses, other than those from his field of study. But, it is implied that, choosing a variety of
courses outside his field, must not hinder his concentration on core subjects relevant to his field.
Instead, a fair distribution of courses, both from his field and other fields, must be followed for
each semester. It is the responsibility of the universities, to give the students freedom to select
the elective courses of their choice. Consequently, this will contribute to a full-fledged learning
in a university.

First of all, every field in this field is dependent on every other field. There is no single field,
which can work on its own. For instance, computer science has few of its courses from
electronics - which form the base for understanding computer architecture; for instance, basics of
electronics and digital signal processing are prerequisite for a computer science major, to delve
deep into the interior of computer architecture. Also, in the workplace, an IT professional cannot
expect all of his peers to be from a computer science background; he has to collaborate with
those from other fields like electronics, electrical engineering etc... Hence to have a basic
understanding of other fields is required for guaranteed success in our chosen field.

Moreover, taking up courses outside ones field of study kindles one's interest in other fields.
Additionally, it exposes a student to a variety of courses, from which a student can identify his
field of interest. For instance, a computer science graduate can take up few courses from
economics and management. Consequently, if he happens to be driven more towards this
economics course, then he can decide to an MBA and become a successful businessman. Thus,
course selection can go even to the extent of diverting one's career path, and assuring success in
the chosen path. Only if a student is exposed to few courses outside his major, will he know the
gamut of career oppurtunities from which he could branch out, after his graduation.

Furthermore, research is of utmost importance to the development of any university. A university


which produces successful research scholars, is deemed in the academia as the best university.
Living examples include those universities in the United States such as Carnegie Mellon,
University of California-Berkeley etc... These U.S universities are special for a single cause -
they allow the students to choose a variety of courses to complete his degree; there is no
restriction on the type of courses to be chosen to complete a degree. Because of this freedom,
students are able to devote more concentration to their fields of study. This ends up in fruitful
results when researches are being conducted. Research in any field, requires a sound knowledge
of the related fields too. For instance, a research scholar in artificial intelligence, might require
some help from the mechanical engineering department, in order to validate the mechanical
parameters and construction of the robot. The programming in robots is done by the computer
science majors; but the ultimate working of the robot and its manufacturing, requires assistance
from the fields of mechanical and electronics engineering. Thus, any research field, requires
atleast little assistance, from the related fields. Hence, taking up courses outside one's field, will
go a long way in researches to be conducted in the long run.

Taking up additional courses, has one another advantage: it enables a student to do part-time job;
for instance, if an engineering graduate is damn interested in English literature, he can take up a
course in English literature. Ultimately he can turn out to be an engineer cum story-writer. Thus,
he has a possibility of always keeping him busy. Hence, additional courses, can even contribute
to the increased earning potential of an individual.

Some may state that taking up courses outside one's field, deviates his attention. But, this is
possible, only if the courses are very much unrelated to a chosen field and they are large in
number. Courses, if taken outside one's major, must be limited in number and they must not be
completely unrelated to one's major. They have to possess some motive of helping an individual,
in reaching his desired goal. Also, the universities must be in a position, to provide experienced
staff members, to handle these courses, kindling interest in the students. Except for these
conditions, these additional courses, do not deviate a student from his chosen major.

Thus, choosing a multitude of courses, outside one's field of study offers various advantages like:
flexibility in choosing the courses, chance for potential research, increased earning potential,
second chance to divert one's career to a more relevant choice. While some may argue that these
courses seem of no use to the student, the courses actually come in handy to a student in the long
run. "Known is a drop; Unknown is an ocean" goes a saying. Hence, there is no limit to learning
and learning continues throughout one's life. Thus, there is really no harm in choosing a variety
of courses outside one's field of study.
A nation should require all of its students to study the same national curriculum until they enter
college.
Write a response in which you discuss your views on the policy and explain your reasoning for
the position you take. In developing and supporting your position, you should consider the
possible consequences of implementing the policy and explain how these consequences shape
your position.

Already for years there are speculations about reforming the school system in a lot of countries
including the United States. The leading example for all nations is of course Finland which
consistently strands on the top spot in all global educational rankings, from math to grammar.
They have a national curriculum that every student needs to follow until they enter college.
Should other nations adopt this approach to education or isn't everything as bright as it seems? A
scrutiny regarding this topic will be necessary to give a decent answer.

One the one hand, one can say that a national curriculum certainly has its plus sides. It provides a
whole nation with the same knowledge, which creates a coherent pack of students with the same
educational background. This makes it easier for colleges to adjust their classes, since they now
exactly know with what previous knowledge students arrive. For the same reaon, this approach
makes it easier to compare schools with each other. Having the same curriculum will make clear
which schools offer a better education by comparing graduating and drop out percentages
(although other factors influence these percentages as well). For students who need to transfer
schools there is also a benefit. Instead of comparing all courses they had with the ones the new
school offers, they will immediately fit in and be able to pick up exactly where they left. For
comparing purposes, the national curriculum has without a doubt a benefit, ranging from
cmparing individual students to whole schools.

On the other hand, a national curriculum may limit certain students in what they are able to
achieve. By not offering the classes they would like to see, they might become demotivated and
drop out of school. After all the point of education is to make students warm for studying, to
trigger their interest in certain subjects. If a school fails to achieve this goal due to a forced
national curriculum, that would be a pity. The student's willingness and eagerness to learn should
come in the first place. However this is a valid argument, this approach suits more the one of a
college or university, where a student can study what they want. With the necessary flexibility, a
national curriculum might thus do the job for high school students.

What should not be forgotten is that a national curriculum alone will not get countries to the
same international standards as Finland. However it may help, the true heart of education lies
with the teachers. If the teachers are good and can get their point across, students will benefit
greatly. Therefore one should take a look how teachers are looking against this national
curriculum. If they agree, they will give their heart and soul for this program, resulting in an
overall better education. If they oppose the reform, their teaching will most certainly deteriorate,
leaving us with a possibly better school system, but with less educational classes due to
demoralised teachers. Therefore the implementation of such a reform of the educational system
should be agreed upon by the teaching community before government requires it.

In sum we can say that a national curriculum certainly has its benefits, enabling us to easily
compare students and schools with each other. However there should be some flexibility in the
program leaving the possibility to make minor adjustments to suit the learning interests of the
student so as to keep him/her interested in learning. But after all the big question is if the
teaching community agrees with the proposal, because they will determine the level of classes. A
national curriculum will therefore be a succcess if the teachers are standing behind this
reformation of education.
Educational institutions should actively encourage their students to choose fields of study that
will prepare them for lucrative careers.
Write a response in which you discuss the extent to which you agree or disagree with the claim.
In developing and supporting your position, be sure to address the most compelling reasons
and/or examples that could be used to challenge your position.

Students should have the freedom to choose, and explore fields of study that are appealing to
them. Thus, the role of educational institutions is to train students in fields of their choice, and
not actively encourage them to choose fields on the basis of opportunities at lucrative careers.
The reasons for these views are as follows: having a lucrative career depends on the individual,
and not the field of study; also, changes in government policies and economic stability may
render these so called lucrative jobs unavailable.

First, and foremost, such a statement creates the notion that some fields of study do not lead
students to lucrative careers. Having a successful career depends on the student, what the student
makes of the field, and how the student effectively applies learning in the classroom to the real
world. There are numerous lucrative areas available within any field of study. For instance, in
Nigerian universities, there is a cachet placed on courses like medicine, engineering, and law.
However, agricultural students are viewed ludicrously, but many of them are able to start
lucrative poultry, and fish rearing businesses before their counterparts get jobs.

Furthermore, the continuous availability of these so called lucrative jobs or careers in particular
fields of study is not certain. Changes in government policies and economic stability may reduce
the presence of these jobs. For example, the effect of the economic recession in countries like
Greece, and Portugal has been a lack of jobs, even high paying jobs. In addition, in the Nigerian
banking sector, the apex bank implemented a recapitalization policy for banks, these lead to
bankers taking pay-cuts and in other cases been let go. Thus, a banking sector once considered
lucrative is now less benign.

Opponents against my idea argue that educators are helping students make the right decisions for
their future careers. However, students should make the decision of what kind of career paths or
options they plan to take; if they fancy careers in academic research or want to be lawyers that
take pro bono cases or work in the oil industry, it is their choice, a career choice should be one
were an individual feels fulfilled.

Conclusively, lucrative opportunities are inherent in all fields of study, educators should help
students meet their aspirations and not advice them to take particular fields of study because the
chances at lucrative careers in such areas are apparent.

Some people believe that in order to be effective, political leaders must yield to public opinion
and abandon principle for the sake of compromise. Others believe that the most essential quality
of an effective leader is the ability to remain consistently committed to particular principles and
objectives.
Write a response in which you discuss which view more closely aligns with your own position
and explain your reasoning for the position you take. In developing and supporting your
position, you should address both of the views presented.

Political leaders are those who lead the society. They must have well-defined principles and
objectives in order to support their stand. Their objectives must be concerned towards the
welfare of the people. If they happen to compromise on their belief based on public opinion, then
it will only lead to confusion on the part of their credibility. There are many examples in realtime
to illustrate that political leaders who have been stubborn in their opinion are remembered for a
long time in the history.

First of all, political leaders who adhere to a cause are venerated by the people and they become
the cynosure. They are deemed as heroes by the people who in many ways try to emulate their
behavior and consider them as their role models. Therefore, stricking to a particular cause often
gains acclaim for the political leaders and they become icons in due course of time. For example,
Mahatma Gandhi who vouched for independence through non-violence, amidst lot of protests, is
remembered even today by millions of Indians. This is solely because, Mahatma Gandhi has
sticked to his belief throughout his lifetime and has achieved success too.

Secondly, political leaders who never comprise on the principles are considered to be credible
and they can be trusted to eradicate the miseries of the people. Whereas if a political leader
happens to heed to the desires of the people, then he will least likely be considered as an
exemplary person; his morality will be doubted and ultimately he will turn out to be the most
hated person in the society. Hence trust can be established in a leader, only when he moves
according to the established principles and rarely does change them. For instance, Abraham
Lincoln is considered to be an Africans because he struggled a lot in rescuing the Africans from
the clutches of slavery. Had he compromised and discontinued his struggle, neither would
Africans have achieved independence nor would Abraham Lincoln have created a mark in the
history.

Finally, a leader is shaped up by his principles and objectives. At any point of time, if he changes
his beliefs, then it will mar his position in the society and create chaos because people will not be
able to discern his real character. For instance, Mahatma Gandhi can be rendered as the "Symbol
of Peace" - because in no case, did he resort to violence; Hilter who promoted Nazism and
Mussolini who promoted Fascism can be regarded as the great leaders who "achieved success by
struggling through violence"; the current President of India - Dr.Manmohan Singh can be
regarded as the "best economist cum politician" because he is always punctilious in making
economic decisions in Political matters. Thus each and every leader is identified uniquely by
their opinions and the principles that they adhere to. Consequently, a political leader creates an
'idiosyncracy' for himself and marches towards the path of success.

Though consistency in sticking to principles and objectives is required, some may argue that, in
exigent cases where no other alternatives exist, we may have to change the principles so as to
suit the needs of the current population. It is true to a certain extent that, there is absolutely no
use of keeping a principle which no longer does hold valid. As times are changing, the principles
need to be changed. But they must not occur at the expense of people's suggestions. An effective
leader is one who considers multiple points of view regarding a politcy change, analyses their
pros and cons, and finally arrives at a decision on whether to really manipulate the existing laws.
For instance, an Indian rule states that "Child labor is defined as employing the children whose
age is below 14 years"; But now, the trends have changed and everyone need to have education
atleast until 16 to 18 years of age. Hence, the political leader can take steps to accurately set the
standards for the new definition of child labor.

All in all, an effective leader in any society is identified by his commitments towards principles
and objectives; it creates a clear picture of the leader and engenders trust on him in the minds of
the people. He who stubbornly adheres to his belief is sure to create indelible impressions in the
annals of the nation's history. Though, the leaders can afford to change the principles sometimes,
it must never go against his established motives that defines his character. Therefore, great
political leaders are always those who have established set of opinions.
Formal education tends to restrain our minds and spirits rather than set them free.
Write a response in which you discuss the extent to which you agree or disagree with the
statement and explain your reasoning for the position you take. In developing and supporting
your position, you should consider ways in which the statement might or might not hold true and
explain how these considerations shape your position.

Education is of vital importance. It has the power to grant one the basic needs of living like -
food, shelter and clothing and contribute to the development of the country. Indeed, many
problems existing across the world can be solved, if formal education is made compulsory across
the world. Formal education has become a prerequisite if anyone needs to succeed in life. It
kindles our spirits in an orderly manner and encourages us to reach great heights. It serves as the
right platform for the children to bring out their talents. Whether formal education restrains our
minds and minds rather than set them free, is a contentious issue. A stand on this issue can be
taken, only after a scrupulous analysis.

Firstly, children ought to be aware of their environment and they must learn to be sociable. All
this can happen only in a formal education environment. The importance of subjects taught in
school cannot be discounted. Even if some children feel difficult to learn certain subjects, they
must do so. For instance, proficiency in English is a necessity to succeed in this globalized
world. Moreover, expertise in one's own local language is also important to move freely with the
people. Social studies involving history and geography, helps the children to be aware of their
environment and intrigues patriotism in them. Above all, the subjects concerned with Science
and Maths are the most important, since majority of the children in future will choose to pursue
either Engineering or Medicine. These subjects, when taught in a formal environment, are
imbibed into the minds of children. These ideas will surely help them when they venture out to
the real world.

Secondly, formal education teaches the students how to be disciplined and act accordingly. For
instance, children in schools often complain of homework burden. But, they somehow manage to
finish their homework fearing authority and the consequences. This instills the quality of respect
in them and also the requisite of doing the activities without considerable procrastination.
Moreover, in most of the formal education environments, time-tables are strictly adhered to. This
will intimate the students to plan out their work and work out their plan. Consequently, the
students study at a normal pace everyday and speed up their pace once some exams near them.
Thus, prioritization of work and allotment of time to the most important activity are the other
great concepts learned indirectly by undergoing formal education. All these are done in order to
execute the desired tasks smoothly without tension. Therefore, the students will be set free rather
than feeling difficult on how to proceed.

The status of formal education today is not perfect and needs some improvement. On the other
hand, it does not mean that, we can do away with the concept of formal education. Some people
feel that their children are denied their interests through formal education. In the reality, formal
education opens up multiple pathways which a student can consider to pursue. As a result, it is
aimed at bringing out the innate talents of the students in the long run and help them realize their
true potential. It is sad to note that formal education in some schools in countries like India,
emphasize more on memorizing and reproducing the facts. While it is true that basic concepts are
required for doing latter research and for thinking out of the box, it does not imply rote-
memorization be followed. These kinds of minor flaws in formal education across the world
needs to be corrected to clear people of many misconceptions.

Thus, formal education, in most cases frees our minds and spirits and bestows upon us the ability
to think clearly and logically. A person devoid of formal education, is prone to misunderstand
facts, figures and, concepts. On the whole, formal education is essential for the following: being
aware of our alma-mater; being disciplined through practicing good habits; thinking out of the
box and contribute to the society through fruitful research works. The United States of America
can be quoted as the best example for a country that has soared to great heights in research-
works in all areas, primarily because of its well-structured formal education. In conclusion,
formal education is a boon and not a bane at any cost.

The well-being of a society is enhanced when many of its people question authority.
Write a response in which you discuss the extent to which you agree or disagree with the
statement and explain your reasoning for the position you take. In developing and supporting
your position, you should consider ways in which the statement might or might not hold true and
explain how these considerations shape your position.

When humans started living in clusters, there was a need to address the needs of the people
without encroaching of the rights of others. Hence, came in the idea of governing. The
government changed in forms over the time, from heads of small tribal units to rulers to elected
democracies. But, the underlying motive of forming any governing body was the same; well-
being of society. To ensure the same, the entire society must participate and be watchful of the
governing body by constant questioning and demanding for reforms.

Since the reason for any government body is welfare of the people, it is the people that should be
in charge of who governs them. Therefore, the slogan for democracies is: of the people, by the
people, for the people. Hence, it is the basic right of the people in any country to choose who will
lead them. But, with every right, there comes a duty. It is the duty of the same people to question
the authority chosen for all steps they take. For a better governing body, all people should
participate, either directly or indirectly.

Power has the tendency to corrupt people. When Hitler was chosen by the Germans to govern
them, he became the only person governing them. He abolished the system of government and
no one questioned him for his ways. Although, in his example, he had a lot of supporters, yet the
people who were against him ignored his policies long enough to let him concentrate that much
power in his hands, that he would crush everyone who questioned him for years to come. This
led to him forming policies that were against the welfare and interest of the masses.

Concentration of power in the hands of a few leads to monarchy, corruption and serving biased
interests. To avoid the same, people should stay vigilant, and active in all policies that are made.
They should be aware of the activities of the authority and should question the same when they
feel cheated or confused. Wrong policies should be immediately reformed. In cases where the
authority becomes monarchy, people should revolt the same people they chose.

Everyone is responsible for themselves. Hence, even after electing the people to govern them, it
is still the responsibility of the people to ensure that the authority is doing what they are made
for. Only vigilant people make a better country.

Governments should focus on solving the immediate problems of today rather than on trying to
solve the anticipated problems of the future.
Write a response in which you discuss the extent to which you agree or disagree with the
recommendation and explain your reasoning for the position you take. In developing and
supporting your position, describe specific circumstances in which adopting the
recommendation would or would not be advantageous and explain how these examples shape
your position.
The assertion that governments should focus on solving the immediate problems of today rather
than on trying to solve the anticipated problems of the future is a controversial one. On one hand,
it is very important to solve current issues hampering the country instead of focusing on the
future problems. On the other hand, if government pays attention to anticipated problems, than it
would be very much helpful in developing the nation as more problems can be avoided in future
if they are sorted out on very first hand. However in the final analysis, it is much more beneficial
if government put forwards its resources in solving the vulnerable present and on-going issues
rather than putting in much more effort for anticipated ones.

Firstly, it is well known that today is the best time to begin any work. Government should
undoubtedly focus on current issues and use their resources to curb them. For example, issues
like poverty, education, malnutrition, etcetera should be addressed first instead of putting more
focus on long term planning. If current issues of malnutrition are not looked upon with more
importance than in coming days, it will convert into a bigger health issue in near future. If proper
focus is not put on educational reforms then it will prove to be serious blow to literacy rate.

Moreover, government should currently focus on funding the employment sector more and more
rather than increasing funding for scientific research for making innovations that would be
helpful in future to avoid the rise of unemployment on larger scale. Similarly, government should
prioritise its focus any unfortunate calamity prone area more than developing any non-affected
region. Also, issues like fuel crises, economic and social reforms await immediate action and
planning. These issues on not being looked upon properly in current time would hamper any
nation’s progress in upcoming years.

However, focusing on future problems is also of very much importance in this modern world.
Focusing on anticipated issues and problems created due to global warming or any natural
calamities is equally important. Being oblivious of such problems would lead to more and more
chaos in the country. For example, it is far more tactical to take constructive actions for issues
like flood or earthquake rather than waiting for it and then spending more money and manpower
to overcome it when it actually occurs. For example, defence planning should be done by
anticipating future issues that could strike the nation. One cannot wait for any rival country to
attack with lethal weapon. Instead, scientific research should be triggered to overcome such
issues.
To sum up, it is necessary that a country focuses on anticipated problems of future but it is very
much important to look into present problems first because if present problems are overlooked,
then they would harbinger bigger future chaos and problems.
Some people believe that college students should consider only their own talents and interests
when choosing a field of study. Others believe that college students should base their choice of a
field of study on the availability of jobs in that field.
Write a response in which you discuss which view more closely aligns with your own position
and explain your reasoning for the position you take. In developing and supporting your
position, you should address both of the views presented.

Should college students base their choice of a field of study on the availability of jobs in that
field or should they consider only their own talents and interests? While the availability of jobs
in a specific field of study may offer more return in terms of money and job options, it may also
result in professional disappointment. In my view, due to the myriad of possibilities our
globalized world offers us, a balance between personal interest and jobs availability should be
sought according to individual values and priorities one might have.

Despite the assertions of many that the availability of jobs is the main aspect in college students
choice of a field of study, other aspects must also be taken in account. The advantages of basing
this choice on the jobs market is that as more jobs are available in a specific area, the jobseeker
will probably have more options regarding places to live, companies to work at, work
environment and work-life balance. Additionally to these options, a field of expertise with higher
demand for professionals will also offer better salaries than those where jobs are not so much
available. The disadvantages of such a choice of profession is that life after all may become
nonsense or boring because at the end of the day you may feel like you did not do your best as a
contributor to the world, or because you spent most of your time doing something you do not
really like.

On the other hand, choosing a field based on personal interests and talents will probably bring a
more enjoyable work activity and this will reflect on personal life as something that motivates
you. Also, one's skilfulness in a field of expertise will surely make him a better professional,
which is able to contribute to the world with the best one can offer. The motivation that a
realized professional can have is sometimes priceless. The bad side of it is that the choice of field
of study based on interest may result in times of struggling on finding a job and the offers will
surely be more limited.
Ultimately, after considering both sides of the issue, it must be concluded that a balance between
personal interest and jobs availability should be sought according to individual values and
priorities one might have. The globalized world we live in today offers us a multitude of
possibilities of fields of study, so that many options are available in all the different areas of
expertise
Laws should be flexible enough to take account of various circumstances, times, and places.
Write a response in which you discuss the extent to which you agree or disagree with the
statement and explain your reasoning for the position you take. In developing and supporting
your position, you should consider ways in which the statement might or might not hold true and
explain how these considerations shape your position.

Law, by definition, is that which the regulations that keep the peace and prosperous of a society.
Proponent of the view that law need to be more flexible, believe that law need to consider
different circumstances, times and places. However, a more flexible law,to large extent, will
mean that the criminals have the possibilities to get away with the punishments and sentences.
After weighting the evidence, it is certain that laws need to be rigid and redoubtable to the
criminals.

Consider the example of the legimitations in Suviet Union, which used to be considered as a
lawless country. The government announced that the people in power were , on the basis of
security, more flexible when they were in chareged with any crimes. Consequentky,many party's
leaders were not chareged with corruption when they received prodigious amnout of wealth in
varuous ilegal ways. The entire authorities was in debauchery and was overthrown by its people.
It showed that the flexible of laws led to the dissolution of a society and its people.

The government creat laws to keep its civilians' safety and prosperities. The flexible laws
enhances criminals' opportunities to threaten other's safety and properties, which will unstablize
the development of a society, particularly when no one feels sense of safety. More importantly, if
such a policy were to be implemented,the people in power would intend to apply the policy when
they broke the laws. Both of the cases mentioned above would, to a large extent, casue the
disorder of a society.

Furthermore, decideing when and how to be flexible to the criminals is diffcult. Different judges
would have different judgements on the basis of the flexiable laws, particularly, it easier to casue
plunderations when the cases are more complicated.
In final analysis, it is clear that such a statement cannot be followed. To consider the various of
circumstances, times and places for the laws, more emphasis need to be on the whether it is
against the credo of legimiation other than casually demanding of the flexiable.
Claim: The best way to understand the character of a society is to examine the character of the
men and women that the society chooses as its heroes or its role models.
Reason: Heroes and role models reveal a society's highest ideals.
Write a response in which you discuss the extent to which you agree or disagree with the claim
and the reason on which that claim is based.
A society is not a singular entity; rather a multidimensional, multi-layer function. In order to
comprehend its complex characteristics, it is important to take into account the diverse societal
elements in addition to studying men and women who are chosen as idols and heroes.

The complex societal function is defined by parameters such as its people, culture, traditions,
beliefs, education, politics, laws, food habits, architecture and a myriad of factors.  If an attempt
is made to judge the character of a society, as claimed, by examining the characters of role
models and heroes the society idolize, it might just provide us a façade. In order to comprehend a
society, it is deemed necessary to examine in greater depths by studying the above mentioned
factors.

Undoubtedly, heroes and role models of a society represent the pinnacle of ideals and morality
that people should embrace, but is no way a direct reflection of the entire society at large. The
society’s most important element is its people, along with their different attributes and diverse
properties. For a clearer image of the societal characteristics, one must study how people within
the society vary, and their interaction with other people or within the community. A greater
understanding of their food habits, traditions and beliefs, studies of festivals celebrated and Gods
worshipped will throw a light on the cultural and spiritual sides of the society. Investigating into
art and architecture of a society will help fathom the creative aspects while probing into
education, laws and politics as practised by the people will give us a clearer picture of the
maturity of the society. Hence, examining in greater details the social, educational, spiritual,
economic and political aspects of a society shed more light in the study of a society.

Moreover, as heroes and role models climb up the rung of the societal hierarchy, enjoying the
popularity of the people and basking in the glory of admiration, they might fail to echo the
characteristics of the strata in which they have their roots. Studying Barrack Obama’s
characteristics now will not reflect his humble beginnings of his past. Neither will Mother
Teresa’s devotion to the poorest people in India echo her affluent society she belonged to, nor
Brutus’ betrayal reflect the “honour before death” axiom of the Romans. Thus, studying heroes
and role models will prove to be a misnomer. However, after a society has been studied upon, the
characteristics of heroes and role models may be used to emphasize and determine how
culturally vast and rich the society is.
Thus, to delve deeper into the characteristics of a society, one must undertake to study in depth
all the subtle, implicit factors in addition to major parameters. Only a clear understanding of
every element and its interaction and contribution to the society will enhance the interpretation of
society’s characteristics.

Governments should place few, if any, restrictions on scientific research and development.
Write a response in which you discuss the extent to which you agree or disagree with the
recommendation and explain your reasoning for the position you take. In developing and
supporting your position, describe specific circumstances in which adopting the
recommendation would or would not be advantageous and explain how these examples shape
your position.

Science and developments flourish where they are supported and researches are conducted with
freedom. As one can observe from the history, societies live their best periods in an environment
that promotes free-thinking. Improvements in science can be achieved if it is not restricted by the
government. In addition, development of science would enable the society to reach a better level.
In the following paragraphs, the reasons for governments should not restrict the science will be
discussed.

To begin with, there might be situations when governments should limit the scope of scientific
applications and researches. The researches that might harm human dignity or aim to produce
chemical weapons should be banned. For example, developments in genetically engineering
reached to a position where it is possible to clone a human. Even though these new technologies
are amazing, they might produce serious consequences. People might desire to use them for
harvesting the organs of clones. This aim is not parallel with human rights then it should not be
permitted. Moreover, chemical weapons might have disastrous effects as in the case of
Hiroshima. As a result, any form of usage and development of weapons that might affect such a
large population should be banned.

In addition, although there might be some restrictions as stated above, they should be few. The
countries where there are only few restrictions on science might attract the researchers. As a
result these countries would enjoy the economic benefits of these researches. Researchers and
scientists are most likely to prefer where they are supported and free to focus on whatever they
find interesting and promising. For example, there are severe restrictions on genetic researches in
United States and many European countries. Beside, Singapore does not ban any research and
they create a supporting environment for the researchers. Consequently, several eminent
professors and many promising young scientists who specialize on genetic choose to dwell on
Singapore. They conduct important researches on stem cell. Hence, any revenue that might be
generated as a result of developments in these areas would be harvested by Singapore. Hence,
allowing to scientific researches would increase the revenue and prestige of governments.

Finally, governments might prefer to restrict scientific researches under the name of showing
respect to religious feelings of the populace. However, the aims of religious people is most likely
to protect their authority and do not allow people to enlighten. For example, in Middle Ages
Roman Catholic Church put many restrictions on scientific researches. They did not permit any
research that might contradict with the doctrines of Bible. For instance, Galileo claimed that the
Earth revolves around the Sun. Even though thinking in the opposite way is impossible today,
the Catholic Church disclaimed the assertion and they conducted a trial to punish Galileo and
pushed him to agree with the premises in the Bible. As in the case of the example, if these
restrictions had continued to exist, then the technologies such as sending satellites to the space
would not be possible today.

To sum up, there might be some cases harmful for humanity that need to restricted. However, the
restrictions should not be expanded since they would generate economic value for governments
in the long term. Furthermore, the attempts to restrict the researches to not to insult the religious
feelings of the people is not logical because religious authorities might use these restrictions to
pursue their rein. As a result, there should be only few restrictions on science.

The best way to teach is to praise positive actions and ignore negative ones.
Write a response in which you discuss the extent to which you agree or disagree with the
statement and explain your reasoning for the position you take. In developing and supporting
your position, you should consider ways in which the statement might or might not hold true and
explain how these considerations shape your position.

It is often said that the best way to teach is to praise positive actions and ignore the negative
ones. At first glance it may appear to be true. Some may agree with the author. To make
somebody more confidence in what he is very good at, we need to ignore his little negative
actions. However, I disagree with the above premise. The reason for my stand will be elaborated
in the subsequent paragraph.

First, teachers play a very crucial role in the making of an gentleman. A student may be very
brilliant in his studies. He gets the top rank each year. But his attitude towards his mentor and his
collegues is not good. He is very rude, abusive. These actions should not be ignored. Clearly if
these actions are ignored, he will never quit his habit of rude and abusive character. His actions
will make him recluse from his collegues and he will be left alone in his coming years. Even if
he is a brilliant student, his negative actions should be be taken into account and be taught that
these behaviour or attitude is wrong.

Second, apart from education, parents also plays a very pivot role in thier making of children's
character. Any wrong traits should be immidiately rectify. Suppose a kid is very good in
behaviour and everyone praise his character. But he is having a habit of stealing money. Clearly
these traits should not be neglected. Parents should teach thier children about what is good or
bad. It's true that as a kid he might not know what is good od bad so it's the responsibility of the
parents to teach them about good and bad.

Third, it's a common trend now a days that student are not willig to go to institution rather
gelling with their friend in some wrong doings. Telling lies and bunking classes should not be
tolerated. Some strict actions from the school administration should be taken against these kids.
Not only actions but they should also be taught about the importance of classes and not to bunk
the classes.

To conclude, the ideas illustrated above, justify the stand that we need to take actions on the
individual negative actions. It is not always neccessary that we need to be very harsh on the
negative actions but we need to remind them that with best possible ways that this is not correct
and they need to feel sorry for their wrong actions.
Governments should offer college and university education free of charge to all students.
Write a response in which you discuss the extent to which you agree or disagree with the
recommendation and explain your reasoning for the position you take. In developing and
supporting your position, describe specific circumstances in which adopting the
recommendation would or would not be advantageous and explain how these examples shape
your position.

Education is of utmost importance to an individual in this competitive world of ours. It is almost


impossible to survive in this world without proper education. Illiteracy is the root cause of all
corruption and crimes happening all over the world. Governments do play a major role in
financing a part of the education fee for the poor students. However, this issue of providing
college and university education free of charge to everyone is complex. A thorough analysis
must be undertaken before arriving at a clear-cut decision.
While it is true that many people all over the world are unable to finance their higher education,
it does not necessarily imply that the governments can offer university and college education free
of cost. First of all, money is everything in today's world. The costs of necessary commodities to
lead a honorable life is going up everyday; so is the case with education. Generally, teaching is
regarded a service and is considered with high respect. But that does not mean that teachers will
be able to educate students without receiving proper compensation package. Hence, if university
education is offered free of charge, then the mutually beneficial relationship between teachers
and students will be severely affected. For example, there exists Government schools and Private
schools in many parts of the world. The Private schools charge a little higher than those of the
Government ones, because they hire highly qualified teachers, who are willing to contribute their
service for a higher pay. Colleges and Universities will not run properly, if the necessary funds
don't come from students. Even if they are run by some trusts, there is a limit upto which
contribution can be done. Beyond a certain level, students must make monetary contribution for
the service they receive. If not, then the ultimate aim of achieving a wholesome education will
not be attained. Moreover, Governments cannot fund all the students because it has certain
amount allocated for higher education sector. They have the responsibility of striking balance
between a variety of factors that contribute to their national development. In most of the cases,
the Governments spend as per the decide budget and hence it seems impractical to provide free
education. For example, the United States Government sets apart a certain amount of money for
funding international students. If a university fees exceeds that amount, then obviously the
excess had to be paid by the student.

Second of all, quality education matters a lot to all students. As of now, we have cut-throat
competition for any kind of job, out there in the market. Unless an individual has clear
understanding of concepts and sufficient practical knowledge, it will be impossible for him to
survive the competition. As a result, introduction and familiarization of new technology to
students is of paramount importance. For example, recently the newer version of Windows titled
'Windows-8' was released; it has lots of new features like touch-screen access, efficient memory
management facilities, exemplary user-friendliness etc... But, all over the world, this operating
system found its way first into the premier institutions which are private in nature. It will take
years for the changes to be updated in the Government funded institutions. On the other hand,
these technical advances can be let known to a wider range of populace, if students monetarily
render a contribution to their college or university. Even the methods of teaching are dependent
upon the money available at the disposal of the college. For instance, the Smart-Classrooms
involving projectors were to be introduced all over India by the Government. But only the
private institutions which collected funds from the students, were able to successfully install
these systems. Therefore, to update the students with the latest trends and confer them with
innovative teaching methodologies relies upon the amount of money that students can spend
from their pockets.

Above all, some may state that there are lots of unfortunate people who are unable to even look
after their daily needs. Hence, they might claim that, these students must be offered free higher
education. But this claim has lots of fallacies. First of all, it must be understood that, financing a
student does not stop with paying the college

The luxuries and conveniences of contemporary life prevent people from developing into truly
strong and independent individuals.
Write a response in which you discuss the extent to which you agree or disagree with the
statement and explain your reasoning for the position you take. In developing and supporting
your position, you should consider ways in which the statement might or might not hold true and
explain how these considerations shape your position.

It is hard to imagine a life without the internet, smartphones, electricity, dish washer, washing
machine, air-conditioner and other conveniences. The invention of these tools has made living
easier and reduces stress. I do not have to leave for England three months before to attend a
conference because i can catch a 6 hour flight thus giving me more time to complete my
research. I disagree with the above claim that the conveniences of contemporary life enables us
to become independent individuals.

The emergence of technology and devices such as electricity, internet, cars, good healthcare has
improved the quality of life and life expectancy of individuals. This is because individuals can
devote more time to developing their human faculties rather than spending time on mundane
things. For example, before the industralization era, families spent their time on their farms and
lived subsistent lives. Life for them was brutish as emergences such as diseases were seen as
punishment from gods that had to be appeased. During and after the industrialization process,
things changed. As living standards improved, thought processes and mindsets also changed.
Individuals began to diversify and improved their quality of life. Other occupations apart from
traditional occupations emerged.
Modern day conveniences has also given individuals the time and ability to explore the world
they live in and make it better. For example, research is being conducted on how to tackle issues
such as global warming.

The luxuries and conveniences of contemporary life has also helped women come into their own.
The role of women was limited to their homes. However, women and their roles have evolved as
a result of the conveniences of modern day life. For example, a woman does not have to spend
all day toiling in the kitchen. Devices such as the microwave, cooker and mixer has made it
easier to cook such that she has the time to attend school and have a career whilst fulfiling her
role as a home- maker. Relatedly, women are now seen as comtemporaries of men rather than the
weaker sex because they have been able to grow. This might not have been possible if we were
still stuck in the time when women had to spend most of their time taking care of their familiies.

Nevertheless, the improved standard of living and situation of machines are the beck and call of
humans may have made us lazy. Take for instance the use of calculators to solve mental sums or
the use of phone reminders and alarms. Lore states that our fore fathers were more developed
than we are now because they lacked all these tools and had to survive.

In conclusion, the comforts of modern day life has helped human beings to develop and utilize
their God-given abilities whilst at the same time made them lazy. However, individuals are
better-off with these conveniences rather than without them

In any field of inquiry, the beginner is more likely than the expert to make important
contributions.
Write a response in which you discuss the extent to which you agree or disagree with the
statement and explain your reasoning for the position you take. In developing and supporting
your position, you should consider ways in which the statement might or might not hold true and
explain how these considerations shape your position.
Beginners are full of potentials to create new ideas as they are more likely to be free from
established habits and attitudes. On the other hand, experts are wise to make use of experiences
they have acquired through a variety of errors and times. Both have played a vital role in
advancing our society in history. However, I truly believe that in most fields that constantly need
inspiration and are filled with new thoughts, experts are the ones who make more significant
contributions.
Granted, in business industry, where state-of-the-art products are required to be created without
pauses, it is likely that beginners are more instilled with a sense of entrepreneurship. They are
not reluctant to take risks and make numerous tries until success since they have nothing to lose.
Take example for Steve Jobs, best known as a founder and CEO of Apple Inc. He started his
business out of his garage with a minimal equipment after dropping out college. He
experimented on mobile phones and computers by incorporating his calligraphy knowledge into
their design, which yielded highly popular products such as iPhones and Mac series. This could
have not come out from hands of IT technicians who could not see beyond functions of mobile
phones and computers. Despite a lack of expertise and resources, he could make such
accomplishments by fully using his imagination and unrefined ideas.
As more and more complex our society becomes, however, solid knowledge base and expertise
are required to advance our society and make significant progress. In particular, science does
need more experts who have a deep comprehension of existing theories, ideas, and structures.
Consider the Theory of Relativity proposed by Einstein. The theory transformed theoretical
physics and astronomy, superseding a 200-year-old theory of mechanics created by Newton.
Einstein was already a distinguished physicist when he published two interwoven theories of
relativity. Without profound understanding of the law of gravitation and its relation to other
forces in the universe, the remarkable progress in physics such as discovery of space-time could
have not made, not to mention that nuclear energy production and synchronization of GPS are
compelling examples of his theory application.
Moreover, in the political field, experts are more capable at coping with challenges based on
experiences and trials they have made through age of times. Take example for the United States
presidential history. The leaders who weathered through national crisis were those who could use
their expertise to guide the nation wisely, such as Franklin D. Roosevelt and Barak Obama. The
Great Depression in the 1930s and the Global Financial Crisis in 2008-2009, respectively, were
the most challenging tasks they had to resolve. Both administrations took extensive measures to
alleviate economic recessions by implementing New Deal programs. During national hardships,
each of them took consecutive terms of presidency since they were acknowledged by their
citizens that their incumbent presidents were competent to take care of their livelihood. With
more accumulated expertise and comprehensive knowledge of what role presidents should play,
they contributed to maintaining the nation as the world’s political and economic titan.
In summary, although the beginner is likely to foster positive contributions with fresh ideas and
risk-taking entrepreneurship, it does not hold true in most fields. Experts deserve
acknowledgement since they are the most competent in their fields.
The surest indicator of a great nation is represented not by the achievements of its rulers, artists,
or scientists, but by the general welfare of its people.
Write a response in which you discuss the extent to which you agree or disagree with the
statement and explain your reasoning for the position you take. In developing and supporting
your position, you should consider ways in which the statement might or might not hold true and
explain how these considerations shape your position.

Nations assume greatness for different reasons such as size, wealth, previous world domination,
achievements of citizens, and charisma of rulers. Hence, the above assumption is extreme. This
is because the term greatness has various interpretations and the indicators of a great nation are
not subject to precise measurements.
One might argue that China, India or America are great nations because of their size and others
would reject that claim and state that they are not great cause human rights are violated in China
and the United States cannot reach a consensus on universal healthcare. Therefore, the
assumption that the greatest of a nation can be measured in a precise manner is flawed.

However, America is an example of a great nation, it is a great nation not because of the
achievement of rulers or the welfare of the people but because of its values and culture as a
country where dreams and hopes are realized. This also shows that measuring greatness is
subjective.Also, countries like Britain are regarded as great because it colonized many countries.
Furthermore, Western European countries assumed greatness because they gave the world
scholars, poets and ineventions.

Pre-industrailized Britain was regarded as great despite the poverty and lack experienced by
citizens. Similarly Communist Russia focussed on the welfare of its people. Rather than being
regarded as a great nation, it was despised as an enemy of the world n addition, countries like
Norway and Sweden where the citizens experience high standards of living are not regarded as
great. Thus, the assumption that greatness can be measured by the standard of living of her
people is flawed.

Nevertheless, some countries like Dubai and Singapore are regarded as great because the leaders
of these countries drove development that lea to the welfare of the people.

In conclusion, the above assumption would not be valid in all cases because measuring greatness
is not precise. Some nations are regarded as great because it excels in sports, the scholarship and
innovations it has bequeath the world, size, sometimes because of the general welfare of the
people.

The best way to teach — whether as an educator, employer, or parent — is to praise positive
actions and ignore negative ones.
Write a response in which you discuss the extent to which you agree or disagree with the claim.
In developing and supporting your position, be sure to address the most compelling reasons
and/or examples that could be used to challenge your position.

who is a teacher?? a teacher is a person who helps us learn something,gives us knowledge and
puts us on the right path when we are heading towards the wrong path.he is the person who
makes us capable of deciding the right from wrong.the one who leads us from darkness to
light,from mortality to immortality is a teacher.a teacher need not be only a school teacher.it can
be anyone around us like our parents,educator,friends,relatives or even an unknown well-wisher.

from the times a child is born he starts learning new things observing his surroundings.the first
stage of learning is at home where the child's parents are the primary teachers.the child learns to
speak,walk and behave from this point of time.it is a crucial time when the child has to be told
about the pro and cons of things.from here on-wards the child will find different people as
teachers at different stages in his life.the main strategy of the teacher should be to cater the
student with verve and vitality.in order to do this the teacher should entice the child so that he
doesn't loose interest in learning things.but that doesn't mean that the child should always be
praised for the positive actions of his and neglect or ignore the negative ones.i agree to the point
that the child should be encouraged for his good deeds but ignoring the bad deeds would give
him an education which is unfruitful.

the child would rather become indecisive with the half knowledge being provided.this would
drastically effect the child's character and would he would become recalcitrant at a particular
stage of life.encouragement is the best tool to make a child succeed in what field he has chosen
for life but not at the cost of his degraded character.for example if a child,during his childhood, is
playing caroms with his friends under the guidance of his parent.he won the game by cheating
his friend.the parent does know that what actually has happened.but in order to keep his child's
spirit he cheers the child for winning the game and ignores to teach him play a fare game.this
particular instance gives the child a pace wherein he develops the character of cheating in order
to get what he wants.this would ultimately change the future life of the child.had the child been
shown the correct path he would have a better life.

in sum,i strongly oppose the fact that a teacher should praise positive actions and ignore the
negative ones.this would make the child recalcitrant and intractable.its blatant to give such type
of knowledge.

Teachers' salaries should be based on their students' academic performance.


Write a response in which you discuss the extent to which you agree or disagree with the claim.
In developing and supporting your position, be sure to address the most compelling reasons
and/or examples that could be used to challenge your position.
In most societies teachers are considered one of the most important people in a person’s
life.Teachers make a major contribution in shaping a student’s life for better future.Hence it is
very important for a student that he is getting the most accurate knowledge and guidance from a
distinguished teacher.Therefore many people might consider that a teacher’s salary should be
decided based on a student’s performance.However,according to me it is unjustified to put this
opinion into implementation due to reasons mentioned below.

Each person has its own unique personality and talent that sets him/her apart from others.Some
students are very bright while others give mediocre output.It would be injustice to blame the
teacher for an abysmal performance of a student,even though when the teacher might have tried
to the best of his abilities to rectify the students infirmities. We cannot expect all the students to
do equally well in any kind of activity.There will always be few failures,else we won’t be able to
differentiate between success and failure.There could be cases where in a class of 50 students
,only two have failed to give the desired performance,so because of two failures would be it right
to lower the salary of the teacher?.Absolutely no,primarily because not all students are equally
talented by nature and the most important responsibility of a teacher is to help students shape
their talent to achieve future success.

Further,one might posit that deciding a teacher’s salary based on a student’s performance will be
a motivating factor to perform better.However,this could be counterproductive to a teacher’s
performance.Suppose even after delivering the best performance,there are students who failed to
give adequate results due to negligence or lack of seriousness on their part and a teacher’s salary
is reduced due to that,he/she might get demotivated to perform better and hence might go into a
state of dejection and helplessness.Hence it would be a cruel act to put the whole blame on a
teacher for a student’s fallibility.

As not all students are equally bright,not all teachers are also equally good at teaching.Hence one
might argue that reducing the salary of the teacher is the only best option to enhance a teacher’s
performance.However,there are other alternatives like taking student feedback periodically on
every teacher.This will even help the teacher to know what are their flaws and what is expected
out of them by their students.Such feedbacks will help teacher’s understand their students and
hence will improve their performance for better.Instead of resorting to salary decrease for a
teacher’s lack of performance,the alternative of taking student’s feedback is more contributive
towards helping teacher ‘s and students get the best out of their relationship.
In sum, teachers cannot vouch for a student’s success. Deciding the salary of a teacher based on
a student’s performance is an harmful measure for both teachers and students.Other alternatives
of students feedback or even feedback from contemporaries would be more fruitful towards
getting the desired performance from each teacher.

Society should make efforts to save endangered species only if the potential extinction of those
species is the result of human activities.
Write a response in which you discuss your views on the policy and explain your reasoning for
the position you take. In developing and supporting your position, you should consider the
possible consequences of implementing the policy and explain how these consequences shape
your position.

How far should we as a society attempt to save the endangered species? Should we help them
only in case we have caused the harm or should we help them because we understand their pain ?
There are many species which have died due to human encroachment, and suffered a great loss.
While human population has increased from 2 billion to 8 billion in last 200 years, many species
have died because of our attack on natural habitat. At least now we should realize that we should
come forward and revive them back into this world, independent of the fact who caused the
harm. This is only possible by implementing proper policy and covering all regions.

Consider the Indian tiger, which are found in western parts of India, there population came down
to 1000 in the recent years and still decreasing. They are killed for their skin and various body
parts which are used for different purposes, mostly for superstition and orthodox beliefs. They
might be predators but they are balancing the food chain and natural life cycle. It may get
imbalanced due to human intervention. These species hunt the weak links of the wild part and
keep the wild life healthy. If we as policy makers doesn't save them by imposing sever
punishments on the hunters, then the whole wild life balance may change and overall health of
other species may get affected.

Also, because of global warming many other polar regions are getting affected. Even we are
directly affecting those regions but these temperature rise is the causing many polar wild life a
grave danger. We as a society try to take steps to overcome this environment change, by
encouraging more eco-friendly products than petroleum products. This may cause little
disturbance in present market functionality but if not done today then it may cost more in future,
by some estimates the expenses to undo these effect will cost many more trillions dollars in 2100
than today.

Not only the terestrial life need to be considered, the aquatic life which is seriously hampered by
human presence. The rise in water temperature caused loss many coral reefs, and immense
fishing has left many aquatic species in state of near extinction. Not only their life is disturbed
directly but also indireclty, as in case of coral reefs. Only correct policy measures to stop fishing
of some species and limiting the fishing troops can save them, all this will cause temporary
disruption but will bear fruits in long term.

Ultimately, from the bulk of evidence, we can conclude that there is a urgent need of the policy
which will check the terrestrial, polar and aquatic life balance , even if it causes temporary
disruptions in our life. We are humans, who can understand others pain, thus it is our duty to
save them by bearing some pain.

College students should base their choice of a field of study on the availability of jobs in that
field.
Write a response in which you discuss the extent to which you agree or disagree with the claim.
In developing and supporting your position, be sure to address the most compelling reasons
and/or examples that could be used to challenge your position.

From my perspective, various students could have different choices about this topic. For one
thing, students with an undergraduate degree could find jobs in all kind of domains, even
different from their fields of study. For another thing, students with a graduate degree should
find jobs in their domains. In addition, all majors are actually related to each other. Furthermore,
regardless of degrees, different people should have their own choices on employment.

On the one hand, it is not inappropriate that students with an undergraduate degree find jobs in
other fields. Even though for most students, getting employed in their study of fields seems
beneficial for their career development, the job in other fields is also a considerable choice. Since
young people often have various interests in their majors, it is feasible for them to change to
other fields when they get employed. For example, students who major in mechanical
engineering, after they experience some suffering internship in the factory, find they are more
interested in programming engineering. Furthermore, most majors are different from what they
seem to be. For instance, students who major in software engineering may find that they will not
be so famous like Bill Gates or Mark Zuckerberg. In fact, most programmers are also very
toilsome.

On the other hand, it is much better for students with a graduate degree to get employed in their
study of field. Masters and PhDs usually have studied long term in their majors. If they choose
jobs without consideration of their studies, they will lose lots of advantages in the majors. Not
only will numerous courses in the profession provide the related knowledge, but also
interpersonal relationship in the industry will make it convenient to work in the study of field.
For students who major in financial engineering, they not only could trade the stock based on
their majors, but also could get abundant information from their relationships in school.

Besides, there is no discipline in the world at primitive phase, and all majors are related in fact.
People who major in civil engineering could also get a job in the software company, so that they
can help code some professional program for engineers. People who major in biology could also
get a job in the electricity engineering, so that they can help make the print-locker, voice and
facial recognition. People who in major in education could also do some research in the internet
company, so that they can help make some professional study plan based on the Internet, like
khan academy.

Moreover, regardless of degrees, individual character is more significant in this issue. Different
people have various attitudes about their career planning. Some people like to change their jobs
several times in their life, so that they could enjoy the different experience and might become a
versatile person. Meantime, some people like to drill in one field to become an expert. Both
kinds of persons are important and indispensible in the society. Students should make their
choices based on their characters and personalities.

Based on the aforementioned points, I recommend that students with graduate degree should find
jobs in their study of field. For students with undergraduate degree, it is not unsuitable to change
their fields. From a large picture, all majors are related in fact, and both all-round person and
experts are indispensible. Hence, people should get their choices, based on their own individual
characters.

As we acquire more knowledge, things do not become more comprehensible, but more complex
and mysterious.
Write a response in which you discuss the extent to which you agree or disagree with the
statement and explain your reasoning for the position you take. In developing and supporting
your position, you should consider ways in which the statement might or might not hold true and
explain how these considerations shape your position.

Currently, it is debatable whether acquiring more knowledge makes things to be more complex
and mysterious. Some people hold a view that getting more information makes things easily
understandable since if people have no information, they will not be able to comprehend
anything. However, I personally assert that having further knowledge about something makes it
more complex and mysterious to understand than comprehensible since people want to earn
deeper understanding about it.

Granted, in a handful of cases, learning makes things clear and more comprehensible. For
instance, when children first learn alphabet, they do not even know how to spell a simple word
such as apple or car. However, as children start to acquire how to write alphabet, they become
more easily learn a wide array of knowledge and begin to even read and write at an exponential
rate. Consequently, those children who once were ignorant about reading and writing are able to
access to a wide array of information and understand it more clearly than before. Therefore,
gaining more knowledge appears to make things more comprehensible than before.

However, in most of cases, acquiring more knowledge lead things to become more mysterious
and complex. To illustrate, when people have become interested in astronomy long time ago,
they were simply concern about a basic thing such as whether the earth is the center of the
universe. However, as people have become more intelligent, they are no longer curious about
this kind of simple question. Rather, astronomers nowadays want to know about things such as
black matters, which cannot even be discernable to human beings. As we can see from this
compelling example, getting more knowledge makes people to acquire even deeper
understanding of things, so that further information on something makes it to become more
complex and mysterious.

In addition, the argument that a plethora of information yields things to become more
complicated and mysterious is credible. Through the technological development, people can
acquire more and more knowledge through the Internet. Even though this is a meritorious point,
there is also a drawback. Since any people can spread their idea through the social network
system, many unverified information is prevalent on the Internet. Consequently, a mixture of
various kinds of information is gathered and engenders people to be difficult to comprehend.
Therefore, obtaining more knowledge is not leading things easily understandable.

In conclusion, in most of cases, things become more complicated and mysterious by learning or
earning knowledge. However, for the betterment of the society, we should go the extra mile to be
interested in learning.

In any situation, progress requires discussion among people who have contrasting points of view.
Write a response in which you discuss the extent to which you agree or disagree with the
statement and explain your reasoning for the position you take. In developing and supporting
your position, you should consider ways in which the statement might or might not hold true and
explain how these considerations shape your position.

Any situation in life can have different interpretations depending upon the way of perception of
its observer. An optimist may say that the glass is half full while a pessimist would say that it is
half empty. Human mind is a complex computer whose interpretation of a particular event or
situation may depend upon several complex factors like past experiances, present environment,
an individual's ideals and many other yet unthought of factors. Thus in any situation, individuals
may and do have contrasting views depending the way they interpret the situation. As a result,
starting from a situation or problem there maybe several approaches to arrive at the possible
solution depending upon the mindset of the solution provider. However, to arrive at the best
possible solution, contrasting opinions will have to be examined. Therefore, In any situation,
progress requires discussion among people who have contrasting points of view.

Each individual,depending upon his past experiances, his standing in life, his ideals and his
requirements has a particular interpretation of any situation or problem. Different individuals
perceive different aspects of a situation as well as those of its probable solutions. For example,
the elected chair person or mayor of a city may have a desire of building a bridge or flyover at a
particular place in the city to decrease traffic jams. In his own mind building the flyover may be
the most elegant solution fully worthy of the taxpayer's money that will be spent on it.
However,on closer examination analysts may find that the cost incurred in building the bridge or
flyover will not be offset by the decrease in traffic problems. The flyover may not have the
predicted effect and the taxpayer's money which could have gone in health or education will be
ill-spent on a potentially redundant structure. In this situation the mayor should listen and closely
examine the contrasting views. He should analyse if the contrasting opinions are the ones which
are more likely to be correct. Only then a proper decision will be made. By examining such
opposing views he may be able to arrive at better solutions to solve the traffic problem.

In the modern world,the democratic system is considered to be the most humane system of
governance. Such a system generally gives a two party or multi-party rule. Each such party
represents a particular school of thought, a vision of ruling and administration, a method solving
nation's problems and so on. All important decisions need an approval of majority of the
parliament which compirses both the ruling party as well as opposition.The major reason behind
including the opposition party is that they can adopt a view which can be different and at odds
with the ruling party without the fear of being regarded as traitors within the party. They can
present contratsing arguments to any important decision made by the ruling party. As a result
ruling party will be forced to consider the ways in which any decision will affect different classes
of the people.SO there will be decreased possibility of the ruling elite overlooking an important
aspect of any major national decision.

Summing up,it is clear that in progressing with any situation,heed must be paid to both
supporting and contrasting views on any path of progress. Only then all the aspects of the
situation can be considered and the situation can be understood in its entirity. Thus in any
situation, progress requires discussion among people who have contrasting points of view.

. Governments should not fund any scientific research whose consequences are unclear.
Write a response in which you discuss the extent to which you agree or disagree with the
recommendation and explain your reasoning for the position you take. In developing and
supporting your position, describe specific circumstances in which adopting the
recommendation would or would not be advantageous and explain how these examples shape
your position.

Scientific research is an important segment of scientific and social development today. The
money governments commit to the research is constantly increasing every year. In order to make
the most efficient out of this money, someone recommends that governments should only fund
the research whose consequences are clear. In my opinion, this recommendation has a good
standing point but lacks of some considerations of other important factors.

First of all, this recommendation oversees the potential of long term consequences of scientific
research. It is doubtless to me that long term outcomes are crucial, sometimes long term effects
are even more important than short term ones. Take scientific research in agriculture as an
example, if we want to test a new kind of pesticide on vegetables such as tomatoes, a significant
amount of researching and experiencing time should be expected. Not only because we need to
wait for tomato to grow, but also because we want to collect data of remaining pesticide from
offspring. It may be difficult to identify any upcoming results for scientific research like this at
first. However, governments should still support it because it helps improve the quality of food.

Moreover, in circumstances like space scientific research, the government should provide
financial support even though the outcomes are unknown. We have spent a lot of money in space
studies, for example, building up satellite spaces stations and developing new spaceships. Is all
this money meaningless? Of course not. Scientists are learning more and more about the space
though the way with these developments. If we cut the budget of space research because there is
no clear consequence, we would never understand what surrounds us, which consequently cuts
any further development in the space.

Nevertheless, this recommendation may be adapted under certain situations that require fast
results with limited budgets. Many university researches would fall under this category. Hence
this recommendation is beneficial for select the best candidates quickly; government official may
consider it upon their needs.

To sum up, I partially disagree with the recommendation that only scientific research with clear
outcomes could get funded by government. Even though this recommendation may be useful in
some situation, a detailed case by case analysis should be conducted before any action has been
committed.

Society should identify those children who have special talents and provide training for them at
an early age to develop their talents.
Write a response in which you discuss the extent to which you agree or disagree with the
recommendation and explain your reasoning for the position you take. In developing and
supporting your position, describe specific circumstances in which adopting the
recommendation would or would not be advantageous and explain how these examples shape
your position.

Many of us sometimes realize that some children have special talents. However, they train with
all other students with same education system. I agree with the statement that these children
should get special training as early as possible to develop their talents. When we put this children
to a different class, we will see their talents is getting incerase.
Recommendation of the statement is true that unfortunately many talented children oes sae
school, and get same education with other children, however, if they continue to get normal
education, their talents will be disappear by the time. On the other hand, if we put these children
to specific class, and train them with their own talent, we will create the most genious group of
people in the future. Society did discreminate many genious people in the past, Bethoven,
Einstein, Van Gogh, etc. They thought there is some anomality with this children, mostly they
start to speak late, they were not good at Mathematic, their relation was not good with other
people also. However, nobady think about maybe these child have some special talent, maybe
they can't speak early, but they can paint wonderful. Maybe they haven't discovered their genious
math, maybe they will be a good musician. Instead of eliminating these talented children, not
only obvious talent, but also we should consider anomaly children also, we should identify them,
and provide them special traning to make their genious or talent more sharp.

Moreover, if we provide special training to those children, they have a chace to train with other
talented children, rather than normal children. This is also good fot both type of children. It is
good for normal children that they don't bother these talented children, and not feeling dump
around taleted children. It is also good and beneficial for talented children that nobady abusing
them, or eliminating them from the society. Think a education place, full of talented person,
some of them talented with music, some of them dance, some of them math. Unevitably, they
will represent our society for future. They will invent something for whole world. For instance,
think we stracture a class, and its students are Einstein, Newton, Edison, etc.. We can't imagine
how successfull they will be in the future. Many of the companies do almost same thing now.
They create an team, which are talented and useful for company. And this team invent some
goods or technology, and company gets its beneficion by creating a intelligent team.

In contrast, some people may think that, if society identify talented children, and provide them
training will create some problem, such as they may think these talented children will be
antisocial, or society will create unintentionally two different group in future. Talented people
and normal people. And this discrimination will create further problems, like elites, nobels and
mid group of people. However, there is always some talented children, some of them got special
training some of them not. But they will create problem for future, they are genious people are
they want to be talked by their talent.
In conclusion, if society identify those talented children, and provide them training at an early
age, it definitely helps them to develop their talents, otherwise they will be normal people
forgetting their talent.

It is primarily through our identification with social groups that we define ourselves.
Write a response in which you discuss the extent to which you agree or disagree with the
statement and explain your reasoning for the position you take. In developing and supporting
your position, you should consider ways in which the statement might or might not hold true and
explain how these considerations shape your position.

Social groups refer to the group of people whom we have an association with. They go a long
way in help us identifying ourselves. Our character and behavior are determined to a great extent
by the social groups that we associate ourselves with. Since, it is impossible to judge a person
solely through his actions, it is by means of groups that we decipher the innate qualities and
aspirations of a person. In today's modern world, people take decisions based on discussions held
among their close friends who most probably form a group. Consequently, the influence of social
groups can be found at each and every stage of our lives.

First of all, when a child is born into a family, the persons whom the child knows are his or her
parents. As a result, the child learns new habits and develops an idiosyncracy of himself by
imbibing the qualities he likes from his or her parents. Additionally, the other people in the
child's home contribute significantly towards the development of the child's character. Thus, the
first kind of social group that a person encounters when introduced into this world is his or her
parents.

As time goes on, children move on to school and here, the peer group influence plays a major
role. As the saying goes "Birds of same kind flock together". Thus the children associate
themselves with like-minded people and form friends. Further, they discuss all sorts of relevant
issues within themselves, create policies to determine which kind of behavior is right or wrong
etc... This behavior is exemplified in majority of the schools, wherein we can find gangs among
the students - formed due to like-minded thinking among the students in a group. Eventually, the
children develop moral behavior and qualities as a result of linking themselves with their peer
group.
In the corporate world, the quality or the nature of an individual can be determined by the nature
of profession that a person occupies in the company. A highly estimable post is attributed to the
person's hardwork; At the same time, if a person happens to be an ordinary clerk, then he is
deemed to be associating himself with mean people who do not have financial power or are
illiterate. The kind of company environment affects the behavior of the employees to a great
extent. For instance, if the rules in the company are lax, then hardly will any employee take
efforts to complete the work in time; a majority of the employees will turn out not to complete
the work within time and ultimately the morale of the employee will go down. In contrast, if an
employee finds himself in the most devoted work environment, the environment will itself instill
a sense of confidence in him and motivate him to reach greater heights irrespective of his
qualification or calibre. Thus, the work environment plays a major role in determining the
behavior of employees.

Moreover, considering the case of politics, we have several factions or parties competing for the
ruling post, because each one has different set of motives. For instance, in the United States, we
have two major parties: The Democratics and The Republicans; Each one has unique set of
ideals and strives towards the development of the nation. By examining which party a person
belongs to or associate himselves with, we can come to a conclusion about the belief held by the
person. This case is applicable to party workers, who choose to devote their lives for the party
because of the ideals found in the concerned party allures them to a great extent. Hence,
identification with political parties also pave way for extracting the nature of a person without
delving deep into his activities.

Some may argue that identification with social groups does not completely identify a person and
can sometimes lead to us in a misleading direction. These people seem to claim that, the persons
associated with a group may not necessarily possess the same characteristics as other members of
the group. But this case occurs very rarely; only when a person relates himself with multiple
contradictory political parties, we cannot come to a conclusion of the real nature of a person.
However, in most of the cases, a person's character is reflected by the social party that he
associates himselves with.

Thus, social groups help shape us right from the moment of our birth till we end up our lives.
During each stage of our life, we associate oursleves with social groups that guides us to tackle
the pitfalls and misadventures that we encounter in our lives. While it is true that social groups
do not paint a complete picture of an individual, for the most part of the case, we will be able to
clearly delineate the character of the individual.

College students should be encouraged to pursue subjects that interest them rather than the
courses that seem most likely to lead to jobs.
Write a response in which you discuss the extent to which you agree or disagree with the
recommendation and explain your reasoning for the position you take. In developing and
supporting your position, describe specific circumstances in which adopting the
recommendation would or would not be advantageous and explain how these examples shape
your position.

College students should be encouraged to pursue subjects that interest them rather than the
courses that seem most likely to lead to jobs.

Write a response in which you discuss the extent to which you agree or disagree with the
recommendation and explain your reasoning for the position you take. In developing and
supporting your position, describe specific circumstances in which adopting the recommendation
would or would not be advantageous and explain how these examples shape your position.

Composition:

Whether college students should be encouraged to pursue subjects attractive to them or to seek
programs that help them prepare for job career is a complex issue. For personal perspective, the
purposes of education are to improve students' morality, and to cultivate their critical thinking
and practical ability; as to college education, students should follow their interests and then grasp
a skill to adapt to the competitive society. To better illustrate my viewpoints lets me present it in
details.

Initially, interest to a student is the best teacher who is capable of tapping his or her full
potentials to pursue the dream inside. He who does what interests him shows little fatigue and
boredom when the task takes up too much time; he also lifts up courage and confidence when
confronted with difficulties in the way. If a graduate engages in an occupation he is interested in
after graduate, he tends to be full of ambition and enthusiasm, with a favorable occupation
morality. On the contrary, it is widely accepted that the education is a failure if the students do
not know where their interests lie. He lives without interests in something what we live without
the Sun. In a word, college students should be guided to follow their interests and pursue their
interested subjects to cultivate their interests, beneficial to the individual as well as the society.

However, it is not uncommon that a student is not likely to become an expert in what interests
him or her. For example, a student who studies theoretical physics shows interest in music;
however, actually, his voice is too common and not attractive. That means it is impossible for
him to become a popular singer in the future. But, music turns to cultivate his emotions and
interests, and makes him creative, and thus helps his physics study indirectly. That is to say, it is
not advisable for students to take their interests as their choice in the future; interest is a very
means to make our life colorful and joyful; it is a way towards happiness.

Furthermore, it is important for students to grasp a skill to survive in such a fiercely competitive
society. He who is out of employment after graduate will be a heavy burden to our society.
Unemployment is a pressing societal problem, which is always associated with other pressing
problems, such as poverty, crime, fraud, drug abuse, smuggle, etc. In this sense, college
education would amount to little value if the majority of graduates failed to hunt a job in the job
market. In a word, students on campus should join in some programs to adapt to the job market.

In sum, college students should strike a balance between their interested courses and the
programs that promise entry into the job market. Interest is the source of enthusiasm and
happiness, contributively to build up confidence and courage. And, we are endowed with certain
practical skills by the programs to survive in the competitive society. In the final analysis, it is
amazing for a graduate to engage in what interests him.
Claim: When planning courses, educators should take into account the interests and suggestions
of their students.
Reason: Students are more motivated to learn when they are interested in what they are studying.
Write a response in which you discuss the extent to which you agree or disagree with the claim
and the reason on which that claim is based.

There is nothing more beautiful than the sight of an inquisitive young mind. Such a mind, is
engrossed in such an academic pursuit that education no longer becomes a burden but a passion.
Course planning which take into account suggestions from students are bound to result in
effective curricula.
Interest is the main motivation to education. If education to a student is merely an exercise of
burden then such education would make no lasting impact on the life of the student. However, an
education that catches the interest of a student becomes not just an education where it is the duty
of the teacher to input knowledge to the student, but the student makes his responsibility to see
that he is well learned. Education becomes a hobby for the student. The only way to achieve this
is to utilizes curricula which instigate such interest in students. However, knowledge of such
interest inspiring techniques can only come from student inputs. Student may suggest a
game/sport approach to learning. Educators may incorporate academic materials with video
games to gain student attention. Course materials may be organized into puzzles which may
attract student attention. Students may also suggest their liking for stories and movies. Educators
could relate course material through story telling (like interesting history behind a concept) or
through documentaries. Also students may also mention procedures which they find difficult to
cope with – perhaps too many assignments. Educators in this way are able to have a feedback on
their teaching style and objectively judge what means to better their curricula.

However, the downside to taking student input is that sometimes student suggestions may be
borne out of laziness. Elementary and high school students in particular are usually not bothered
with the quality of education they receive and are may want academic courses which are not
effective. Such suggestions do not have to be taken by educators but these sugestions help notify
educators of the need to find ways of gaining student attention. Also, since humans vary, the
suggestion of various students may be conflicting and adopting all student contribution may
prove impossible for educators to accomplish.

The greatness of individuals can be decided only by those who live after them, not by their
contemporaries.
Write a response in which you discuss the extent to which you agree or disagree with the
statement and explain your reasoning for the position you take. In developing and supporting
your position, you should consider ways in which the statement might or might not hold true and
explain how these considerations shape your position.

The world of today is full of different fields of human endavours.Each of such fields has its
greats which are essentially those people who made a distinction in their respective fields, in the
process made a name for themselves. However, most of such greats have been put on the
pedastal of greatness by those who have followed or lived after them rather than their
contemporaries. Thus the statement, "The greatness of individuals can be decided only by those
who live after them, not by their contemporaries." holds true in most cases.

Primary reason of the greatness of the distinctive individuals being recognised by their followers
rather than contemporaries, is the fact that it takes a long time for the world to realise the full
importance of the work of such individuals. By the time, the world becomes aware of the full
potential of their work and consequentally regarding them as greats, there lifetime is either
already over or they are in the latter stages of their lives. Hence their greatness is decided by
those who follow, rather than their contemporaries.

The field of science is one such field where many examples of greats being recognised by their
followers rather than contemporaries can be found. Nicolas Copernicus, an early European
astrophysist had come to the conclusion that Sun rather than Earth is the centre of the Universe,
as opposed to the popular belief held at that time. He published his work secretly due to the fear
of regarded as heretic by the Church. However, Copernicus's postulates were proved only after
Galileo invented the telescope few centuries later. Thus Copernicus received his due credit only
after his death by those who followed him.

Thus we see that the primary reason of distinctive individuals not being regarded as greats by
their contemporaries is because the full implications of their contribution does not get realised
within their lifetimes. The distinctive individuals or the potential "greats" of any field are those
people who have a greater foresight and who go on to explore some yet unexamined areas of
their fields. For rest of the world, including contemporaries, it takes a longer time to realise the
full significance of their contribution. Once the world realises this significance, the contributer is
made a great. However this generally takes one or more generations.

However,there are fields which are exeption to this phenomenon. For example, in fields of sports
or performing arts, majority of individuals' talents and contributions are get noticed immediately.
The world realises their potential based on their on-stage or on-field performances. Greats in
such fields get decided by their contemporaries. Even in fields such as science, if a contribution
has immediate impact, the contributer is regarded as great by contemporaries themselves.
Summing up, the statementThe greatness of individuals can be decided only by those who live
after them, not by their contemporaries." holds true generally but there are exceptions when the
contributions of the greats get noticed early.
Students should always question what they are taught instead of accepting it passively.
Write a response in which you discuss the extent to which you agree or disagree with the
statement and explain your reasoning for the position you take. In developing and supporting
your position, you should consider ways in which the statement might or might not hold true and
explain how these considerations shape your position.

When we were kids there are many questions come to our mind but our innocent questions are
never answered and left doubts forever. When we grow old, we know where the problem lies and
therefore we encourage students to ask questions and get their doubts cleared. Gutham Buddha,
he himself mentioned during teaching to pupil that do not blindly follow and accept what I said,
you have to experiment by yourself and keep asking questions to clear doubts. Therefore,
Buddhism can able to go parallel with science.

Question comes to mind not only to students but to everyone in day to day life and one has to
come up with the solutions. Therefore, one should not hesitate in asking questions what the
teacher taught because there are many questions to be solved in life after finishing school. if
students hesitate to ask questions then their whole life will be accepting what other have to say
and when they grow up they will be followers instead of being leaders.

Moreover, students who ask questions will become more open minded and doubt free compared
to the students who does not. But one should be careful when asking questions. While asking
questions to someone students should be appropriately ask question and not being judgmental.
Ask questions to teacher in an appropriate way. Student should not doubt or hurt the feeling of
the teachers.

In addition, teacher should encourage students to come up with questions and solve every
questions being asked. Class room of 21st century should be discussions and interaction between
teacher and students which results in clarifying doubts.

While it may now be clear that the students should always ask question to teacher, it should also
be apparent that the asking question not only solves doubts but also relaxes students mind. Also,
teacher should encourage the students to ask question that will improve in teaching as well. In
next class teacher will become aware and come prepared for the class and hence students will
gain more information.
The increasingly rapid pace of life today causes more problems than it solves.
Write a response in which you discuss the extent to which you agree or disagree with the
statement and explain your reasoning for the position you take. In developing and supporting
your position, you should consider ways in which the statement might or might not hold true and
explain how these considerations shape your position.

Science and technology has revolutionized the face of mankind. Technological innovations,
inventions and discoveries have evolved from their natal state in the Iron Age to the 21st century
today. This has paved the way for a very luxurious lifestyle for man and has led to an increased
and more comfortable pace of life. Though it may have seemingly solved many arduous tasks in
our day-to-day life, in reality, it causes many problems today.

Today, man has no time for anyone except for himself. He is so busy and pre-occupied with his
work and life that he has started ignoring his family too. This rapid pace of life has made him a
workaholic and is responsible for having affected many a family relations. Statistics reveal that
this is the primary reason for the increase in the divorce rates. Everyone is leading a mechanical
life being nonchalant about one’s own household.

In this age of cut-throat competition, Charles Darwin’s ‘the survival of the fittest’ theory holds
true. Today, man wishes to accomplish his desires by hook or crook. Everyone wants to excel
and be the best that no one is bothered about the repercussions his actions can have on him and
the society. This has led to deterioration in the moral values and ethics especially in workplaces.

Happiness and contentment with one’s work have taken a back-seat in the peregrination of life.
Man has fallen into an abysmal chasm of avarice striving for more money, success and fame. He
sees nothing beyond that. One should be passionate but not to the extent that he is even driven to
commit suicide if he doesn’t achieve the desired goals. The life in the older times when there
weren’t as many opportunities or a slower pace of life had helped people to live happily. But
today man is doomed and trapped in his own pit to achieve success that he has forgotten how to
smile and enjoy the nuances of life.

It has become difficult and onerous for man to manage the rapid pace of life. He is so much
burdened with responsibilities and worries that even perfunctory tasks have become difficult for
him to perform. He expects and desires only for success that he will not be able to countenance
failures in his life. Hence he may face psychosomatic disorders which further hurl him downhill.
The luxurious lifestyle and the pace of life have caused many environmental problems too. High-
end vehicles have replaced the horse buggies and the bullock carts. This traffic and vehicle
congestion on the roads has increased the rate of pollution which leads to more number of
problems. People are suffering from respiratory ailments, diseases etc affecting their health. The
climate change and global warming will have long-term effects for the future generations too.

The victuals consumed by people are also having a bad effect on them. New preservatives,
flavoring agents have adulterated today’s food. Tin Packaged items and junk food have made the
people obese causing diabetes and other diseases. People are so busy with their lives that they
have resorted to fast food and junk fried items thereby staying away from healthy food. This has
caused long-term problems.

Everything is automated in today’s world. People have become very much dependent on
technology and machines for their survival. They get everything at their beck and call that they
have become sluggards since the manual labor is at a minimum nowadays. The day is not far
away when automated robots will replace men.

The life of mankind is changing at a rapid pace. This increasingly rapid pace of life today causes
more problems than it solves. Hence one should take care that the problems caused by this rapid
pace are solved first for life is short!

Claim: It is no longer possible for a society to regard any living man or woman as a hero.
Reason: The reputation of anyone who is subjected to media scrutiny will eventually be
diminished.
Write a response in which you discuss the extent to which you agree or disagree with the claim
and the reason on which that claim is based.

Role models functions as a valuable and indispensible component of the balanced development
of a society, especially in this digit era of rapid advancement in technology and communication
methods leading to more and more information previously unknown exposed to the general
public. Indeed, along with the boom of media, particularly the internet media, celebrities who
subjected to media scrutiny are likely going to suffer more intense scrutiny than before through
the disclosure of their personal information. Nevertheless, to purport that the reputation of
anyone who suffered the media scrutiny will definitely be diminished is to present an overly
simplistic view.

It is true that with fast development of mass media, the extent and quality of reports about
celebrity news appears especially inordinate today. Consequently, scandals and other negative
news about public figures previously unknown get more chance to come to the masses. Due to
people’s attention are focused more easily on these celebrity news rather than those social issues
in economy or politic, some arguers claim that under this intense scrutiny of media nobody’s
reputation could escape from destroying.

To further illustrate their position, the proponents may take some cases for example, such as the
Wikileaks. Wikileaks is a website through which a tremendous amount of scandals have been
exposed to the public all over the world. Information disclosed by Wikileaks reveals the masses a
genuine and dark operation of the governments by exposing details about officials’ conversations
and documents. Obviously, in this case the internet media played an absolutely passive role in
the maintaining of government image, because large private conversation details were showed
which contained many unpleasant personal assessments and view points.

At first glance, the statement above appears to be somewhat convincing, but further reflection
reveals that it omits some substantial concerns that should be addressed, thus not cogent enough.
In this argument, only reports and news about public figures’ passive image are taken into
consideration, while positive information ought to be considered equally. Let us consider the
wealthy person Bill Gates. Not only are his tremendous amounts of money and successful
company available in the newspaper, but his charitable institution is reported and disseminated
through the mass media.

Finally, whether the reputation and image of a person will be damaged by media scrutiny or not
should be considered on circumstances, not principles alone. The careful scrutiny will only
provides us with more available messages and details about a person, after all the judgment and
evaluation are made by ourselves. Regardless of the positive reports about figures, to conclude
that media scrutiny will only be destructive on anyone is unjustifiable.

There is a famous Chinese idiom underscoring the importance of a full analysis, “three men
touching an elephant”, which tells that each one of them touched a body part of an elephant but
failed to regard the elephant as an ensemble. By the same token, the balance between accepting
both positive reports and passive scandals will ultimately serve to draw the correct conclusion.

Competition for high grades seriously limits the quality of learning at all levels of education.
Write a response in which you discuss the extent to which you agree or disagree with the
statement and explain your reasoning for the position you take. In developing and supporting
your position, you should consider ways in which the statement might or might not hold true and
explain how these considerations shape your position.

In this age of cut-throat competition, getting good grades is of prime importance for all students.
Many believe that success, particularly financial is directly determined by one’s grades. This has
naturally led to a rat-race amongst students at all levels of education. However, critics beg to
differ. They argue that intense competition for high grades amongst student has seriously
affected the quality of learning. Although true to a small extent, I believe such competition has
indirectly raised the quality of education.

Competition for high grades brings the best out of students. Many students have great hidden
talent that goes unnoticed until they compete. For example, an MBA Finance student may have
excellent skills for marketing. A competitive environment may help him/her to use his/her
untapped talent to succeed.

Furthermore, healthy competition encourages one to work harder and achieve better results. In
short, it acts as a stimulus. For example, competition may stimulate a B-grade student to work
harder and get better results next time.

Students aiming for high grades are highly focused, smart and diligent. A direct consequence of
this is that such students ensure all round development. For example, consider a top-tier
university student getting high grades. This student would indirectly raise the quality of learning
through his/her efforts – research work, good internships, extra-curricular activities, social work
etc.

On the other hand, certain people criticize this intense competition for high grades. They argue
that such competition unnecessarily puts a stress on students, thereby affecting their ability of
radical thinking. This, they believe, hampers the quality of education.
To conclude, intense competition for high grades has its merits and demerits. However, its merits
like bringing the best out of students, exploring hidden talent and encouraging students to work
harder far outweigh its demerits. Therefore, such intense competition indirectly improves the
quality of learning rather than limiting it.

Universities should require every student to take a variety of courses outside the student's field of
study.
Write a response in which you discuss the extent to which you agree or disagree with the
recommendation and explain your reasoning for the position you take. In developing and
supporting your position, describe specific circumstances in which adopting the
recommendation would or would not be advantageous and explain how these examples shape
your position.

Large universities nowadays are moving away from their rigid structure and have become highly
flexible in education. A diverse set of programmes with a variety of courses helps universities
reputation as one where a student can develop a plethora of skills. But, this change has stirred
debates in various universities about whether to make courses outside the student's field of study
compulsory or not. I believe, universities should stay somewhere between rigid and fully flexible
and adopt an approach where some necessary courses are made compulsory, while other
peripheral courses made optional.

For example, courses relating to issues such as ethics, communication skills, environment should
be made compulsory in the 1st year of curriculum. These are subjects which are certain to help a
student in future and helps them become not only an academically good student, but also a better
person socially.

Other courses such as philosophy and psychology should be kept optional. These subjects are
highly specialized and should be offered only to those interested in it. In fact, universities can
develop and incentive scheme by offering minor or double majors to students interested in field
apart from their's if they complete a certain number of subjects other than their prescribed set.
Audit courses, where exams are not conducted and students study just for the sake of learning
should be encouraged.

Imposing courses on students are never welcomed, as coercing may cause students to develop a
sense of disdain towards the whole field of study itself. Rather, there should be some scope of
experimentation. For example, Harvard has introduced provision for courses where students get
to decide at most 1-2 subjects in a semester by public opinion. This has been highly successful
and students have opted from field as varied as religion and outer space.

But, in implementing flexibility, certain care needs to be takes. Undergraduate programs are a
better platform for such experimentation, as students at that stage are in a process of developing
their interest, while postgraduate programs are highly specialized, and thus care needs to be
taken while taking such step.

So, in conclusion, universities should increase flexibility, but only certain courses should be
made compulsory to all while others should remain optional. Students' opinion needs to be taken
into account and universities should also take care of the level (UG/PG) at which they are
implementing such subjects

Educators should find out what students want included in the curriculum and then offer it to
them.
Write a response in which you discuss the extent to which you agree or disagree with the
recommendation and explain your reasoning for the position you take. In developing and
supporting your position, describe specific circumstances in which adopting the
recommendation would or would not be advantageous and explain how these examples shape
your position.

From the very start of the human civilisation quest for knowledge has been put up in the top
priority. So goes the saying "knowledge is wealth". Man in the quest of knowledge has
discovered every aspect of life, and so did he lived through it. Many a discoveries and inventions
have been made in this un-finishing journey.

People in the very start learnt every thing through experience, though it was affective it was time
consuming. So schools were established and people profound in certain aspects were made
pedagogues. As a single person can not teach every thing, the aspects were virtually split into
portions and assigned to persons profound in respective portions. As the education system
evolved, lot of changes have been brought. People interested in a pirticular field were allowed to
have a deep study in that respective field. Hence started the concept of 'curriculum', this time
pertaining to respective interests.
But now a days knowledge is being looked about in a narrow view. In past years, it took many
ameanings from equivalent to marks to weight of a pay cheque. Educational instutes have started
making fixed curriculum for a certain field. For years through the same curriculum is taught and
studied. Institutions have been focusing on ways to get their students a better job with a better
pay, rather than making a person useful to society. This has already induced narrow mindedness
in the youth, so called back bone of any country; developed or developing.

As the time changes, every thing keeps changing, so should be the curriculum. Very less student
realize this fact, but the educators should consider this. Including student intrest topics in the
curriculum would increase the students fondness towards gaining knowledge rather than just
learnig to pass exams. Subjects in which less people show interest can be made as electives while
subjects attracting large number of students can be made major courses. This would build up
more enthusiasm in students in gaining practical knowledge.

So educators must include what a student wants in the curriculum. This would revolutionise the
education system, yielding not only good students but also a responsible citizen.

Educators should teach facts only after their students have studied the ideas, trends, and concepts
that help explain those facts.
Write a response in which you discuss the extent to which you agree or disagree with the
recommendation and explain your reasoning for the position you take. In developing and
supporting your position, describe specific circumstances in which adopting the
recommendation would or would not be advantageous and explain how these examples shape
your position.

The author of the issue highlights that educators should take facts into consideration when it
comes to teaching barely after examined the ideas, concepts, methods and so forth, which could
patronize those facts. Due to the controversial nature of this question, there is not a general
agreement on this issue. I am, however, to a great extent of the conviction that individuals could
act wisely and make a sound decision if they encounter with reality and facts. In the following
paragraphs, some convincing reasons will be delineated to elaborate my personal stance.

To commence with, it is established beyond doubt that educators play a dominant role in every
given society and people who learn are pillars of that society. Indeed, what they will learn is of
utmost critical importance because it can enhance their personal life and career in near future.
Broadly speaking, when students regardless of their level of study do not realize deeply the
concepts, ideas, and crowds of other things related to that subject, they could not afford to
understand the real meaning behind the subjects not only in that field but also in their lives. For
instance, one of the major goals of learning facts is that people should be able to use them in real
life and if they do not make an effort to crack the meaning behind facts, they will run into a huge
problem and do something wrong in the personal lives. To elucidate more on this issue,
educators should encourage people to learn fundamental concepts, ideas and so on, which not
only can make people decide their path but also can make them analyze facts meticulously and
precisely and become knowledgeable in near future.

Another reason that should not go unnoticed is that there is a close correlation between learning
facts and becoming successful. As a matter fact, vast majorities of people bend over backward to
make a progress and become successful in their lives. Generally phrased, if educators coerce
people to learn basic ideas, indeed, they indirectly help them become successful. In fact, hardly
ever people can touch progress without taking facts into account. As a vivid example, if a
businessman wants to have a prosperous career or even run a successful business, he/she should
be taught basic needs by the experienced educators and confront with facts in order to recognize
and seize great opportunities. Delve further into the issue, if students do not have any
information about facts and its fundamental and necessary rules, it could have deleterious effects
on their lives and create a gap between them and progress day in day out.

However, what was alluded to above can't be overgeneralized to all contexts. Alongside with the
innumerable benefits, it has, indisputably, myriads of facts happen by experience. It means that a
plethora of people face facts just by their previous experiences and do not need to know more
about a fundamental need. As far as I am concerned, taking various ideas, concepts and so forth
could not only be waste of time but also could be annoying for some people and make them
confused. In such case, myriads of people will lose their enthusiasm to learn facts.

In light of reasons elaborated, I reiterate that students or people need to appreciate concepts in
order to perceive facts in their real lives. Indeed, this methodology will make people do not
something wrong and not to make an idiot decision. In addition, it will provide fertile ground for
people to reach success and make progress day in day out.
Claim: We can usually learn much more from people whose views we share than from those
whose views contradict our own.
Reason: Disagreement can cause stress and inhibit learning.
Write a response in which you discuss the extent to which you agree or disagree with the claim
and the reason on which that claim is based.

It is true that debating may cause mental stress and even conflict while disagree each other. If
everyone agrees and conforms on one thing, what is going to happen in the world? Of course, it
would be easy to rule the society and the world. But Is it right way to go? Without conflicting
and debating, I believe there would no way to improve the society and the world.

Because we challenge ideas, we are able to discover and learn important things. Historically,
there has been many hypothesis and theories discarded that we thought so true with no doubt.
For instance, people thought the Earth has a flat surface in the past. In 5 BC, Pythagoras came
out with different and ridiculous claim that the Earth is round like a ball. Eventually, he was right
about the shape of the Earth. If we only disagreed what he had to say and ignored it, we would
never know significantly important fact about the Earth. Furthermore, what if Columbus did not
believe that the Earth is round? He was never able to explore the America continent. In this
regards, contradicting views actually give you more opportunity to explore and learn rather than
inhibit learning.

While we only learn and support the views that we share, the other side of views also rises by
finding fallacies of our views. Adam Smith believed in invisible hands that solve every problem
in the market. Many people and government followed and worship about Capitalism “All might
is the money. However, there were people who looked at the relationship between labor and
bourgeois. They asked themselves, “Why are there poor workers, and rich bourgeois?” If there
was no listening and learning from Communism, there would not be the world like today which
governments provide welfare to the poor, and implement the private property systems.

Of course, the life must be easier everyone agree on everything. Since each individual have
different personality and lifestyles, basically, it is impossible that we only learn from people we
share the same views. Friends, teachers, and even, parents sometimes have different views on
certain things in everyday life. One of the aspects we differ from other animals is that we think
reasonably and accept the differences. For the improvement human being, I believe that
disagreement comes at first as an stress and pressure, but it directs us improvement and advances
at the end.

Some people simply do not like to debate or conflict because they do not want worsen the
relationship with which they care and it is very comfortable to be with. However, it is impossible
to only deal with people who share the same views, and it delays improvement of my ideas and
cannot go beyond them. Thus, I strongly believe that every opinions or views should be
considered even though it conflicts with our own despite of stress because it can compensate you
more.

Government officials should rely on their own judgment rather than unquestioningly carry out
the will of the people they serve.
Write a response in which you discuss the extent to which you agree or disagree with the
recommendation and explain your reasoning for the position you take. In developing and
supporting your position, describe specific circumstances in which adopting the
recommendation would or would not be advantageous and explain how these examples shape
your position.
Author of the Constitution, James Madison once stated, "If men were angels, no government
would be necessary." I think that this quote aptly sums up why government officials should
balance their own judgment and expertise with the will of the people they serve. This is the best
way for the system to work because there are problems with the other forms of government, no
one has perfect judgment, and it is the best way to ensure that quality policies are being carried
out.
The founders of the United States set out to create a system like no other because they saw the
problems that arose for other kinds of governments. They rebelled against a cruel and foreign
monarchy which either didn't know the will of the people under its rule or didn't care. They knew
from firsthand experience that the judgment of people in power was not always the best.
However, at the same time, they saw the collapse of a pure form of democracy in ancient Greece
where the people's whims were always carried out. Democracy was a dirty word, and equated to
"mob rule." The majority always got its way even at the expense of the minority's rights.
Therefore, they decided it best for the people to have some say in the policies and laws that
affected them, but to ultimately put the power in the hands of representatives who were qualified
and could use their expertise to make the tough decisions about what is best for the country.
Since its inception, the representative form of government has been successful in the United
States and when it is duplicated abroad.
It should be apparent that it would be dangerous to give lawmakers or the people complete power
over what laws are made in the country. No one has perfect judgment all of the time, whether it
be an expert policymaker or the general citizenry. There are a great many examples of this in the
United States. For instance, the issue of same-sex marriage was put to a popular vote in many
states. The more liberal and open minded state populations voted to allow it, while other more
conservative and traditional state populations created added amendments to their state
constitutions to ban same-sex marriage. This created problems when married couples moved to a
new state with a ban and could not get divorced because their new state of residence did not
recognize their union. Ultimately, it the judges appointed to the Supreme Court that decided this
complex issue. These experts on Constitutional law took the rights of all citizens into account,
and using their years of accumulated wisdom and experience, while also taking into account the
changing nature of public opinion regarding same-sex unions, decided that it best served the
nation’s interest to guarantee and protect the rights of same-sex couples.
However, it is also true that the representative form of government is far from perfect. Far too
often, policymakers tend to lose touch with what their citizenry wants, especially those in
Congress and the Senate, who spend much of their time in Washington, DC, far from the
everyday lives and concerns of their constituents. This day and age, politicians at the national
level may be influenced more by lobbyists than by their constituents. For instance, the recent
debate about gun control. The vast majority of American citizens would like for their to be some
new laws regarding purchasing guns, specifically related to background checks for gun buyers.
Yet, even though bills related to this have been introduced in the Congress, they have either
failed to pass them or failed to even schedule a vote on the particular bill. In theory, lawmakers
should be held accountable (by citizens voting them out of office if they are unhappy) by the
citizens they serve. However, in practice, this is not always the case. If government officials no
longer even had the threat of being held accountable for their voting records by citizens, they
could just pursue the policies that benefit themselves and the special interests that donate money
to them for every issue, without regard to what would happen to the people they serve. In my
opinion, this is a very flawed way to govern.
Thus, for a representative government to be effective and result in the best policies, it is
imperative that a combination of the judgment of government officials and the will of the people
is in place. It is impossible for the general public to be knowledgeable about every governmental
issue. This could be for a variety of reasons: they don't have the education or the time to study
the issues, or certain areas of policy should not be made known to a large number of people (i.e.
information that might impact national security). On more technical issues ( which also tend to
coincide with things people do not care about), government officials should use their judgment
and expertise to make policy. But when it comes to issues that their citizenry care greatly about,
they should also take into consideration the will of the people they serve. The United States
government was set up in such a way to ensure that the best policies are made. This involved the
two chamber system, where the House could be more responsive to the citizens as they were
directly elected by citizens every couple of years. The Senate is more deliberative (though now
they are also elected by the people). But, the idea is to have a diverstiy of people and procedural
steps involved to ensure that everyone's voice is heard and that policies that are being created can
actually solve the problems they are meant to address. With some exceptions, I would say that
this process has worked out astonishingly well. So, in summation, it has been shown throughout
time that no men (or women) are angels, which is why it is vital that both the personal judgment
and expertise of government officials and the will of the people are necessary are consulted when
making policy.

Young people should be encouraged to pursue long-term, realistic goals rather than seek
immediate fame and recognition.
Write a response in which you discuss the extent to which you agree or disagree with the
recommendation and explain your reasoning for the position you take. In developing and
supporting your position, describe specific circumstances in which adopting the
recommendation would or would not be advantageous and explain how these examples shape
your position.

It is said that young people are the future of a nation, and they will define the shape of a country
in the coming generation. Taking this into account, I believe, youth should be encouraged to
pursue long term and realistic goals, but at the same time they should not forget about the present
they are living in.

Young people, especially teenagers are a capricious and whimsical lot. They need proper
guidance and counselling to grow in life. Instilling in them, a thought to focus on long term goals
will help them in maturing, and contributing to the society they live in. Youngsters who know
their goals will take positive steps to achieve it, rather than wandering about in search of
recognition. This will make them more disciplined, punctual and mature. Such youngsters will
achieve their targeted goals (say becoming scientists, engineers, researchers) and these human
resources in future will be of great help in the overall growth of the society/country. On the other
hand, focusing only on fame and recognition may make them wayward, without a proper
direction. In the attempt to seek immediate fame, young people may even take wrong and
immoral steps, thereby harming the society and in the future even themselves.

However, it should noted that, blinders (used to make horses see in a single direction), should not
be imposed on youth, such that they focus only on long term goals and forget the means by
which they are planning to achieve it. For example, a teenager whose goal is to become rich in
future may do so, by a series of robberies, thereby achieving his goal. But this kind of mentality
should not be fostered. Rather, youth should be taught proper means to achieve their goals.
Another problem which may arise due to singular focus on long term goals is lack of interest in
present and living in future. This may harm the young people psychologically.

So, in view of these points, I believe, young people should be encouraged to pursue long term
goals, but achieving such goals should be made a continuous process, such that they continue to
do well in the present, live in the present, while at the same time, get closer to the goal step by
step, achieving a part of it every day, which accumulates slowly to get them to their ultimate
goal.
The best way to teach is to praise positive actions and ignore negative ones.
Write a response in which you discuss the extent to which you agree or disagree with the
recommendation and explain your reasoning for the position you take. In developing and
supporting your position, describe specific circumstances in which adopting the
recommendation would or would not be advantageous and explain how these examples shape
your position.

The speaker asserts that best methodology to teach students is to laud them and never critise. The
purpose of teaching is to develop all round personality ,to develop ability to decide what is
correct and what not which cannot happen without knowing where they are going erroneous .
Therefore,I do not agree with the author that bringing out negative points should be ignored.

No doubt , praise is very crucial for developing self confidence and keep students motivated.It
helps them to be in positive frame of mind.That does not mean we neglect when students go
wrong because it will have more adverse effect than doing any good for them.For instance, let us
consider a student is very good in presenting ideas but is very poor at grammar.Now,if teacher
does not point out his weakness he has to suffer.If we take the example of a sports team where a
student is a exceptional player but lacks sporting spirit and team co-ordination.If the faculty don't
point it at correct time may be he will never get past it and can never become a good player.

Secondly, it will make students over-confident and complacent.Because they think they are
perfect whereas the fact is otherwise.It will create a false aura around them.

The speaker might retort that ignoring negative aspects will reduce stress and students will be
more optimistic towards life.In actual it is only a fleeting dream because when they will go
further in life outside school and in jobs they have to bear consequences of their actual
actions.Moreover,they will not know how to handle criticism and this can even lead to severe
depression problems.

All the above points,helps get an insight that to become a successful individual a person needs to
rectify his or her follies. The best way that could happen is that during their learning years
faculty bring out both the positive and negative aspects and helps to correct them.A healthy dose
of criticism is essential to turn out into a balanced person who can positively contribute to
society.
If a goal is worthy, then any means taken to attain it are justifiable.
Write a response in which you discuss the extent to which you agree or disagree with the
statement and explain your reasoning for the position you take. In developing and supporting
your position, you should consider ways in which the statement might or might not hold true and
explain how these considerations shape your position.

A goal will always be worthy from prospective of person who is preserving that goal and urge to
accomplish it. But veterans and elite people has made laws and rules which are not supposed to
be broken otherwise people can be punished for doing counterfeiting things. Goal will be worthy
and praised if it did not break any laws and order as well as respect rules that societies have
made. I believe that achieving goal by what way is more important than goal itself. so I take side
against the statement and I will justify my statement by taking two examples into consideration
which will explicitly declaim my reasoning.

Thing which is right for someone can not be right for whole crowd. For instance, take example
of examination in school. What is motto of examination? To test knowledge that student has
grasped throughout the year. If student wants to set goal of higher percentile then obviously he
has to do persistent, sedulous and relentlessly hard work for examination. What is motto for that
student? To get higher percentile in examination. That is worthy goal for him. He can
accomplish this goal by cheating in exam or by copying some other student paper. Will our code
of ethics allow it. Absolutely no. Because we categorized this as unjustifiable. No matter under
which circumstances he surmounted society ethics. They will not permit it. But for student goal
of higher percentile is accomplished so here student and society thoughts backlash. Which imply
that only justifiable means are permitted towards worthy goal.

Take example of politicians who are elected by votes of public. Motto or goal for politicians is to
get high number of votes. If they start using illicit techniques to accomplish it for example by
purchasing votes of public or by giving some thing money to community if all of people of that
community will give vote to him. By using theses illicit methods he may won elections and call
himself as diplomat but does our laws allow it? Never, but he won election, his goal which was
worthy for him is may be accomplished by illicit ways but it is illegal for laws and whatever is
illegal for laws can not survive in society.

So goal which is worthy will be praised by people if it is accomplished without surmounting


moral standards. Everyone will even respect it because ways through which it is accomplished is
compatible with us and laws. Example of student cheating in exam and politicians are buying
votes, so these people are accomplishing their worthy goals but they can never justify their
means through which it is accomplished. Which explicitly declaim my reasoning towards
counter statement.
In order to become well-rounded individuals, all college students should be required to take
courses in which they read poetry, novels, mythology, and other types of imaginative literature.
Write a response in which you discuss the extent to which you agree or disagree with the
recommendation and explain your reasoning for the position you take. In developing and
supporting your position, describe specific circumstances in which adopting the
recommendation would or would not be advantageous and explain how these examples shape
your position.

It is true that in order to become a complete individual, one needs to have diverse knowledge
which would enable him to tackle problems in different aspects of life. But in order to acquire
that, enforcement of rules on the students by colleges would be a grave mistake and would not
satiate any useful purposes. In this situation, college should allow a student to choose what they
want to study outside their curriculum and provide flexibility which encourages them to do so.

A standard curriculum in primary and high-school education is justifiable as children are in the
process of developing their own interests and diverse subjects like arts and sciences would help
them to grasp primary knowledge. Also these fundamental knowledge like imaginative literature
would help them encompass a larger imagination, social sciences would help them learn basic
ethics one should not forget in his lifetime, history would let him know the past of his own
country, sciences would teach him to explain his surroundings logically. But college courses are
entirely a different ball game where courses dive deep into the subject and are little concerned
with basics. To become a well-rounded individual, a child has already acquired enough in his
school and can apply those in future if needed.

Now, when a student steps into a college, he pursue his subject of interest only. Times are now
limited to him within which he has to acquire as much knowledge as he can as this will help him
to succeed in professional life more than his social life. After completion of his own courses,
there may be little time left which would allow him to pursue other literature topics like poetry,
novels, mythology which are completely irrelevant to his career. Forcing him to take up this
subject would prove to be onerous to him which could be reflected in his final scores and that
would seriously hamper his academic and professional life.

Students are precious resources to a nation. They are the one to carry a nation forward and any
discrepancy in curriculum can prove fatal to their future. Different students have different
interests according to which they have taken up courses for higher studies in college. If a student
who has a profound knowledge in Medical is forced to take up literature , he will end up
studying more for literature whereas he should be spending time for Medical only. This will lead
to a psychological pressure on him and he may decide not to continue any further study. This
situation would be disastrous to both the student as well as the nation as the student could have
studied Biology in depth and could serve the nations’ poor as a doctor.

In conclusion, the effect of forcing a college student to take up subjects irrelevant to their
professional life could prove fatal and would hamper the future of both society and nation.
Rather than enforcing students, if colleges provide the flexibilty to allow students to take up any
subject they wish to pursue, then that could prove beneficial to them . For example, a history
honors student can take up a foreign language class as they have no practical class to attend and
can utilize the free time effectively. However the same cannot be said for every one and that
decision should be rested on students only as they are the one responsible to drive their career
forward.
In order for any work of art — for example, a film, a novel, a poem, or a song — to have merit, it
must be understandable to most people.
Write a response in which you discuss the extent to which you agree or disagree with the
statement and explain your reasoning for the position you take. In developing and supporting
your position, you should consider ways in which the statement might or might not hold true and
explain how these considerations shape your position.

Although it is good to be understanded by the majority, the true value of any work of art lies in
the thought it conveys. Sometimes the authors or composers make it easy to be found out, while
sometimes the ideas are just to complex or obstruse to simply express, leaving the work needing
some kinds of intelligence and having to be explained. After all, the real merit depends on the
idea it conveys.

First, in order to have merit, the work of art needs to convey some important thought such as the
distinct aspects of controvertial issues or making people know the need of some poor. The real
greatness of any work lies in all the important thought underlying the work, that is, the meaning
of the work. If a work just full of good skills and some hard efforts but simply be done for the
show off of the artist and leave nothing more, the work will be meaningless, and, as a result, the
merit will be pretty low, even though the praise it may get from some critics.
Second, there are kinds of work that are meanful but easy to be sensed. Some factors are the
ability to convey complex idea, some are the complexity itself made the main theme of the work
difficult to be expressed. No matter what is the principal cause, the merit of the work doesn't
depend on the complexity, but the meanings. It needs not to be hard to sensed, but needs some
kinds of meaning.

Third, there are still some brilliant works that are meanful but hard to be understanded. Even
though the majority may hard to know what the author really want to convey, the true value of
the work lies in the meaing in it, and the aboundance in it guarantees that it deserved the merit.
They may need some one to explain what the work has, but once some guy gives them
instuction, they will find that it deserved the merit.

In the end, it is the meaning that give the work merit, not the extent of understability. To have
merit, one needs not to try to convey their complex ideas into simple ones, but simply express it.

Many important discoveries or creations are accidental: it is usually while seeking the answer to
one question that we come across the answer to another.
Write a response in which you discuss the extent to which you agree or disagree with the
statement and explain your reasoning for the position you take. In developing and supporting
your position, you should consider ways in which the statement might or might not hold true and
explain how these considerations shape your position.

The given statement implies that most of the discoveries made by human yet are accidental and
some of them were found as a bi-product of a research which carried out for finding another
questions answer. This statement is not completely true about all discoveries or creations made
by human, so i would rather disagree with it.Though it is true that many important discoveries
made by human were kind of accidental, but the person discovering that thing must have made
many efforts to discover it.

One of the most important discoveries in the history is discovery of gravity which was made by
Sir Isaac Newton. This discovery was made after an apple fell upon head of Newton, but that
doesn't mean it was an accidental discovery. Because after fall of that apple Newton who was
already doing some research in the field of physics though of a force which makes the apple fall
downwards on the ground and that's why the gravity was found. Suppose if the same incident
was happened with an ordinary man, then he may or may not found gravity. That’s why this kind
of discovery isn't completely accidental.
Similarly, though some important discoveries are accidental, it isn't necessary that the discovery
is a bi-product of some different research. For example, the discovery of fire in the stone age was
accidental but the human at that time was seeking something that creates heat and burns the
wood chopped by him. The human then after made many discoveries like wheel which was
created by human for the reason he wanted and not for some different reason he didn't.

In the same fashion, some experiments do find answers questions that it aren't supposed to find,
but those answers may or may not be a discovery. It may be a useless thought. For example, if a
research is going on producing alternative eco-friendly fuel for petrol and in that research if a
new fuel is found which can generate more energy than petrol but harmful to environment, then
that does answer some other question but isn't of much concern for that current experiment. So
such alternative discoveries aren't each time useful.

Conclusively, it is true that most of important discoveries made by human are kind of accidental
and some of them also found as a bi-product of experiment. But that doesn't mean the people
who discovered that didn't made any contribution in its explained by the example of discovery of
gravity the accident was made discovery by that discoverer.

The main benefit of the study of history is to dispel the illusion that people living now are
significantly different from people who lived in earlier times.
Write a response in which you discuss the extent to which you agree or disagree with the
statement and explain your reasoning for the position you take. In developing and supporting
your position, you should consider ways in which the statement might or might not hold true and
explain how these considerations shape your position.

The given statement implies that study of history is mainly done to disappear or vanish the
misapprehension that people living in current world are a lot different that those existed in
history. This concise statement is true as the peoples' natures really don’t change completely and
some part of their ancestor’s nature or temperament remains in the new generations also. Some
people might disagree with it by thinking that new generations bring new mindset which is partly
true, so i would prefer to agree with it.

Firstly, the nature of people never completely differs from their ancestors as it is proven by many
examples. A very compelling example would be of terrorists, many terrorists groups exist in this
world and yet none of them have completely vanished. The major reason for extant of these
groups is that, they convey their thoughts from old to new generations and most of the times as
the new generation also finds those thoughts reasonable, they accept it and become part of
terrorism. This example completely establishes the truth that some part of mindset of ancestors
from our history is conveys further and people never change completely.

Similarly, study of history not only benefits us by vanishing the misconception stated but also it
makes us aware about our societal development. For example, the laws existed in many centuries
ago about racism are now completely changed and racism does not exist nowadays. In history of
many societies like American society the racism was severely present and black people were
harassed a lot in those days but today the society have changed and all kinds of people live
together in peace. This example proves that there was some better developments happened in
society.

Conversely, even there were a lot of good developments done in society and study of history
does prevent some people to get evil, there are still some people which will never change. For
instance, an authoritarian like Saddam Hussein who ruled Iraq for many years and nagged people
of his country for his own reasons. These kinds of people always exist and cannot be prevented
permanently for entering the world, because "for every good there exist some evil also".

On conclusion, it is wise to say that study of history helps many people to get out of illusion that
people have changed and a lot different than those people existed in history, but it isn't the only
major benefit of it.History also gives us feedback of our developments in various fields and
admonishes us about the bad possibilities in development.

Learning is primarily a matter of personal discipline; students cannot be motivated by school or


college alone.
Write a response in which you discuss the extent to which you agree or disagree with the
statement and explain your reasoning for the position you take. In developing and supporting
your position, you should consider ways in which the statement might or might not hold true and
explain how these considerations shape your position.

The statement given implies that in any kind of learning whether it is sports, education or
something else, the person who wants to learn must have some strong will, persistence and other
personal discipline abilities and learner cannot be motivated by motivations by his or her school
or colleges alone. As the statement is a factual truth itself and there are plenty of real life
examples that can be seen in every field of learning, so i would prefer to agree with it.
Firstly, as a matter of fact in any kind of field things like hard work, persistence is mandatory to
enthrall the success. For instance, in field of education every student is not the same and not
everyone gets pure erudite. There are some students have great hunger of knowledge and they
just give their hundred percent to learning and show persistence in their learning, but other
reluctantly learn who definitely don't get success. So such a simple example shows that self-
discipline is must in learning fields.

Similarly, some students do have the ability to persist learning, but they haven’t given proper
motivation and they fail to achieve what they want. For example, some students are neglected in
schools by teachers for their outside physical impression like a student who is interested much in
sports and not in studies, but such student might have great potential in studies also. If these
students are motivated properly by their parents or some other people then they can perform
great in both sports and studies also. Therefore, motivation plays an important role in learning
process and some students really need properly motivated.

On the other hand, the schools or colleges sometimes also play major role in helping and
motivating a student for learning.A better example would be of students who are neglected by
their parents. Some parents don't care much their child’s school or college and they just blindly
believe on the schools and colleges to take care of student’s studies. In such cases the schools
and colleges do have great responsibility to motivate the student and lead him to good learning
habits. So sometimes depending on type of student the schools and colleges also play important
role in motivation.

On conclusion, the learning is surely the matter of self-discipline in most of the cases. The
schools and colleges do play their role in motivation depending on the situation of the student,
but even with the greatest motivations it is on the student that how much he or she gets
influenced by motivation for learning.
Scientists and other researchers should focus their research on areas that are likely to benefit the
greatest number of people.
Write a response in which you discuss the extent to which you agree or disagree with the
recommendation and explain your reasoning for the position you take. In developing and
supporting your position, describe specific circumstances in which adopting the
recommendation would or would not be advantageous and explain how these examples shape
your position.
Any major scientific discoveries and inventions are conducted by long-term researches in
advance and some of them experienced some insuperable difficulties before reaching the goals.
That is the reason that there are many research groups has been founded for centuries on a
paricular project that scientist foreseen its benefit for further progress rather than predicting its
immediate applications.

First, throughout history, many eminent concoctions were accompanied with several decades of
research. Gunpowder, for example, was the one of remarkable inventions in human history.
However, the process of discovery on gunpowder was full of danger and difficuties. Nobel, who
devoted his life on the research of gunpowder, experienced insuperable difficulties. In addition to
unpredictable consequence, Nobel's contemporaries and the society did not understand the
ramification of gunpowder and derided his effort as useless and myopic. Despite of critisim from
others, Nobel insisted on his project and eventually reached the success. Nobel was a foresight
sicentists; his comtemporaries, on the other hand, shortsighted on the potentials that masked
underlying. What must be addressed, however, is if Nobel acquesced to the authorities and those
myopic contemporaries, the discovery of gunpowder would have been delayed for several
centuries, and consequently industrialization would have been far behind the progress of
civilization.

Likewise, the research on the modern science are not able to achieve immediate recoginition but
would benefit prospects in long-term. The groundbreak in quantum mechanics, as one of the
most preeminent breakthroughs in twentith century, trigger the revolution of the discovery in
universe, as well as the revolution in information technology. Had Albort Einstein gaven up his
research due to the fact that he would not be able to see the achievement in short-term, the entire
progress of modernization would have been all but impossible. Furthermore, in addition to
unforeseen results in researches, some projects do not benefit to human straightward but make
great contribution to understand the world around us. Biologist conducted research on fishes,
which triggered the invention of submarine in military; Chemists run the projects on several
unknown elements, which revolutionized the development of medicines. All of which are not
observed results in short period benefit contemporaries and posterities.

In conclusion, it is clear that researchers need to be professional and proscient on their


researches, in spite of influence of other people and authorities. That way they will reach their
goals, which reinforce the discovery in the fields and consequently benefit society as a whole
Politicians should pursue common ground and reasonable consensus rather than elusive ideals.
Write a response in which you discuss the extent to which you agree or disagree with the
recommendation and explain your reasoning for the position you take. In developing and
supporting your position, describe specific circumstances in which adopting the
recommendation would or would not be advantageous and explain how these examples shape
your position.

Who has not seen politicians fighting each other? Think about it and it is probable that you have
seen as politicians of your own country as foreign politicians dealing with personal disputes
between them, which could not have reached a good solution or simply ended nowhere.
Therefore, politicians should come to an agreement instead looking for personal or party ideals.

Many good bills stay "behind scenes" waiting to be supported by a majority of congressmen in
order to be applied. Due to this delay, many benefits for the population are sadly lost. For
example, in Peru, many congressmen debated a law related to increase the profits to medical
services for the people. A necessary policy for my country. This took several months to every
politician gets consensus and this, consequently, led medical doctors to be on strike. This action
prejudiced the population the same quantity of time that the bill was delayed to be approved. In
addition, this problem is not particular to my country, United of States could face possible
economical problems because its politicians do not come to a common ground about the country
budget. Thus, political disagreements led bills to be delayed which can harm not only few people
but the entire country.

At present political conflicts between countries have born in order to satisfy particular interests
which lead to no consensus between parties. These conflicts could end in wars, which could
bring a high death toll. For instance, the conflicts in the Middle East have been there a long time
for reasons which involve the control of some zones. Many intents of get an agreement between
parts have taken part of the history of the conflict for example, between Israel and Palestina, but
this only brought more casualties. Similar to this example, many countries have asked the help of
The Haya Court in order to resolve conflicts for sea delimitation as for instance, between Peru
and Chile. Both parties have had meetings to arrive in a consensus but there was no agreement
because of Chilean interests. No one knows the possible response of Chile but its president
mentioned that Chile will make to respect its territory with the force independently of the
decision of the Court. If politicians between different parties pursue a common ground, it could
avoid many people lose their life.
It is true that many laws have been polished before they were approved for a majority of
congressmen due to different thoughts between parties. If all the congressmen had come in
agreement with the bill as soon as it was proposed, had it been really a good law if this had
approved at first instance? If it was polished before, it is probable that the answer was No. Thus,
because of politicians do not always think in the same way, this situation has permitted that bills
have been enhanced before it was applied beneficiating the population. For example, a bill which
regulated recently the private pension fund of workers in Peru. At first instance, the party of the
government had made a regulation of this fund which prejudice in certain form the workers. The
opposition noticed that and negated to approved this bill unless this be checked before. The result
was a stronger law which got rid of excesive costs of the entities in charge and did not threaten
the stability of the worker's funds.

In conclusion, if political forces do not arrive in an agreement, many beneficial bills could stay in
written words and not to be applied and many army conflicts could be developed, which may
lead people to worse situations than they are now.
People should undertake risky action only after they have carefully considered its consequences.
Write a response in which you discuss the extent to which you agree or disagree with the
recommendation and explain your reasoning for the position you take. In developing and
supporting your position, describe specific circumstances in which adopting the
recommendation would or would not be advantageous and explain how these examples shape
your position.

Risky action, by definition, is that which it has the possibilitiy that people might not achieve
what they expect throught actions.While proponents of the view that higher risk higer return,
applying the case in stock market, believe that risk comes with opportunity, others assert that
people need to be meditative whenever they attempt to hold risky position. After weighting the
evidence, it is clear that people need to control the risk by full evaluations before any impetuous
action being taken.

Considering the example of the investors in Asia stock market thirteen years ago. The economy
seemed to be prosperous and adventureous when investors enjoyed the benefits that stocks
markest brought, though they were forewarned about the volatile in the market. Consequently,
many investors held risker actions than they used to, without considering any consequence that
might occur, which further exacerbate the crisis that outbrusted later. So when the economic
recession was sweeping out the Asian countries, those who ignored the consequence of prepitiate
actions suffered most and resulted in the banks which loaned them.

Likewise, in politics, governors need to fully evaluated the risk might result from the implement
of a new policy as capricious adjustments leads to unstailization of society. Twenty years ago,
Japan, as the second economic mainstay of world, implemented its unprecedent finance reforms,
without being concerned about proper risk evaluations and whether it fitted the national
economic development,copying from other countries. Consequently,It caused the scour of mass
unemployment in the country and instead of managing the risk, Japan took impetuous action ,at
the absence of risk control, implemented the previous plicy back which aggravated the problem.
As a result of the prepitiated decisions, the economics collapsed and citizens had to take the
consequence that those irresponsible politicians brought them.

Nevertheless, it can be argued that great opportunities are fleeting, particulary those seem
extrorndinary. However, what must be addressed is without evaluating the risk that the actions
might result and making strategy to avoid, it is impossible to reinforce the chance to achieve the
goals.

In final analysis, it is certain that being pensive to the consequence of action is categorical to
achieve the results with minimum risk. At the absence of relevant risks evaluations would only
lead to unpredictable consequence and maximize the risks.

Leaders are created by the demands that are placed on them.


Write a response in which you discuss the extent to which you agree or disagree with the
statement and explain your reasoning for the position you take. In developing and supporting
your position, you should consider ways in which the statement might or might not hold true and
explain how these considerations shape your position.

First, we would have to define a leader, and what characteristics does it have. A leader is
someone that excels at his job, no matter if he is a student, an intern, a junior worker, a senior
worker, the boss of the company, a politician, a soldier, anyone can be a leader if he wants it
enough. The world leader by no means has to be confused with boss, leaders can be bosses, but
not all bosses are leaders. A leader is not created by the demands that are placed on them, but by
the actions he takes on those demands and the attitude he has towards them. Leadership is about
knowledge, sympathy, responsibility, delegation, and timing.
A leader knows what he is supposed to do, what are his capabilities and skills, and how he can
improve himself through experience and help from others. He not only has the knowledge
needed for his job, but also to make improvements and offer new and more practical solutions
for the problems he deals with. He also has to know when he can assign part of the
responsibilities to other people to help him; he cannot do everything himself, so he ought to learn
to delegate responsibilities and jobs to others in order to be more productive.

A leader has to be sympathetic, he has to acknowledge that his workers, co-workers or superiors
are human beings, with their own abilities and feelings, and he has to know that they are affected
by them. He has to be capable of form a good environment where he and others feel good to
work, and know when it is fine to let them attend personal situations. For example, a worker that
has a boss of the company that remembers and congratulates him in his birthday, when no other
boss has ever done it, will be happier to work for him. Or if the worker has a family problem,
and the boss don’t let him go to see his family, the worker will probably not want to work for
him or will underperform.

A leader must be responsible, this way, people around him will not only see and recognize his
work, but he could in fact motivate them to do the same, improving results for the company.
Also, being responsible means to know that timing is essential for good results, asking when the
job is due and exactly how they need it to be done, will allow people to hierarchize work and get
results in time and with good quality.

Outside of work, all of these qualities serve equally important purposes, with our families,
friends, and with society in general. A leader could somehow grow with the demands that are
place in them, but most of all, someone becomes a leader when it learns how to deal with these
demands, and how to have a good attitude about it.

There is little justification for society to make extraordinary efforts — especially at a great cost
in money and jobs — to save endangered animal or plant species.
Write a response in which you discuss the extent to which you agree or disagree with the
statement and explain your reasoning for the position you take. In developing and supporting
your position, you should consider ways in which the statement might or might not hold true and
explain how these considerations shape your position.

Today, extinction of species is a big problem in ecology. The number of species in the world has
been decreasing since humans started to develop the nature. Wildlife has lost its habitat and has
been hunted indiscriminately by people. Therefore, many people argues that the government
should take extensive measures to protect species which are endangered by human activities in
order to protect ecological diversity. I agree with the argument.

Admittedly, human intervention may yield a bigger chaos to the ecosystem if we make efforts to
protect species which are endangered because of natural processes, all species in the ecosystem
are interconnected. In other words, if one species is affected, other species will also be affected.
We can find one good example in an effort to protect wolves in a region. In the region, the
number of wolves decreased drastically due to excessive hunting. Therefore, scientists
maintained that the society should protect wolves, and the society did so. After few years,
scientists found that the number of wolves was significantly increased; however, they also found
that the number of deer was significantly decreased. Thus, the society lifted the ban to hunt
wolves and protected deer. As a result, the number of deer was increased; however, the amount
of grass was plummeted due to feeding of deer. Finally, the society decided not to intervene in
the ecosystem, and the number of every species restored to the equilibrium state.

However, doing nothing may cause bigger disaster to humans. As mentioned above, every
species is interconnected, which means decrease in the number of a species may affect humans.
To illustrate, let's think about a scenario. Extinction of carnivores may increase the number of
herbivores. Some herbivores feed on flower. Thus, increased number of herbivores may lead to
decrease in the amount of flower. Bees cultivate honey from flower, so decrease in the amount of
flower may lead to decrease in the number of bees. Bees are important to human life because
they are pollinators. Without bees, plants cannot be pollinated; therefore, there will be no
fruition. That means the amount of food will significantly decrease. As we can see from this
scenario, not protecting endangered species may take a toll on humans.

Moreover, humans are the only species can make efforts to protect endangered species. Even
though dolphins and monkeys are smart, they cannot think of protecting endangered species.
Human is the only living thing that can think rationally. As a member of the nature, human
beings have obligation to go the extra mile to protect the nature. Therefore, people should protect
endangered species.

Efforts for saving endangered species may disturb the ecosystem. However, not protecting the
species may cause bigger problems--such as food shortage--to human beings. Therefore, we
should make efforts to save the species to the extent that the efforts do not severely disturb the
ecosystem.
The human mind will always be superior to machines because machines are only tools of human
minds.
Write a response in which you discuss the extent to which you agree or disagree with the
statement and explain your reasoning for the position you take. In developing and supporting
your position, you should consider ways in which the statement might or might not hold true and
explain how these considerations shape your position.

Some people may contend that machines overcome human minds whereas others argue that
human minds are superior to machines. Admittedly, both ideas are equally important, and, in
fact, these two approaches are mutually exclusive. However, as far as the fact that all machines
are created by human beings is concerned, the idea that machines are not so superior as human
minds is crucial.

The chief reason for my assertion lies in the assumptions that all machines and computers cannot
work without human beings. If we analyze that all machines and computers are programmed by
engineers, and following the programs, we can have a certain level of agreement that machines
cannot overcome human’s minds. Let us clarify this point by using several cases.

First, machines do not have minds. All machines, as I pointed out, cannot consider the cases that
they are not programmed. Even though it is true that translators can translate quite accurate;
however, they are still unable to understand connotations and nuances. Besides, there are a lot of
safety equipment on recent cars, but cars themselves cannot predict and prevent traffic accidents.
For instance, drivers must step brake to avoid accidents. These examples demonstrate that
machines do not have mind to care about the users if they are not programmed so.

Second, machines cannot create new things as human beings are continuously doing because
computers can do what engineers programmed. No scientific theories are found by only
machines or computers themselves. Calculators cannot create new mathematical theories. Super
high quality telescopes cannot find any new stars or galaxies. Coffee machines do not provide us
unprecedented coffee. Without a doubt, machines cannot improve without human beings’
knowledge.

However, still some opponents disagree with my viewpoint. They may contend that calculators
can tally much faster than people. Moreover, chess machines can win the best chess player in the
world. It is true that machines overcome people’s ability; however, to perform so, they must be
programmed so. Unless engineers program a machine to calculate faster, calculators cannot even
compute.

I strongly believe that human minds are more superior than machines in terms of the fact
machines are creations of human beings is concerned. This is because machines cannot consider
expansively, and, furthermore, cannot create unprecedented things. Even if the great abilities and
skills of machines are considered, still they are all based upon human’s ability.
People who are the most deeply committed to an idea or policy are also the most critical of it.
Write a response in which you discuss the extent to which you agree or disagree with the
statement and explain your reasoning for the position you take. In developing and supporting
your position, you should consider ways in which the statement might or might not hold true and
explain how these considerations shape your position.

People who are the most devote them self to an idea or policy also the most critical of it. It is
obvious that who ever deeply committed to an idea, they are also criticize their idea or work to
get better result. Every try or every new idea to support prior one they always in the term of
questionize their work an idea. I agree with the statement that not only history but also our
generation has these kind of people.

When we look at past, we can see a lot of thinker, inventor, committed people who were deeply
focus on their idea. Albert Einstein, Graham Bell, Isaac Newton, Thomas Edison, these are
inventor the first come to my mind. We all know that Before the invention of bulb, Edison tries
his examination a hundred thousand times, but he never gave up. Because he committed an idea,
and he is the most critical one to his own idea. In this manner these people are successful, the
more critical their self the more success is on their work. Our generation has also many people
who most deeply committed their idea like Steve Jobs, when we look at his career, he humbled
over and over but he never quit to criticize himself. Now we all using these people invention,
how much effort they showed in the past how much we know all of them.

Moreover, all these people was seen crazy in their time, most of them rejected by their society,
even they were sometimes punished because of their ideas. But all these criticizes only forced
these genius people to commit more to think more. Their ideas and was getting more sharp with
every critical. When I see these people, I found them really strong. This is really hard work, they
committed their self on the other hand a lot of people criticize them. Thus, this is the proof of
how criticize their self. Their criticism was much more than the other people hence they were so
successful. Otherwise if they listened garrulous criticism they never be finished their idea and
work.

There is another way to look at this too, some people may think that the more commitment to an
idea and policy creates more unreliable person, like some of the politicians. When we look at
them, most of them like to get more and more favor instead of criticism. However, these
politicians are from the commitment, and far from the hard work, that is the reason they can not
stand on criticism. Moreover, we can not agree that these people are the most deeply committed
to an idea or policy, these are only representative of the some specific group, only a talker not a
doer.

In conclusion, the real deeply committed to an idea or policy people has always come to world,
and they found most vital idea or work as a result of their devoutness to an idea or policy. They
criticized their self more than anyone to get the best result.

Some people believe that society should try to save every plant and animal species, despite the
expense to humans in effort, time, and financial well-being. Others believe that society need not
make extraordinary efforts, especially at a great cost in money and jobs, to save endangered
species.
Write a response in which you discuss which view more closely aligns with your own position
and explain your reasoning for the position you take. In developing and supporting your
position, you should address both of the views presented.

Healthy ecosystem is a fundamental component of the balanced development of society. It is


based on the intervention of all living creatures present on planet earth. By making changes in
the natural environmental resources on earth we do not expose animals and plants to the negative
effects. We are also exposed to consequences that affect us negatively. Therefore I believe that
our duty is to save endangered species from extinction and society should try every plant and
animal species.

Firstly, mankind has an inherent belief that earth belong to them alone. They do whatever they
want to make their life easier. They are ignoring the fact that all living creatures have the right to
be protected from the cruelty of people. Animals have the right to live safely without being
threatened by loss of their main food. To illustrate, the extinction of the bamboo plant, the main
food for the panda has affected it greatly. The Panda has to go to other unfamiliar places
searching for food and expose itself for dangerous places and live in different environment.

Secondly, wood is being used in most of the industries with large amounts. Therefore,
deforestation has been the solution for getting use of the wood in trees, leaves and barks for
making papers, and certain medicines. On the other side, we are being affected negatively by
decreasing the amount of oxygen in the clean air and thus affecting the health of humans. All
species should live in equilibrium on this planet because any change in any of its components
will lead to imbalance.

Finally if the society did not make any extra ordinary efforts to save endangered species of plants
and animals, this will lead to their mass extinction and earth would be one day a deserted place
with no plants in it. For example, the government had imposed fishing restrictions on some
species of whales that are endangered to be extinct because of human activity.

To sum up, all species of plant and animals are part of the ecosystem. If any part of this cycle is
affected, the whole system will be disturbed. And mass extinction on earth has been a result of
human intervention which should be controlled and minimized if threatened the natural
environment on earth. That’s why believe that society should try to save every plant and animal
species, despite the expense to humans in effort, time, and financial well-being.
Some people believe that the purpose of education is to free the mind and the spirit. Others
believe that formal education tends to restrain our minds and spirits rather than set them free.
Write a response in which you discuss which view more closely aligns with your own position
and explain your reasoning for the position you take. In developing and supporting your
position, you should address both of the views presented.

Some people contend that the formal education would set our minds and spirits free whereas
others argue that formal education would restrain our minds and spirits. Admittedly, both ideas
are equally important, and, in fact, history informs us that these two approaches are mutually
exclusive. However, as far as formal education is defined as public education and not education
at cramming schools, the idea that formal education would restrict our minds and spirits is more
crucial.

The opponents to my viewpoint may insist that formal education can expand people’s knowledge
equally. Even though students are poor, they can take a certain level of education. Moreover, if
they seek to study further, they can take upper-level education by receiving a scholarship. It is
definitely true that everyone can take public education with the little amount of money; however,
acquiring knowledge does not guarantee formal education would set our minds and spirits free.
Let us clarify this point by using several point of views.

First, public educations are usually censored by governments; therefore, students often fail to
acquire comprehensive knowledge. Most Chinese students do not recognize China as a
capitalism country but a socialism country due to Chinese public education. Japanese students
tend to lack Japanese negative historical part, especially after the beginning of World War
Second. Korean students have learned distorted historical facts about the invasion by Japan
during World War Second. Hence, it is obvious that governments control public educations.

Second, public educations are score centered education. Students need to take a lot of exams to
pursue their education. Students need to take SAT and ACT to enter undergraduate school. GRE
or GMAT are mandatory for most students who seek further study in graduate schools. Addition
to these standardized tests, students need to pass mid-term and final exams to pursue their higher
grades. Not only the education in the United States, but also other countries’ education are exam
centered. For instance, Japanese students also have mid-term and final exams and need to take
several distinct standardized tests to enter even junior high school. Students recognize their
academic achievement by their scores by following public education. Hence, public education
would provide the education for exams, not for “think out of the box”.

I strongly assert that formal education would restrain students’ minds and spirits because this
education is biased by government and students would concentrate on only exams. Some people
may challenge to this idea by referring to the opportunity of acquiring knowledge; nevertheless,
knowing does not guarantee the free spirits and minds.
Young people should be encouraged to pursue long-term, realistic goals rather than seek
immediate fame and recognition.
Write a response in which you discuss the extent to which you agree or disagree with the
statement and explain your reasoning for the position you take. In developing and supporting
your position, you should consider ways in which the statement might or might not hold true and
explain how these considerations shape your position.

It is said that young people are the future of a nation, and they will define the shape of a country
in the coming generation. Taking this into account, I believe, youth should be encouraged to
pursue long term and realistic goals, but at the same time they should not forget about the present
they are living in.

Young people, especially teenagers are a capricious and whimsical lot. They need proper
guidance and counselling to grow in life. Instilling in them, a thought to focus on long term goals
will help them in maturing, and contributing to the society they live in. Youngsters who know
their goals will take positive steps to achieve it, rather than wandering about in search of
recognition. This will make them more disciplined, punctual and mature. Such youngsters will
achieve their targeted goals (say becoming scientists, engineers, researchers) and these human
resources in future will be of great help in the overall growth of the society/country. On the other
hand, focusing only on fame and recognition may make them wayward, without a proper
direction. In the attempt to seek immediate fame, young people may even take wrong and
immoral steps, thereby harming the society and in the future even themselves.

However, it should noted that, blinders (used to make horses see in a single direction), should not
be imposed on youth, such that they focus only on long term goals and forget the means by
which they are planning to achieve it. For example, a teenager whose goal is to become rich in
future may do so, by a series of robberies, thereby achieving his goal. But this kind of mentality
should not be fostered. Rather, youth should be taught proper means to achieve their goals.
Another problem which may arise due to singular focus on long term goals is lack of interest in
present and living in future. This may harm the young people psychologically.

So, in view of these points, I believe, young people should be encouraged to pursue long term
goals, but achieving such goals should be made a continuous process, such that they continue to
do well in the present, live in the present, while at the same time, get closer to the goal step by
step, achieving a part of it every day, which accumulates slowly to get them to their ultimate
goal.
Governments should not fund any scientific research whose consequences are unclear.
Write a response in which you discuss your views on the policy and explain your reasoning for
the position you take. In developing and supporting your position, you should consider the
possible consequences of implementing the policy and explain how these consequences shape
your position.

Scientific research is an important segment of scientific and social development today. The
money governments commit to the research is constantly increasing every year. In order to make
the most efficient out of this money, someone recommends that governments should only fund
the research whose consequences are clear. In my opinion, this recommendation has a good
standing point but lacks of some considerations of other important factors.

First of all, this recommendation oversees the potential of long term consequences of scientific
research. It is doubtless to me that long term outcomes are crucial, sometimes long term effects
are even more important than short term ones. Take scientific research in agriculture as an
example, if we want to test a new kind of pesticide on vegetables such as tomatoes, a significant
amount of researching and experiencing time should be expected. Not only because we need to
wait for tomato to grow, but also because we want to collect data of remaining pesticide from
offspring. It may be difficult to identify any upcoming results for scientific research like this at
first. However, governments should still support it because it helps improve the quality of food.

Moreover, in circumstances like space scientific research, the government should provide
financial support even though the outcomes are unknown. We have spent a lot of money in space
studies, for example, building up satellite spaces stations and developing new spaceships. Is all
this money meaningless? Of course not. Scientists are learning more and more about the space
though the way with these developments. If we cut the budget of space research because there is
no clear consequence, we would never understand what surrounds us, which consequently cuts
any further development in the space.

Nevertheless, this recommendation may be adapted under certain situations that require fast
results with limited budgets. Many university researches would fall under this category. Hence
this recommendation is beneficial for select the best candidates quickly; government official may
consider it upon their needs.

To sum up, I partially disagree with the recommendation that only scientific research with clear
outcomes could get funded by government. Even though this recommendation may be useful in
some situation, a detailed case by case analysis should be conducted before any action has been
committed.
Knowing about the past cannot help people to make important decisions today.
Write a response in which you discuss the extent to which you agree or disagree with the
statement and explain your reasoning for the position you take. In developing and supporting
your position, you should consider ways in which the statement might or might not hold true and
explain how these considerations shape your position.
May the knowledge about the past help us to make a right decision today? There is no simple
answer on this question because contemporary world is dynamic and mercurial; at the same time,
as far as I am concerned usage of our past experience may help us to avoid repeating of past
mistakes in global and personal levels. Thus I strongly believe that with some limitations
knowing about the past may be extremely beneficial for making thoughtful decisions of today.

To begin with, many of us see no use in our past because contemporary world is dynamic and
complicated as never before. Many recent discoveries and inventions has changed the world in
new unprecedented way, for instance, the internet and personal smart phones have become
ubiquities and have changed the way we work, interact with each other and even think. From this
viewpoint the past seems to be useless and even harmful for making today's decisions. For
example, if we want to invest our money in today's market, our past experience may suggest
investing in hardware, heavy machinery and steel manufacturers which used to be highly
profitable; however, wise and acute person will be invest in the internet enterprises and software
companies which much more promising in today's situation. In other words, the data of past
seems to be rather obsolete and consequently perilous than useful. Thus past is meaningless for
making today's decisions in intensively developing world.

However, closer look at the example above reveals an interesting situation. The predictions of
future in both cases are based on past experience in both cases. From this point of view, our past
experience is indispensable source of precious information which should be wisely used. In this
case, it may be significantly helpful for making today's decision in global and personal levels.
For example, many science, perhaps all of them, collect and combine information and knowledge
to create new theories which, in their turn, are used to forecast probabilities of future processes.
Macro-economy may be a good example of this idea due to the fact that it was funded in
response on the great depression in attempt to preclude repetition of such situations. For instance,
when in 2008 American economy faced crisis, the government used macroeconomics concepts to
revitalize it. Money was lavishly spent on social services; infrastructure and banks received bail
outs. As a result of those government's actions, which were based on past experience, today
state's economy prospers and thriving.

In conclusion, although it might seem that today's world is so strikingly different from the past
one that knowing the history is meaningless for making decisions of today but in reality more
careful look at the situation demonstrates that the past gives us a precious source of information
which, if wisely used, may help us avoid repeating mistakes of past and make right decisions as
well.
In this age of intensive media coverage, it is no longer possible for a society to regard any living
man or woman as a hero.
Write a response in which you discuss the extent to which you agree or disagree with the
statement and explain your reasoning for the position you take. In developing and supporting
your position, you should consider ways in which the statement might or might not hold true and
explain how these considerations shape your position.

Indeed, because of fierce competition around the entire globe, individuals tend to take any step
that can help them to reach acme of success. Today, most of the people do not think about the
path that will lead them towards the success. Some may even opt for ill considered or unethical
route for becoming front runners of their field. And once they succeed in achieving so, people
term them as "hero" of the respective fields. But then this might result in dooms day for their
followers, the day when they find out what their real heroes are made of.

Latest example of cycling legend Lance Armstrong supports the above stated view. It shattered
millions of hearts when the fact that Armstrong might have consumed drugs came into reality.
He was the man that inspired millions of people who where and who are fighting cancer. It is
was indeed appropriate to mark him as "hero" considering the fact that he fought bravely with
cancer all through out his life. But this new fact that was brought into open recently, might harm
his heroic stature. The youngsters who want to pursue their career in cycling wills till consider
him as a man to look upon, solely for his achievements in the sport. But one cannot be blamed if
he or she does not consider him idol anymore.

However not all the heroes fall in same category. There are few "heroes" who have achieved
success purely on fair and diligent efforts. And even a fastidious media scrutiny would not tame
them of their heroic stature. The greats like Sachin Tendulkar have always been under scrutiny
all through his career. But no one has ever blamed him for taking invalid means for achieving the
heroic stature. Similarly, People like Dr. APJ Abdul Kalam, Kalpana Chawla and many more are
considered pure and clean heroes because they had always been transparent and fail.

Society expects a lot when it nominates an individual as hero. Heroes have great responsibility of
not only satisfying the expectations of society but also inspire its young population in right way.
Not all people are gifted with these great qualities. And hence not any Tom, Dick and Harry can
be and should be termed as hero. At the end of the day, its depends on society whether to absorb
the positive actions of an individual or to deride him by highlighting his negative actions.
We can usually learn much more from people whose views we share than from people whose
views contradict our own.
Write a response in which you discuss the extent to which you agree or disagree with the
statement and explain your reasoning for the position you take. In developing and supporting
your position, you should consider ways in which the statement might or might not hold true and
explain how these considerations shape your position.

It is true that debating may cause mental stress and even conflict while disagree each other. If
everyone agrees and conforms on one thing, what is going to happen in the world? Of course, it
would be easy to rule the society and the world. But Is it right way to go? Without conflicting
and debating, I believe there would no way to improve the society and the world.

Because we challenge ideas, we are able to discover and learn important things. Historically,
there has been many hypothesis and theories discarded that we thought so true with no doubt.
For instance, people thought the Earth has a flat surface in the past. In 5 BC, Pythagoras came
out with different and ridiculous claim that the Earth is round like a ball. Eventually, he was right
about the shape of the Earth. If we only disagreed what he had to say and ignored it, we would
never know significantly important fact about the Earth. Furthermore, what if Columbus did not
believe that the Earth is round? He was never able to explore the America continent. In this
regards, contradicting views actually give you more opportunity to explore and learn rather than
inhibit learning.

While we only learn and support the views that we share, the other side of views also rises by
finding fallacies of our views. Adam Smith believed in invisible hands that solve every problem
in the market. Many people and government followed and worship about Capitalism “All might
is the money. However, there were people who looked at the relationship between labor and
bourgeois. They asked themselves, “Why are there poor workers, and rich bourgeois?” If there
was no listening and learning from Communism, there would not be the world like today which
governments provide welfare to the poor, and implement the private property systems.

Of course, the life must be easier everyone agree on everything. Since each individual have
different personality and lifestyles, basically, it is impossible that we only learn from people we
share the same views. Friends, teachers, and even, parents sometimes have different views on
certain things in everyday life. One of the aspects we differ from other animals is that we think
reasonably and accept the differences. For the improvement human being, I believe that
disagreement comes at first as an stress and pressure, but it directs us improvement and advances
at the end.

Some people simply do not like to debate or conflict because they do not want worsen the
relationship with which they care and it is very comfortable to be with. However, it is impossible
to only deal with people who share the same views, and it delays improvement of my ideas and
cannot go beyond them. Thus, I strongly believe that every opinions or views should be
considered even though it conflicts with our own despite of stress because it can compensate you
more.

The most effective way to understand contemporary culture is to analyze the trends of its youth.
Write a response in which you discuss the extent to which you agree or disagree with the
statement and explain your reasoning for the position you take. In developing and supporting
your position, you should consider ways in which the statement might or might not hold true and
explain how these considerations shape your position.

Although trends of the yough do reflect some of the features of the contemporary culture, they
are not enough. The vague things such as clothes, talks, and action, are just a small fraction of
the whole contemperary culture. To realize what lies in the center of the cultule, one has to
examine all the prevalent doings of all the people in that society, the old, the middle-aged, as
well as the youth.In addition, there will be some crucial elements that are not expressed
explicitly in the habits of the public, which needs the detailed scrutiny into the culture of the
nation such as history, art, literature, etc.

First, though important, sometimes even overshoweding other traits in the culture, the prevaling
of the youth is just part of the reflection of the contemperary culture. They indeed tell us about
the shape of the group of people, such as what they like, what lies in their heart, and what do
they really long for. However, the "some" reminds us the fact that studying the trends of the
youth is not enough, which will be supplementary with the full research in the action in the daily
life, preference, and the responds to changes of the majority.

Moreover, through the analysis on the thorough people in the society, more will be found out and
concluded. For example, the style of drinking tea is one of the cultural traditions in China, which
is still maintained by the age more than 40 while the youth just prefer the popular beverages such
as coke, orange juices, etc. Studying the whole population, though exhausted, will give the
researchers more insights on the contemporary culture, some even profound.

Furthermore, there still some essential parts can't be simply conveyed as trends, but one has to do
a full-aspect studying such as gaining knowledge of the history, arts, literature, or taking a glance
at the books, not the popular volumn but the thoughtful one. There are many ideas, traditional
culture, and doings that people gradually neglected as one member of the contemperary culture,
while underlying in the very spirit of eveyone. If one wants to dig them out so as to understand a
culture, he has to studying all the books, history which is the shared experiences of the people,
and many other materials.

In the end, to get an insights on the contemperary culture effectively, one has to examine not
only the trends of the youth, but also the prevalent tendencies of the majority and other materials
such as books, histories, arts in the society. Simply understand things related to the youth is not
enough.
People's attitudes are determined more by their immediate situation or surroundings than by
society as a whole.
Write a response in which you discuss the extent to which you agree or disagree with the
statement and explain your reasoning for the position you take. In developing and supporting
your position, you should consider ways in which the statement might or might not hold true and
explain how these considerations shape your position.

It is popularly believed that our behavior and attitude is widely determined by our nearest
surroundings. However, is this viewpoint unquestionable? As far as I am concerned the main role
in forming our attitude belongs to society, it creates our personality, system of values,
understanding of appropriate and inappropriate conduct and gives us schemes or patterns of
social behavior.

First of all, the role of society on us is enormous. We are the product of society and our reaction
and attitude is widely determined by the society. First of all our system of values and the way by
which we perceive the world is the product of the society and without social impact we have
never become human beings. For instance, the understanding of appropriate behavior differs
significantly in each culture. In western society we tend to be individualists and we see our own
interests more important than others. In Asian culture we have an opposing understanding of the
issue. Moreover, even such fundamental concept as death is distinct in each culture. In African
society, for example, if a child dies, it is not a reason to cry owing to the fact that the child has
done nothing to the social group. However, if a elderly ceases his existence, the tribe will be in
deep sorrow because an important member of the group has passed. In western society, we once
again have opposing perception of death. For a European a death of a child is unrealized
opportunity and therefore the reason to regret, the death of old person is just the way of life. In
other words, our reaction on many grave issues is determined by our society.

At the same time, we are not rigid; we are able to adapt to new condition, however, our ability to
change is limited by our personality which is the product of society. For example, if a person
moves to a new culture, in many cases one will mimic the habits and traditions of new culture. A
Russian who moved to USA, for instance, may start to celebrate national holidays etc. At the
same time, the possibility to adapt to new surroundings is limited. Imagine that you started to
live in Iran, this is a country with strong Muslim traditions. In this culture, for example, woman
is limited in her rights. Will you accept this tradition? In many cases this point of view is
disparate to personal beliefs and therefore it will not be accepted.

In conclusion, our personality, attitude, the way by which we react are mainly determined by
society. It forms us and our view point. Despite the fact that we can adapt to new conditions our
possibility to change is restricted by our personal principles which are settled by society.
Nations should suspend government funding for the arts when significant numbers of their
citizens are hungry or unemployed.
Write a response in which you discuss the extent to which you agree or disagree with the
recommendation and explain your reasoning for the position you take. In developing and
supporting your position, describe specific circumstances in which adopting the
recommendation would or would not be advantageous and explain how these examples shape
your position.

Art is a way for an individual to express his opinions and feelings but apart from this definition,
art also refers to the skill of men specialised in creating things of aesthetic value. And to view art
only as a luxury is wrong. Because art can certainly lead to commerce and help individuals earn
money and thus feed their families.

A nation should always work for the welfare of its citizens and the their upliftment. A nation
whose significant number of citizens are hungry and unemployed should focus more on
developing schemes that help tackle these problems. Therefore, it is wise on part of such a nation
to spend majority of its funds on applying the measures to tackle the problem of unemployment
and hunger. As such art does not feed the citizens of a country but suspending fundings on all art
forms is not a wise decision. For example, If a nation is known for its handlooms and special
textiles, which have a great demand and value in international market, should not suspend
funding for its textile industry.Instead, in such cases, a nation should encourage more people to
pursue this art form and provide more funding for the art made by handloom workers. Because,
as more people get engaged in this art form the output from textile industries augments and as a
result more influx of foreign currency starts when this art sold in international market. Hence,
this in turn bolsters the country’s economy and simultaneously solves the problem of
unemployment and hunger.

In some cases, though funding in art may not essentially help increase in exports and influx of
foreign currency, but it may help other connected industries to create more job opportunities. For
example, funding to develop the aesthetics of a public park may help in creating more jobs for
artists and architects, who in turn will employ more people under them and thus help tackle
unemployment. Such actions can help tackle unemployment immediately without any delay,
which is better because the measures taken by a nation to improve its economy are generally
long-term and time consuming.

Some people may argue that a government should focus all its fundings towards strengthening
the economy and also towards the industrialisation. But they forget to acknowledge that though
industrialisation is necessary and important it should be done gradually because it leads to more
unemployment as machines take over the jobs of men.

Hence, in conclusion, though a nation should primordially focus on sustainable and stable
economic measures to tackle hunger and unemployment, it should not completely suspend its
funding for arts. As arts can definitely lead to commerce and also can help create more jobs in
fields attached to it. Also, while economic measures take long time to be effective, funding for
arts can actually help tackle serious problems more effectively and immediately.

All parents should be required to volunteer time to their children's schools.


Write a response in which you discuss the extent to which you agree or disagree with the
recommendation and explain your reasoning for the position you take. In developing and
supporting your position, describe specific circumstances in which adopting the
recommendation would or would not be advantageous and explain how these examples shape
your position.

Currently, it is debatable whether all parents should required to volunteer their time to their
children’s school. Some people hold the view that all parents should adhere to this
recommendation. Others, however, disagree with the view and believe that this recommendation
is unrealistic. In my opinion, although both options might present meritorious points, without a
doubt, I would settle upon the latter idea because forcing some parents to volunteer some time in
their children’s school is cogent at best.

Admittedly, in some cases, parents’ participation in children’s school is beneficial for students
since those parents are be able to provide some crucial teachings that are not taught by
schoolteachers. To illustrate, although teacher are be able to teach some detriments of drug usage
to their students, but in most of cases, they are not able to provide full information about them.
However, if parents are expertise in this field such as policemen, they can possibly inculcate
students with the adverse consequences of drugs such as side effects and penalty. In such a case,
volunteering some times to their children’s school definitely reaps a myriad of benefits to
schools and students. Therefore, the recommendation of parental volunteering in school is
necessary in some cases.

In addition, volunteering in their children’s school is essential since parents can ameliorate the
school environment and be aware of their children’s progress. To be more specific, through PTA
(parent Teacher Association), parents meet with school staffs and teachers and communicate
about the needs of the students. By doing so, they can provide more effective and amicable
school environment to students. Also, by meeting with teachers, parents can know about the
progress of their children. It is important since parents are able to provide them private tutors if
their children fall through the cracks. Thus, participating in volunteering is indispensable for
some parents.

However, enforcing all parents to involve in school activities is illogical. For example, in our
society, where a wide array of families is faced with economic difficulties, it would be arduous
process for some parents to volunteer. Since they are strapped for cash and barely making a
living, many are working overtime, which makes it difficult for them to volunteer. In other
words, juggling with work and volunteer can actually take a toll on the adults. In such
compelling cases, the involvement of parents in their children’s school should not be vital.

Based on what was previously conferred, we may conclusively claim that recommending all
parents to volunteer in their children’s school is not persuasive. Therefore, the recommendation
should be forced to only those parents are able to abide by it.

Teachers' salaries should be based on the academic performance of their students.


Write a response in which you discuss the extent to which you agree or disagree with the
recommendation and explain your reasoning for the position you take. In developing and
supporting your position, describe specific circumstances in which adopting the
recommendation would or would not be advantageous and explain how these examples shape
your position.

The central assumption of this argument is that students’ performance would improve if teachers
salaries depend on how well their students perfom. However, this claim is flawed because other
factors are responsible for student success, it disincentivizes students from being responsible for
their education and teachers may focus on academic success rather than imparting skills.

It would be unfair to base teachers’ on their students’s success because there are other factors
that contribute significantly to academic performance. For instance, family environment and
parent involvement affect student performance. If a student faces challenges at home, it may be
impossible for the teacher to do much at school. Thus, it is unreasonable to evaluate a teacher on
that student’s performance especially if the number of such students are large in a class.
Similarly in places where students do not speak English as a first language, early years of
education may be difficult as they are just adjusting to a new system and performance may be
low. In this case also, holding the teacher responsible is unreasonable.

Another reason why teachers salaries should not be based on students’ academic performance is
that it leaves teachers at the mercy of their students. For instance, if a particular teacher is not
well-liked, student may sabotage her and perform poorly just to punish the teacher. New York
City offers an example of the demerits of giving students power over teachers with the “Rubber
room” cases. Thus this position would be detrimental to teachers and students.
Also, basing salaries on student success disincentivizes teachers from giving their best. Teaching
is not a financially lucrative career and most people who teach probably do because they love to
impart knowledge. However, if there income is in danger and unstable, they may resort to
dishonest means to achieve high academic performance. At the end teachers, students and the
society lose.This is because the students do not learn the skills they should but are prepared to
pass examinations thereby producing half-baked graduates for the society’s labor markets. The
scandals that have resulted from standardized testing as a determinant of funding also prove why
this position is flawed. Stories have emerged on how schools help their students cheat just to
pass examinations.

Nevertheless, the underlying assumption of the above claim is reasonable and a position such as
basing teacher bonuses on performance may work rather than teacher salaries.

In conclusion, though the premise of this position is sound, such a position is detrimental
because teachers are not solely responsible for academic performance, students may sabotage
teacher and this would also discourage teachers from pursuing the true goals of education.

It is no longer possible for a society to regard any living man or woman as a hero.
Write a response in which you discuss the extent to which you agree or disagree with the claim.
In developing and supporting your position, be sure to address the most compelling reasons
and/or examples that could be used to challenge your position.

Man is a social living being and lives in a society. In order for a society to progress there are few
heroes who are recognized even after they are gone. The above stated claim is not true it is
possible to regard even living man or woman as hero.

We are safe at our home and have a good sleep at night, all of it is possible by our soldiers who
are guarding and protecting us from enemies entering our country. The soldiers are sometimes
unsung heroes they need to be appreciated, awarded for their bravery and the sacrifices they
make for us. There are many Iraq war veterans who have come back home but still they are
living in battle. Almost all of them face post-traumatic stress which needs to be treated carefully
and given proper jobs after they are retired from military life. They have families which they stay
away for long time, most them do not get to see their kids grow. All the soldiers are heroes of
their society.
The other heroes are the ones who protect law and put their life in danger despite the risk they
know. All police officers fight crime within the society and many lives are lost. They make
society a better place for people to live. Again, the society should accolade law protectors for
their valor. They should be given decent amount of salary for the risk they take to protect. Most
of these jobs are low paid salary and lesser incentives.

There are few leaders in society like Nelson Mandela who fought against apartheid, racism,
poverty in South Africa and was in jail for 27 years. He was awarded Nobel Peace Prize in 1993.
He is still living and is regarded as hero. There were many recognized before and are no more
like Mother Teresa who fought against poverty and treated the poor. She was also awarded
Nobel Peace Prize. One more thing Nobel Prizes are given only to living people for their
achievement in their field. There are many heroes in history who were recognized when alive
and honored for the values they left behind.

The movies have depicted well the qualities of a hero .If a hero to imagined we all think of
characters superman, Spiderman, he-man, batman, etc., They are all unselfish, altruistic in
nature. There is a hero in all of us we need to follow good qualities and act right at times.

In conclusion, there are heroes who are living, needs to be recognized and awarded like soldiers
and law protectors for their bravery.

Some people believe that in order to thrive, a society must put its own overall success before the
well-being of its individual citizens. Others believe that the well-being of a society can only be
measured by the general welfare of all its people.
Write a response in which you discuss which view more closely aligns with your own position
and explain your reasoning for the position you take. In developing and supporting your
position, you should address both of the views presented.

The author contrasts two viewpoints about the well-being of the people. While some recognize
the well-being of a society as an individualistic issue, the others believe that the well-being only
can be determined generally. Consider the both views in what follows, and then my point would
be delineated.

To begin with, there are those who believe that the well-being of a society is rather an individual
matter. It is upon the each person to have the well-being by attaining a decent education and
holding a decent job to come up with sufficient money in order to have a satisfying life. We see
that even in the societies with the notably high general incomes, there are people who do not
have the well-being because of their own lack of resourcefulness, their own laziness and many
other personal shortcomings.

Resting on the fact that the well-being has some individual dimensions, these people believe also
that society should prioritize the achievements of society over the well-being of the individual
citizens. It is with this approach that society can thrive. Otherwise, should we care too much for
every individual who because of his or her shortcomings in life lags behind the others, the
society may not thrive. We cannot care for the people who are not caring for themselves, these
people believe.

At the same time, on the other side of the spectrum of opinions, people believe that the well-
being of society can just be generally assessed by people's well-fare. Think about a society where
some people are suffering from overweight, while some are starving. Such a distance between
the economic classes of a society, these people believe, defies any well-being to be ascribed to
that society. Hence there should be an average of the well-fare of whole societal classes in order
to assess the well-being.

Finally, as far as I’m concerned, the well-being of people is the combination of the both views
above. Neither we should sacrifice the possible successes of a society to pamper the lazy
individuals lagging behind the others, nor should the society forget the well-fare of public for the
sake of its successes. The solutions would be solved by the participation of both the society and
individuals; society should provide the people with the infrastructures such as job availability,
and then it is the individuals who should earn those jobs in order to gain the well-fare. By this
approach the society provides grounds for the people to fulfill their life, and then it also can seek
its successes.

In short, as discussed, the well-being of a society can be measured both individually and
generally. A successful society which also has the well-being in it, can be ushered in by
participation of both the individuals and the society.
Claim: Any piece of information referred to as a fact should be mistrusted, since it may well be
proven false in the future.
Reason: Much of the information that people assume is factual actually turns out to be
inaccurate.
Write a response in which you discuss the extent to which you agree or disagree with the claim
and the reason on which that claim is based.

Facts are those which guide us in exploring this beautiful world. Without facts, it will be near to
impossible to examine the exciting world of ours. Though facts can be proven to be inaccurate,
they have to be taken for granted because they have gone through numerous tests to get
established as "facts" in this competitive world. There is no means by which we can disprove a
fact in a short period of time. Therefore, as time goes on, if need arises, facts will change. But,
for the present moment we have to assume the facts as true; otherwise, the world will not move
on.

First of all, facts are borne out of research work conducted by scientists. Because scientists have
gone through arduous work to establish facts, the facts cannot be outrightly rejected.
Additionally, researches are conducted for giving a new phenomenon to the world; it does not
need to be always right. The application of the excellent discoveries made by researches is left to
the industries. Hence the facts must given by researchers must be taken for granted and must not
be denied until a convincing evidence is found. Even these facts get established only after the
research paper undergoes severe analysis by eminent professors and peer research scholars. As a
result, the facts carry a lot of meaning in them and serve useful to the world. For instance, Pluto
was discovered to be a planet long ago; it has been believed for so long to be true. But recently,
scientists have researched to find that Pluto does not meet the criteria to be deemed as a planet.
Therefore, we must accept the outcomes of researches as and when they occur, because they are
most often validated by a third party.

It follows that the researchers have the obligation of producing the correct results, because they
are valued the most in the society. At the same time, we must not forget the fact that they too are
human beings and are liable to err, committing mistakes. Consequently, the researchers can make
mistakes at times. But they subsequently get pointed out by fellow researchers or their theories
disproven by succeeding researchers. Hence, the information that people assume as facts are
probable to be proven false; nevertheless, they must be taken for granted because it is the
outcome of the research work done at that particular point of time.

Additionally, if we are to assume each and every fact to be inaccurate, then new innovations
cannot be made and we cannot think out of the box. For instance, if we assume Pythagoras
theorem to be false or the invention of "pi" value in maths to be baseless without any reason,
then we cannot delve into deep researches in the field of Mathematics. If we are to ignore the
facts, then we will end up establishing the same facts over and again. For instance, if we assume
some fact to be false and ignore it, then there will no way for researchers all over the world to
know that such a fact has been tried to be established in the real world. From this, we infer that,
for sharing of data and discussion regarding attempts made to prove a fact, we have to assume
the facts to be true if they are proven to be valid; Otherwise, we will repent a lot in the future.
For instance, the famous Mathematician Ramanujam's theorems were left unproven and hence
the facts that he established were not taken to be true. Years after his death, many researches
have started realizing the wonderful results that this great Mathematician has strived to find in
the past. Hence, they are now repenting for not having looked into the proof of these theorems
for a so long time.

Above all, facts are considered to be way above myths. Myths are often related to supernatural
beliefs and cannot be assumed to be true; Whereas the information in facts have been
experimented thoroughly and given as results to us. This implies that we must adhere to its
results. For instance, "Women and Children must not step out of the house during eclipses" is a
myth; But "The solar eclipse occurs when sun covers the earth and there is a chance of sun
emitting harmful radiations" is a fact. Hence facts are backed up by scientific evidence and they
can be believed in most of the cases.

Thus, facts established long back have continued to exhibit their relevance till date. For instance,
Moore's law states "Processor size decreases as time goes on, and the speed correspondingly
increases"; this was stated way back in 1960s. But even today, we find new types of computers
coming up in the market; the latest one being palmtop whose processor size fits into the size of
our hand and the speed is 10 times that of a normal computer! Therefore, time and again, relying
on facts has produced the best possible results. Though, occasionally the facts can be proved to
be false, but they will take centuries to do so. Of course, as time moves on, change is inevitable.
This leads to the conclusion that the facts that were once true, need not be true after a long time.
In a nutshell, we must adhere to the facts that exist as of today.
Claim: Nations should suspend government funding for the arts when significant numbers of
their citizens are hungry or unemployed.
Reason: It is inappropriate — and, perhaps, even cruel — to use public resources to fund the arts
when people's basic needs are not being met.
Write a response in which you discuss the extent to which you agree or disagree with the claim
and the reason on which that claim is based.

One of the most important responsibilities of every government is paying utmost attention to it
people’s demands so it should devote the majority of budget on people’s basic needs. In some
societies that the amount of budget is very low, the government should make an effort to provide
its people with general welfare instead of spending public resources to fund the arts.

Although the arts play important role to enrich people’s culture and provide them with golden
opportunity to express their fleeing, their funding should not be on top priority in some countries
that suffered from economical problems. For instance, in African countries that the most
proportion of people are very poor and hungry, if their governments devote their budget on
rearing artists and building cinema and house opera instead of building reasonable apartments for
their citizens, little by little the governments will be outraged. Because the government is
expected to take people’s basic demands into account and rescue its people from bad
circumstances.

In addition, the government cannot make profits by funding arts at these kinds of countries,
because people who are hungry and unemployed prefer spending their time on getting job to
doing luxury entertainments like going to opera houses. Therefore, paying attention to the arts
and consider them at first level of nation’s goals at poor countries is inappropriate because it is
equal to waste time and cost.

Furthermore, there is a wide variety of witness that proves big art projects that were implemented
at poor countries failed and even lead to revolution among people. For example, the king of Iran
spent the most percentage of the country budget on rebuilding one of historical places of Iran
when people suffered from economical problems and paucity. He did not pay attention to
people’s basic needs and tried to use public resources on holding big ceremony which was
related on Iran’s history. Many Iranian people who were very poor and could not meet their
rudimentary needs fight against their kings and made revolution in order to suspending
government funding. Because they thought their king’s funding was injustice when they had to
struggle with different kinds of economical problems.

In conclusion, taking the arts into consideration as a significant factor in order to devote budget
depends on nation’s circumstances. When people would not be able to make a standard living
and have to involve with economical problems, their government should consider general
welfare of all people and spend the majority of budget on providing people’s demands.

Claim: Many problems of modern society cannot be solved by laws and the legal system.
Reason: Laws cannot change what is in people's hearts or minds.
Write a response in which you discuss the extent to which you agree or disagree with the claim
and the reason on which that claim is based.

Laws are the rules which are formulated to bring peace and order in the society. In the absence of
laws and legal system, the society must be chaotic and orderless. Laws could influence the
people's immoral behaviour if implemented in an appropriate manner. I do not concur to
statement that problems of mordern society cannot be solved by law and legal system.

Everybody in the society has his own will and has something going in their minds. Let us say, a
person had a excerbated tussle with someone. In the moment, that person might think of killing
the person. However, he stops and recognizes that this is against laws. Hence, if it were not for
the laws and legal system, everybody would have carried guns and arms; and it would be chaotic.
Also, since the advent of many First Person Shooting games and their easy accessibility to the
children have make children notorious and more violent. Its the laws, which have make them
calm. Otherwise, there would have been spur in the case of young children attacking the schools
with guns like the one happen recently in America.

Similar is the case in businesses, with globalization, there are many multi-national companies.
The Laws and order anre in place to regulate there proper working. In absence of proper
regulations, the companies might not have been able to hold business in different geographies as
they might be marred by many possible corrupt and illegal practices like embezzlment of funds,
copyright infringement etc. Thus, proper laws are necessary for the growth of businesses.

One may content that there are many cases where laws have been broke by many people. Thus,
laws cannot change people's hearts or mind. However, such examples are quite few compared to
entire population. Most of the people abide to the laws and abstain themselves from the activities
hich are immoral and illegal. I also feel, that the government should make sure that the people
breaking the laws are punished and appropriately and shold set an example which would
demotivate people to follow the footsteps of such people.
In conclusion, I am of the opinion that government should impervious laws and legal system
which should encourage people to lead a moral life and demotivate them bring the devils inside
them out. If laws are implemented properly, the power to influence people's hearts and minds.

. Educators should take students' interests into account when planning the content of the courses
they teach.
Write a response in which you discuss the extent to which you agree or disagree with the
recommendation and explain your reasoning for the position you take. In developing and
supporting your position, describe specific circumstances in which adopting the
recommendation would or would not be advantageous and explain how these examples shape
your position.

Laws are the rules which are formulated to bring peace and order in the society. In the absence of
laws and legal system, the society must be chaotic and orderless. Laws could influence the
people's immoral behaviour if implemented in an appropriate manner. I do not concur to
statement that problems of mordern society cannot be solved by law and legal system.

Everybody in the society has his own will and has something going in their minds. Let us say, a
person had a excerbated tussle with someone. In the moment, that person might think of killing
the person. However, he stops and recognizes that this is against laws. Hence, if it were not for
the laws and legal system, everybody would have carried guns and arms; and it would be chaotic.
Also, since the advent of many First Person Shooting games and their easy accessibility to the
children have make children notorious and more violent. Its the laws, which have make them
calm. Otherwise, there would have been spur in the case of young children attacking the schools
with guns like the one happen recently in America.

Similar is the case in businesses, with globalization, there are many multi-national companies.
The Laws and order anre in place to regulate there proper working. In absence of proper
regulations, the companies might not have been able to hold business in different geographies as
they might be marred by many possible corrupt and illegal practices like embezzlment of funds,
copyright infringement etc. Thus, proper laws are necessary for the growth of businesses.

One may content that there are many cases where laws have been broke by many people. Thus,
laws cannot change people's hearts or mind. However, such examples are quite few compared to
entire population. Most of the people abide to the laws and abstain themselves from the activities
hich are immoral and illegal. I also feel, that the government should make sure that the people
breaking the laws are punished and appropriately and shold set an example which would
demotivate people to follow the footsteps of such people.

In conclusion, I am of the opinion that government should impervious laws and legal system
which should encourage people to lead a moral life and demotivate them bring the devils inside
them out. If laws are implemented properly, the power to influence people's hearts and minds.

The primary goal of technological advancement should be to increase people's efficiency so that
they have more leisure time.
Write a response in which you discuss the extent to which you agree or disagree with the
statement and explain your reasoning for the position you take. In developing and supporting
your position, you should consider ways in which the statement might or might not hold true and
explain how these considerations shape your position.

Technological advancements exist in this world, in order to ease our work and aid us in all our
endeavors. But this does not imply that, the sole aim of techological advancement is to ensure
leisure time for people. Because, in today's competitive world, people hardly find any leisure
time regarded as 'leisure' for enjoyment; because, they get caught up in some work or the other,
no one really enjoys leisure time. Thus, technological development acts as a substitute for our
potential, rather than improving our intellectual potential.

First of all, technological devices prevents us from doing monotonous work. For instance, in
today's modernized world, a day without washing machines, mixies and wet grinders will not be
even thinkable for women. This is because, these devices have gone a long way, in aiding the
household women's day-to-day work. As a result, women tend to concentrate in their
professional work or spend more time with their wards. Definitely, they do not remain idle
getting ample amount of leisure time. Had there been no such devices, then women will not find
any time to do their own personal work. Thus, technical advancements averts us from doing
useless work and converges our attention in performing useful work.

Additionally, people's efficiency is not increased by the technological advancements. Instead,


they somehow decrease people's potential and their cogitative abilities. Consequently, the
concept of 'thinking out of the box' is completely forgotten in the process of using technical
devices. For instance, today's children tend to use calculators, even when encountered with a
petty calculation. In days of yore, people were able to do mind calculations and it helped a lot in
their professional life. But today's kids lack this ability and hence the scientific device -
calculator - has marred their actuarial ability. Also, the trend of letter-writing has almost been
completely eroded with the advent of e-mails. This advancement, instead of enhancing our
writing skills, has indeed gone a long way to reduce our craving for vocabulary and the art of
writing structured sentences. Because, many e-mail editors have spell check features, people
rarely give importance to spellings and the words they use. Therefore, technological
advancements have marred our skills to an extent.

Furthermore, people do not always get leisure time, to pursue their hobbies. Indeed, 'hobby' is a
word, which has almost lost significance, in today's competitive world. Since everyone are
money-minded, they have urge to reach the pinnacle of success, in the shortest time possible.
Hence, people try arduously, to make as much money as possible. Therefore, they do not even
spare the leisure time. Scientists all over the world compete with each other, to make people's
live still more easier. This zeal for inventing new things will go on and on, because there is no
limit to which people can be comforted. The standing example for this fact, is that of robotics
which is seeing a huge leap in terms of development, every year. Even, there has arisen a new
field, through which we can get things done, even when we are thinking of them - such a
promising field is 'ubiquitous computing' which is still in its incipient stages. On the whole,
technological developments do not stop us and make us relax in the luxuries they provide;
instead they goad us, to make use of the available time as much as possible.

Some may argue that technological developments, increase people's efficiency. However, no
machine can equal man's knowledge and "machines are man's slaves; not his masters".
Therefore, only the machines have to come closer to the efficiency level of man. Also, the leisure
time, provided by the advancements is only superficial. In reality, people get to do alternative
work, which will make their life beneficial.

Thus, technological advancements prevents us from doing tedious work and eases our lives.
They urge us to find still more exciting inventions. But, they do affect the creative skills of
people and affect their thinking ability. However, technology has multiple goals to satisfy, rather
than primarily increasing people's efficiency or providing leisure time. As time goes on, the
world will witness new inventions; there is absolutely no limit to which, the potential of humans
can be utilized to invent new devices. Therefore, technical advancements will continue to keep
people busy, inspite of they easing the people's lives.

Educators should base their assessment of students' learning not on students' grasp of facts but on
the ability to explain the ideas, trends, and concepts that those facts illustrate.
Write a response in which you discuss the extent to which you agree or disagree with the
recommendation and explain your reasoning for the position you take. In developing and
supporting your position, describe specific circumstances in which adopting the
recommendation would or would not be advantageous and explain how these examples shape
your position.

Students are the pillars of tomorrow's world. They ought to be properly guided in order to take a
country to greater heights. Bringing the students to the right path is the due responsibility of the
teachers. A student spends most of his days at school and college. Hence it is of utmost
importance that educators adopt the right method of assessing students' learning. The assessment
must be based on how well the student is able to apply the facts learned to the real life situations.

First of all, many students today tend to mug up the facts and figures. Henceforth, they find an
easy way to ace the exams with top grades. However, the rote method of learning, does not
reflects one's real understanding. Many of the students, just mug up facts by forming mnemonics
and other funny techniques. They never realize the applications of whatever they are learning.
This is evident in the transition stage from school to college. In schools, students are given
prescribed textbooks and they expect a set of questions to appear in their exams. But, when these
students enter the college, the situation turns upside down. Only a thorough understanding of the
subjects can help students ace their exams with outstanding grades in the college level.

Secondly, the grasp of ideas alone does not serve any purpose to one's life. The reason for many
students blaming the education system of their country is that, they really do not know how to
apply what they have learned to the real world. This conception can be rectified only if students
are in a position to explain the concept and reason behind the facts they have learned. For
example, while learning programming languages, students are often told to memorize the syntax.
Instead of memorizing blindly, they must be taught about the background process: the phases of
compilation of a program and why the syntax has to be given in a specified format. This will
create interest in learning new things and the students will also benefit out of application-
oriented critical reasoning.
In today's world, students are just learning for namesake purpose and earning a degree to land up
in a job. Hardly they realize the beauty of their chosen Majors. Even people who choose the most
coveted Computer Science Engineering, end up as a Consultant in some financial firm.
Therefore, to pull students to do research work, they need to realize the various advantages and
happiness involved in performing a research and inventing a new concept. This will be feasible,
only if the students have a good basic background of the working of already established facts or
ideas. In good olden days, scientists did not enroll themselves in a Ph.D program to invent a new
device. Instead, they found new concepts accidentally, when they were deeply indulged in
performing their scientific experiments. Only if the theory(facts) are backed up with lab
work(illustration of facts), will a student get interested in thinking out of the box and opting for
research.

Above all, some may quote that understanding the ideas, trends and concepts behind each and
every fact can take a lot of time. On the other hand, this time spent in understanding does not go
in vain. They strengthen the foundation and pave way for a bright future. Facts which are learned
with complete understanding through live experiment results, stay in one's mind forever. Hence
they foster creativity and innovation in young minds. Thus, the claim of time being wasted does
not hold good.

Therefore, educators must base their evaluation of students' learning on the understanding of
facts for the following reasons: avoiding rote memorization and encouraging in-depth
understanding, getting interested knowing the real-life applications of the facts concerned,
indulging in hard core research work for the betterment of society. If this form of teaching
methodology is followed in all the countries, then there will be millions of Doctoral Candidates
presenting their thesis every year!

Unfortunately, in contemporary society, creating an appealing image has become more important
than the reality or truth behind that image.
Write a response in which you discuss the extent to which you agree or disagree with the
statement and explain your reasoning for the position you take. In developing and supporting
your position, you should consider ways in which the statement might or might not hold true and
explain how these considerations shape your position.

In a society where communication through social media has become a valuable skill, it is not
remarkable that social profiles get a high social status. The internet however, does not see faces,
but only written text, that can be thought about for days. In this way images can be created that
not necessarily represent the person behind the computerscreen typing it. Because likes and
followers are immediatly linked to fame and money, creating an appealing image has become
more important than the reality or truth behind that image.

The first thing companies do when they receive a job application is google that persons name,
look at his/her facebook, twitter, instagram and other social profiles. This has become a routine
that replaces the first impression one can make by entering the room for a job interview. The
person about to hire you already knows more about you than you could possibly think. If you
wear tattoos, piercings or long hair as a man, you wouldn't be the first one who didn't get hired
because of superficial prejudices about these, before you even get the chance to show them your
true self. Therefore it is important you keep a positive image about yourself on the internet.

This of course implies that one should take all social profiles with a grain of salt. But it gives
people the opportunity again to show you what they really are made of when you meet them face
to face. In the end, one cannot disguise him/herself from who he/she really is. Therefore the
creation of appealing images might not be so bad after all. This way we will always start a
conversation with a positive feeling about that person, allowing us to make our own judgement.

Despite this own judging, one should not forget that the human mind is easily fooled and that
right now it has become more of a quest to create the most appealing image, only deteriorating
the situation. People want to appear more and more appealing to outclass their relatives and
friends. This sometimes results in Facebook-wars and cyber-bullying. The internet has become a
world on its own, and people can be bullied in real life, but be almighty in their cyber-world. A
healthy mix of both worlds would be best for all of us, but the real world is where the
interactions will need to happen eventually.

In sum, we can say that to a certain extent, creating appealing images has its good points, since it
givesstrangers the benefit of the doubt when you meet them, allowing
you to form your own judgement. However, this social hyping is taking
immense proportions which creates new, more severe problems our
society will have to face. Therefore one should read every social profile
with a little doubt, postponing judgement untill a meeting with that person
in real life happens. If you agree with this opinion, please "like" this
message.
The effectiveness of a country's leaders is best measured by examining the
well-being of that country's citizens.
Write a response in which you discuss the extent to which you agree or
disagree with the claim. In developing and supporting your position, be sure to
address the most compelling reasons and/or examples that could be used to
challenge your position.
There is a disputation whether the well-being of a country is the best
ruler to measure the effectiveness of the country's leaders. I think the
question heavily depends on the meaning of "effectiveness": what does
it mean of effectiveness of leaders? I think this means leaders' ability on
governing their countries. Thus I will focus on the status of the country
mainly, and I will evaluate leaders be reflecting this status. I will present
two types of countries, one pursues strong country but seems not care
their citizen's well-being, and the other want to catch well-being but not
effectively. Therefore, maintaining well-being of their citizen is not a best
way to measure the effectiveness of leaders.
Firstly, there are leaders who pursues "strong country"; there are various
aspects of strongness: the most simplest one is getting strong military
and force, and less simplest one is culminating their own culture. For
example, various presidents of the United States during Cold War
develops their scientific techniques and cultural influences, but
surprisingly, none of them developed systems for social welfare:
although many citizens enjoys financial developments, some poor
citizens were not.
There might be an objection: getting a strong military and well-developed
culture is related to earning money, and it also related to the well-being
of citizens. However, both relations are not exactly hold: consider Nazi
Germany as an example. Adolf Hitler depicted a vigorous Germanic
country, so he trained military and eventually led to a war to enlarge their
territory. Does it improves German's well-being? No, lots of their citizens
were exploited by the war and suppressed by the Nazi government.
Secondly, we know that various countries which pursue the well-being of
their people, like Scandinavian and West European countries. Various
communist countries, like Cuba, is also an example. Let me focus on
communist countries: their motto is (in most cases, "was") to provide
welfare services for their people. Unfortunately, their object is ill-
performed: their system hampers productivity of factories and farms, and
It harms their economy and affects their culture in bad ways. Even
though people of the country are happy for free health cares and
education, their country seems not under effective rule.
All parents should be required to volunteer time to their children's schools.
Write a response in which you discuss the extent to which you agree or
disagree with the claim. In developing and supporting your position, be sure to
address the most compelling reasons and/or examples that could be used to
challenge your position.

It is wildly accepted that parenting is one of the demanding


responsibilities and parents are expected to contribute to the
development of children’s education as much as they can. Unfortunately,
some specialists report that the majority of the parents do not actively
and effectively dedicate time for the educational aspects of their children.
As a result, I advocate the statement that all parents should be required
to volunteer time to their children's schools because of the reasons I
would like to elaborate below.
Firstly, any volunteer involvement of parents in their children’s schools
provide several advantages for them. It may serve as a platform for their
social development as they expand their relationships with the complex
of school’s teachers and parents of other children. Furthermore, it aids
them to have a better picture of the educational environment in which
their children is expected to study. Besides the benefits the parents
obtain from the contribution to the school, they can share their own
expertise, knowledge and experience they posses in a specific area. For
instance, a parent who is a specialist in a particular sport can devote part
of his or her time to train students for better performance.
On the other hand, the more parents are engaged with their children’s
school, the more the children will be beneficiaries. As children observe
the cooperation of their parents with their schools, they knowingly or
unknowingly are encouraged and their social development may be
positively influenced. In addition, such volunteer time most effectively
strengthen the affinity bond between children and parents because
children are exposed to a cooperative atmosphere. Above all, since
children would be further overseen by their parents, they can perform
better on their assignments and motivated to move forward.
In perspective of school officials, such volunteer contribution of parents
may be remarkably valued as they feel that their endeavors toward
educating children are supported efficiently with the benevolent attempts
of children’s parents. Clearly, they will be encouraged to make further
attempts to provide students with superior services and satisfy the
children’s parents expectations.
In conclusion, volunteer time parents dedicate to children's schools may
produce various advantages not only to children, but also to their parents
and even to schools.

A nation should require all of its students to study the same national
curriculum until they enter college.
Write a response in which you discuss the extent to which you agree or
disagree with the claim. In developing and supporting your position, be sure to
address the most compelling reasons and/or examples that could be used to
challenge your position.

George W. Bush's adminstration implemented the "No Child Left Behind Act" in the
2000s with the high goals of bringing a standardized quality education to all children in
America. In a speech aired on television, he said that he would make sure that all
schools nationwide would have set goals to achieve, and that standardized tests would
measure the progress of each school. "How can we know how well a school is doing if
we don't have a way of measuring it?" he asked reporters. The "No Child Left Behind
Act" and many other attempts at a national standardized curriculum have the goals of
bringing an equal education for all, an education that can be measured and
standardized to improve problem areas. However, though being able to quantatively
measure a school's performance is important, imposing a national curriculum on all
students is ultimately prone to a disharmonious education for the individual student.
Students study in schools that are engrained in a community. A student living in New
York is surrounded by city lights, pollution, and international cultures, while a student
living in rural Texas has wheatfields and long stretches of empty road as neighbors. The
students living in these different areas have to deal with different issues. The student
from New York might have to learn more about maintaining an environmentally healthy
planet by reducing pollution and trash, while the student from Texas might have to focus
more on weather patterns that affect their daily lives. A national curriculum, however,
does not take into account the distinct necessities of the student's surroundings. In
pursuing the standards of a national curriculum, teachers have less time to spend on
areas that are important for children of a particular area.
Teachers must have this kind of leniency in order to bring tailor-made lessons that
match their students' needs. In certain regions of the country, teachers have their every
minute planned so that every teacher will teach a particular concept in a particular way
at the same time. Not only does this impede teachers of their creativity, this also creates
an environment where teachers acts as cogs in a machinated system where they have
little say on the direction of their classes. Perhaps a certain class has trouble with
adding, more so than other clases, and a teacher can see that this class will need an
extra two weeks for the subject. Looking at the course schedule, however, he might
forgo this extra time because he has to keep up with the curriculum.
Teachers cannot create tailor-made education experiences for their students when they
have twenty to thirty students in their classrooms. It is possibly true that students can
receive individualized education only in one-on-one or small class settings. However,
when a teacher no longer follows a lesson plan made for the twenty to thirty students
she knows, and instead must follow a lesson plan tailored for a generic student
averaged from the many millions of students across the country, she loses much of her
effectiveness as a teacher.  

Educational institutions should actively encourage their students to choose


fields of study in which jobs are plentiful.
Write a response in which you discuss your views on the policy and explain
your reasoning for the position you take. In developing and supporting your
position, you should consider the possible consequences of implementing the
policy and explain how these consequences shape your position.

Should educational institutions actively encourage students to choose


fields in which a plethora of jobs exists? There is no simple answer on
this question owing to the fact that implementation of the policy may
have positive and negative aftermaths as well. Although I can see the
benefits of the policy, as far as I am concerned the drawbacks of its
implementation outweigh them. Therefore I disagree with the issue.
To begin with, why do we go to study at University or College? Why do
we spend comprehensible amount of money on acquiring a degree?
Perhaps, because we want to acquire a profession which will allows us
to earn more money than we spend. From this viewpoint, the proposed
idea is right to the point because for alumnus to find no job is a disaster,
in particular, if he or she has a student loan. The implementation of the
policy will guarantee that a student who invests his or her money and
time will return them with profit; moreover, society as a whole will win as
well due to the fact that social needs will be satisfied in time; educational
institutions will steer their students to choose the most needed
specializations and fields and thus the shortage of employs will be
negligible. So, may we assert that the policy ought to be implemented?
Unfortunately, the answer on this question is "no" because the fulfillment
of the policy has some grave disadvantages and probably the worst one
of them is that it pays no attention to students' inclinations or/and
interests. Each student has her or his dreams and desires which may be
broken by the policy in case they do not match with the current situation.
For example, a person wants to be a psychologist, perhaps, one has
talent and strong desire to pursue his or her dream. However, if there is
not job in the field, the university may convince the student to change
one's mind and become, for instance, a programmer. In other words, if
we pay the high priority to availability of jobs, person's abilities and
talents may be neglected. In this case, we will have an unhappy person
who, perhaps, is an average programmer but might have been a great
psychologist.
The second disadvantage is connected with the fact that it is literally
impossible to precisely predict which jobs and professions will be in
demand and which ones disappear. For example, in the 1980s few
people might predict that a demand in software engineers will skyrocket
soon; moreover, today such professions as economist or accouter are in
great demand but, perhaps, those professions may become extinct if AI
is created. In other words, we cannot be sure about future demand and
thus we have a great risk to mislead people into wrong decision.
In conclusion, the idea to encourage students to choose fields in which
jobs are plentiful seems to be attractive at the first glance, in particular,
because it supposedly guarantees future employment; at the same time,
the precise prediction of the demand is impossible and therefore the
biggest advantage is weaken. Furthermore, the policy does not take into
account people's abilities and interests and thus its realization is highly
questionable.

People's behavior is largely determined by forces not of their own making.


Write a response in which you discuss the extent to which you agree or
disagree with the claim. In developing and supporting your position, be sure to
address the most compelling reasons and/or examples that could be used to
challenge your position.

People's behavior largely depends on the one's attitude at that instant.


Behavior of a person varies throughout the life not constant. All the
events, happenings in their individual life influences behavior. Behavior
is not a job which is too acted in a same way daily. Behavior of a person
mainly depends on the attitude of a person. I agree that people’s
behavior largely determined by their own making. Mainly behavior of a
person varies due to following reasons like consequences faced in life,
difficulties they face at that instant, state of mind, among different
people, in various places,at that particular instant
Firstly we should see behavior of a person depends on the several
consequences in their life till now. If the person has overcomes all
difficulties in his life He will be optimistic and cool in his behavior with
any person. Attitude of a person major factor which changes behavior of
a person by their own making. People largely rely on the environment
around them or situations to act upon own making. People in any
situation or at any instant of life should be optimistic .I don’t mean that if
someone has dashed your car you should be optimistic, but act on
particular situation.
Secondly coming to the state of mind of a person, at that instant
changes according to that of another person in conversation or involved
in activity. The state of mind of differs for a educated person, business
person, employed person, higher officials in organization. Each of these
persons change state of their mind instantly without prior notice. You can
also observer that state of mind of particular person depends on the
other person whether lower official than him or not.
Lastly coming to the behavior of a person at different places. For
example if a person meeting a girl near theater or park will obviously
different than meeting at college or library. Employee meeting boss in
pub differs that from meeting that in office. So location prior to person .
I conclude by saying that people's behavior differs by their own making
at different instants of life. So behavior of person influences others like
friends, relatives ...

Although innovations such as video, computers, and the Internet seem to offer
schools improved methods for instructing students, these technologies all too
often distract from real learning.
Write a response in which you discuss the extent to which you agree or
disagree with the statement and explain your reasoning for the position you
take. In developing and supporting your position, you should consider ways in
which the statement might or might not hold true and explain how these
considerations shape your position.

The speaker asserts that innovations such as videos, computers, and


the Internet too often distract from "real" learning in the dassroom. I
strongly agree that these tools can be counterproductive in some
instances, and ineffectual for certain types of learning. Nevertheless, the
speaker's assertion places too little value on the ways in which these
innovations can facilitate the learning process.
In several respects, I find the statement compelling. First of all, in my
observation and experience, computers and videos are misused most
often for education when teachers rely on them as surrogates, or baby-
sitters. Teachers must use the time during which students are watching
videos or are at their computer stations productively--helping other
students, preparing lesson plans, and so forth. Otherwise, these tools
can indeed impede the learning process.
Secondly, passive viewing of videos or of Web pages is no indication
that any significant learning is taking place. Thus teachers must carefully
select Internet resources that provide a true interactive learning
experience, or are highly informative otherwise. And, in selecting videos
teachers must be sure to follow up with lively class discussions.
Otherwise, the comparatively passive nature of these media can render
them ineffectual in the learning process.
Thirdly, some types of learning occur best during face-to-face
encounters between teacher and student, and between students. Only
by way of a live encounter can a language teacher recognize and
immediately correct subtle problems in pronunciauon and inflection. And,
there is no suitable substitute for a live encounter when it comes to
teaching techniques in painting, sculpture, music performance, and
acting. Moreover, certain types of learning are facilitated when students
interact as a group. Many grade school teachers, for example, find that
reading together aloud is the most effective way for students to learn this
skill.
Fourth, with technology-based learning tools, especially computers and
the Intemet, learning how to use the technology can rob the teacher of
valuable time that could be spent accomplishing the teacher's ultimate
educational objectives. Besides, any technology-based learning tool
carries the risk of technical problems. Students whose teachers fail to
plan for productive use of unexpected down-time can lose opportunities
for real learning.
Finally, we must not overlook the non-quantifiable benefit that personal
attention can afford. A human teacher can provide meaningful personal
encouragement and support, and can identify and help to solve a
student's social or psychological problems that might be impeding the
learning process. No video, computer program, or Web site can begin to
serve these invaluable functions.
Acknowledging the many ways that technological innovations can
impede "real" learning, these innovations nevertheless can facilitate
"real" learning, if employed judicially and for appropriate purposes.
Specifically, when it comes to learning rote facts and figures, personal
interaction with a teacher is unnecessary, and can even result in fatigue
and burnout for the teacher. Computers are an ideal tool for the sorts of
learning that occur only through repetition--typing skills, basic
arithmetical calculations, and so forth. Computers also make possible
visual effects that aid uniquely in the learning of spatial concepts. Finally,
computers, videos and the Internet are ideal for imparting basic text-
book information to students, thereby freeing up the teacher's time to
give students individualized attention.
In sum, computers and videos can indeed distract from learning--when
teachers misuse them as substitutes for personal attention, or when the
technology itself becomes the focus of attention. Nevertheless, if
judicially used as primers, as supplements, and where repetition and
rote learning are appropriate, these tools can serve to liberate teachers
to focus on individual needs of students--needs that only "real" teachers
can recognize and meet.

The best ideas arise from a passionate interest in commonplace things.


Write a response in which you discuss the extent to which you agree or
disagree with the statement and explain your reasoning for the position you
take. In developing and supporting your position, you should consider ways in
which the statement might or might not hold true and explain how these
considerations shape your position.
Are commonplace things too banal to get interested in? Or should
human beings rather pay more attention to them? Would the latter
choice be source of no ideas and discoveries? The issue, even though
simple in appearance, is in fact, pretty thorny.
Many people hold that what is common is most of the time out of interest
and perhaps sometimes repetitive. However, I personally think the
opposite : even though the matter is very field-specific and varies from a
domain of interest to another, I would say that on the whole, the banal
appearance of commonplace assertions should not let us neglect their
importance in some cases.
First if all, it should be borne in mind that in some domains, it is precisely
the every day scences and phenomenons that trigger the action and/or
spark a new idea. Typicall, in art, for instance, the artist finds his/her idea
generally in a common situation. Take the example of a painter. He/She
does not look for any weird or unreal scene to portray, but rather, tries to
explore the uncommon that lies behind the common. Therefore, what we
consider as being commonplace, is only so in appearance. A lot of
situations are qualified this way, only because we have lived them
several times, so at first glance we don't find anything interesting in
them. However, I think that it's important not to get trapped by this
appearnace and look for new things.
Of course, there are also opponents to this way of thinking. Most people
would probably assert that banality does not let us discover any new
thing. On the contrary, what strikes us most are more often than not,
new and unexpected events. An example to illustarte this argument
would be that of a scientific research. The scientific method begins, by
defintion, by observation, before leading to hypothesis and then
verification. Most often, the observation stage relies on a particular and,
especially unexpected or singular phenomenon that draws the attention
of the scientist.
Therefore, one might be tempted by claiming that actually, commonplace
is already viewed and reviewed and there is no interest to spend time on
it again since it contains no new ideas.
Nevertheless, to my viewpoint, event though in some fields of study or
research, the banality seems unintersting, it can in fact, be a source of
new ideas, by a mecanism of judgement. To make my point clearer, I will
take the example of the discovery of the spherical shape of Earth. If
people never took interest in what was commonly asserted and never
thought of criticizing it, they would never find out that in fact they were
mistaken. Thus, it is, to my mind, necessary to take a look back at what
we assume as being true out of commonplace, just in order to get new
ideas about why it might in fact not be as true as it seems to be.
In a nutshell, even though the subject can be viewed from different
angles, since it varies a lot from a field of activty to another, I would
claim that in general, the commonplace hides a lot of interesting and
undiscovered things that would spark great ideas.

To be an effective leader, a public official must maintain the highest ethical


and moral standards.
Write a response in which you discuss the extent to which you agree or
disagree with the claim. In developing and supporting your position, be sure to
address the most compelling reasons and/or examples that could be used to
challenge your position.

Although the efficiency and strategy to reach a goal are important for
being effective, the most crucial element to be an effective leader is to
maintain the the highest ethical and moral standards, to be upright, and
to sustain integrity. It can't be emphasized more that the moral quality is
the most essential part of a great leader.
First, if the public official maintain good ethical standards, he will benefit
his people. Since one openly accepted standard is thinking for others,
keeping it in mind, the official in the government will deliberate or
contemplate every policy, struggle for best solution that will make a
compromise which each party can agree it with happiness, and establish
the most important infrastructure for the majoity. There will be a desire, a
want can't depressed that encourage him to serve the population until
the end of his responsbility, as the crucial requirement for an effective
leader.
Furthermore, he will not make any wicked attempt to do good to himself
rather than the interest for the public. In almost every standard, principle,
or even rule, there is a common ethical thoughts that people shall not
cheat, as the trustworthy is the most valuable wealth people can have.
Therefor, if the official follow principles strictly, he should not deceive his
people, leaving things such as embezzlement impossible. Effectiveness
will be refulgent without the annoyance for all these evildoing.
In a addition, with the shine of justice, people will willing to follow him,
which is the peerless essentiality for an effective leader. The majority will
not be followers for celebrities in doubt, especially for morality. In fact, in
such a worldly world, a official adhering to highest principles will
probably be a maverick, a brilliant star encourage people to follow. Some
enthusiasts may even volunteer to advocate for the leader's purpose. All
these are dreams for a public figure, as an effective leader.
In the end, there is liitle doubt that a public official attaching the highest
ethical and moral standards will be effective. Let's dream for a candidate
with such a high morality in his disposition, as a dream for a better
sociaty.

Claim: Imagination is a more valuable asset than experience.


Reason: People who lack experience are free to imagine what is possible
without the constraints of established habits and attitudes.
Write a response in which you discuss the extent to which you agree or
disagree with the claim and the reason on which that claim is based.
The speaker asserts that imagination is a more valuable asset than
experience. In my view the speaker unfairly generalizes. As we exam
different fields of inquiry, we shall see that in some area, especially in
arts, imagination often have a more significant role than other characters
such as experience. However, in some other fields, particularly in
physics and math, scientific progress relies on the experience of those
brilliant scientists. In still other fields, such as business and engineering,
both imagination and experience are required for excellent
achievements. Therefore, I believe that the view of the speaker is
inaccurate.
An artist would need his or her talent of imagination to create
distinguished artwork. In the process of generating artistic works, artists
could not come to a great breakthrough without imagination beyond the
worldly life. It is possible that abundant experience could be fundamental
to drawing a picture or sculpturing. But if an artist could not imagine
about art itself, even though he or she has abundant experience, this so-
called artist is nothing more than a copier, would not have enough ability
to create his or her own miracle. Only when a skilled artist has sufficient
imagination, great works of art could be finished. Therefore, I believe in
the art world, imagination is more valuable than experience.
However, on the contrary, in the natural science world of physics and
math, experience would hold a more important status than imagination.
Every scientific deduction must be fully based on careful observation and
precise statistic data. A scientist with ample experiences in research,
even though he or she is not very creative, could apply elaborate
experiments, collect detailed experiment data, infer possible theories
responsively and come to a reasonable conclusion eventually.
Imagination could be helpful in scientific research especially when
research meet its bottleneck, but a scientist who imagines a lot but has
little experience, maybe carry on research with errors and irresponsible
conclusions. In short, I take experience for a superordinate character of
scientists compared with imagination.
In the fields of business and engineering, imagination and experience
are both significant. Without imagination, one could not make difference;
without experience, one could not transform his or her imagination into
reality. Only those people whose imagination and imagination are both
abundant would have possibilities to success in these fields.
In sum, in some fields such as art, imagination is a more valuable asset
than experience, while in some other fields, experience is more
important, or they are equally significant. Obviously people could benefit
from both experience and experience as long as they enter into suitable
fields. Therefore, people should take different attitudes toward these two
traits when facing different situations.

In most professions and academic fields, imagination is more important than


knowledge.
Write a response in which you discuss the extent to which you agree or
disagree with the statement and explain your reasoning for the position you
take. In developing and supporting your position, you should consider ways in
which the statement might or might not hold true and explain how these
considerations shape your position.

Knowledge and imagination have propelled society in all fields - since


the dawn of man. It is through imagination man has invented the wheel
and learned to cook meat with fire - and it is through the learning
process - knowledge has been passed on. Knowledge marks an
accumulative process through which society better itself and it is
imagination that allows individuals to impact society - in a symbiotic
process that nourishes all. Society is composed of a population divided,
albeit artificially, to professions - be proper daily professions or academic
ones - all embarked on a journey that requires both knowledge and
imagination.
Most professions and academic fields hold, by definition. the
accumulation of knowledge that has been reshaped into either practical ,
theoretical and academic forms. For a man to pursue a career, be it any
of the former, he cannot exclude himself from the world, as to presume
one could create all the knowledge of a profession all by himself. True,
as in the case for the lone Indian mathematician who found a
mathematical book and has reinvented numerous mathematical
branches, people could live a life of solitude and create all by
themselves - but even within the academic society, these are the
exceptions - not the common. It is through knowledge that can be learnt,
one can gain the experience of tens, if not hundred of years - if not even
more - and to stand on the frontier of his profession - be the best
detective one can be, the best scientist - or a great humanitarian.
Nonetheless, we must distinguish between proper,dull professions in
which little imagination is required, such as insurance agents - opposed
to the works of policemen, detectives and firemen on one hand - and
scientists or engineers on the other. It is not practical to gain all
knowledge of a profession, nor is it scalable with the ever expanding
knowledge of humanity. Those wishing to explore new realms of physics,
invent new gadgets, explore fields for which previous knowledge is
lacking - must employ deep thought processes, exhibit ingenuity, all the
remarks of the great minds, such as Albert Einstein - it is the works of an
alternate thinker, a cerebral thinker, through the use of the left, artistic
side of our brain, imagination can lead us to realms previous explorers
never dreamed of.
Imagination and knowledge are both distinctive marks of professions -
be it academic or not - and to go as to the extreme of stating which is
more important - is a tedious task. All professions could be thought of
requiring a mixture of the both. From the mundane salesman, who must
show creativity in order to support his sales rate - to the common
scientist - for whom imagination is an impeccable trait - his gained
knowledge of many years of study are not less that mandatory for proper
research. One must ask, what professions require no imagination or
pure knowledge alone - and who, in a sane mind, would be keen to work
or participate in such professions? A profession without the use
imagination can be described as robotic - the same very jobs in
manufacturer lines in which actual robots replace human beings. On the
other hand, the realm of pure imagination, arguably, can be assigned to
children alone - since any profession in imagination is used - the product
of such a process must be manifested - a play, a book or a publication -
even for the sheer share of knowledge.
In sum, most professions use both imagination and knowledge - yet in
any profession - the use of either exists on a continuous spectrum.

To be an effective leader, a public official must maintain the highest ethical


and moral standards.
Write a response in which you discuss the extent to which you agree or
disagree with the statement and explain your reasoning for the position you
take. In developing and supporting your position, you should consider ways in
which the statement might or might not hold true and explain how these
considerations shape your position.

Currently, it is debatable whether an effective leader must maintain the


highest ethical and moral standards. Some people hold a view that a
public official must be ethical and moral since he or she should be the
face of the nation. However, I personally argue that having the highest
moral and ethical standards cannot be a crucial factor that qualifies an
effective leader; his or her achievement should also be the factor that we
should concentrate on.
Granted, in some cases, an effective leader does have highest ethical
and moral standards. Take George Washington for example. As the first
president of the United States, he has been highly lionized as one of the
most effective and greatest leader in the country. As he served his first
term of President, he denied accepting a salary so that he could
maintain his image of selfless leader. Moreover, he did not decided to
serve the third term of presidency so that he could establish a precedent
for the future presidents. Because of his aspects of moral and ethnic,
many American people still praise him as one of the most powerful and
effective president of the United States. Therefore, maintaining the
features of high moral and ethnic standard can be a prerequisite to
become an effective leader.
However, there are some leaders who are regard as an effective leader
despite of their low ethical and moral values. To illustrate, the Watergate
Scandal is a famous political scandal, which President Richard Nixon
involved in. To be re-elected, he and his supporters participated in illicit
activities such as wiretapping, illegal break-ins and campaign fraud.
Although this scandals revealed that President Nixon was not having the
highest moral and ethical standards, he was an effective leader who had
various experience in diplomacy and political position. Due to his wide
array of knowledge in politics, he was able to end the Vietnam war, and
could maintain the amicable relationship with the Soviet Union.
Therefore, achievements of a public official can also be an important
factor to measure the effectiveness of the leader.
In addition, a leader can be a pivotal force to save a country that is mired
in adverse situation, despite of missing the highest ethical and moral
standards. For example, Bill Clinton was involved in a sex scandal with
the White House intern, and he was almost impeached from the
congress. However, he was still considered as an effective leader
because of his remarkable leadership. When President first became a
president, the U.S. was experience an economic downturn. Because the
Cold war was just ended, the unemployment was high, and many U.S.
citizens were strapped for cash. However, with his remarkable
leadership, President Clinton fostered the economic growth, and the
U.S. economy had become stable at an exponential rate. Therefore,
simply having the highest moral and ethical aspects cannot be enough to
determine the effectiveness of a public official.
In conclusion, maintaining the highest ethical and moral standards
should is not an indicator of an effective leader. Other aspects such as
an achievement should be taken into consideration.

Critical judgment of work in any given field has little value unless it comes
from someone who is an expert in that field.
Write a response in which you discuss the extent to which you agree or
disagree with the statement and explain your reasoning for the position you
take. In developing and supporting your position, you should consider ways in
which the statement might or might not hold true and explain how these
considerations shape your position.

The true worth of any work comes out when it is subjected to proper
critique. Only then the person who does the work in a given field gets to
know what are the things he has done right and what he needs to
improve. Such critical judgement is only valuable when it comes from an
expert in that field otherwise it does have little value.
Firstly, with world increasingly moving towards specialization, any work
in order to be judged properly requires experts in those fields. It cannot
be simply judged by a layman, as more often than not his judgement is
likely to be superficial and does not consider the intricacies of work
involved. For example, let us consider the example of Leaning Tower of
Pisa. A lay man can come up with judgement that such a building is just
a mistake committed while doing work on the tower. While a structural
engineer would come out with apt judgement that if it were a mistake, it
would not stand at all for these many years. He would rather marvel and
appreciate at the immense work that has been done to ensure that
building is kept standing even at such an angle.
Secondly, any work in any field has definite scope for improvement as
human endeavor is a march towards perfection. It is only when an expert
comes out with a judgement on the work, the person gets to know where
he needs to improve. For example, let us take traditional classic dance
like Bharatanatyam. When a performer presents his work everyone
would be in awe of his/her performance. It is only when a true exponent
of the dance form sees it and gives a feedback on performers act,
mistakes can be highlighted and possible corrections can be made. Only
those critical judgements count which help people to strive towards
perfection.
On the other hand, there are instances in real world where critical
judgement does not have any effect on success or failure of a particular
work. Let us take simple example of a movie like Fifty shades of Grey,
even though it received largely negative reviews from experts in the
movie field, it was still able to garner significant collections world wide.
Yet, it should be noted that such success does not mean that work is of
significant value. As long as the director and performers in the movie do
not take the criticism seriously, one day or the other they are going to
fail.
In conclusion, critical judgement of any work has true value only when it
done by experts in those fields. Exceptions if any do not help add any
value to the work in achieving perfection over long term.
Some people believe that scientific discoveries have given us a much better
understanding of the world around us. Others believe that science has
revealed to us that the world is infinitely more complex than we ever realized.
Write a response in which you discuss which view more closely aligns with
your own position and explain your reasoning for the position you take. In
developing and supporting your position, you should address both of the
views presented.

There have been debates on whether scientific development gives


human better understanding of this world. Some people claims that
scientific development has revealed to us that the world is far more
intricate than we ever known, while others assert that the world has been
demystified by sciences. As far as I am concerned, I agree with the
former more.
Some may argue that, throughout thousands of years of development,
human has created a mature civilization. From the Stone Age,
generations of horse and buggy to the technology era today, human has
gained a lot of knowledge about this world and even applied the
knowledge to make our lives better. However, is our understanding
about this world totally correct?
Glancing back through the history, one can undoubtedly notice that
many once authoritative scientific theories ended up being overturned by
later discoveries. Take the development of physics as an example.
Newton physics was once thought to be the truth since it explains almost
every physical phenomenon perfectly. Not until Albert Einstein came up
with the theory of relativity which gives these phenomenons a even
better explanation, did human realize that we have not reached the
ultimate comprehension of physics. Therefore, we can hardly assert we
understand the rules of this world.
Moreover, human may never have the ability to know how big the
universe is. In the book The Grand Design, Stephen Hawking, the
prominent modern physicist, proposes that human's understanding of
our environment can be analogous to a goldfish's understanding of its
fishbowl. Indeed, if goldfishes had intelligence, they might be able to
develop a series scientific theories that are unarguably applicable to the
world inside fishbowls. Yet, living out of the fishbowl, we know that what
these goldfishes understand are by no means the true reality. Similarly,
as many scientific observations and theoretical deductions have
suggested that there might be many other universes apart from the one
we are in, many scientists start to doubt if we are really able to
understand this world.
In conclusion, the development of science reveals that the world is far
more complicated than we ever realized. The more we learn, the more
we find out we still do not know.

In any profession — business, politics, education, government — those in


power should step down after five years.
Write a response in which you discuss the extent to which you agree or
disagree with the claim. In developing and supporting your position, be sure to
address the most compelling reasons and/or examples that could be used to
challenge your position.

Those in powers refer to people who have authority in particular field.


Power has something do with influencing others. Power can be of two
types downgrade power enforcing the subordinates and upgrade power
where the subordinates obey the order. In any field politics, business or
education people who are in power have to be changed after few years.
In politics, the authority is given to a leader for few years mainly to bring
new ideas and changes in social policies which benefit the society. Most
of the democratic countries have a rule to elect their President or Prime
Minister every five years which is beneficial for their country .Every
eligible leader gets a fair chance to compete for the power and to bring
changes in society. If a particular leader is given the power for longtime
they may become biased to few policies and also corrupt. More
intimidating fact would be that they may lust for power so much that they
become corrupt and carry out monocratic rule which is dangerous for the
welfare of the society. In our history we have read how autocratic rule
treated people with discrimination and most of their policies were
beneficial for the upper class people and degraded the middle and lower
classes.
Education is important for every child today. It is the field which is
important to have better society tomorrow .It is also necessary that our
children have updated knowledge in science, technology, art, etc. For
the proper child development it is essential have curricular activities.
People who heads educational institution like Counselor, Principal, etc.,
also needs to step down after few years because the new always get
new ideas to experiment and are better updated, also they can learn
from the mistakes made by the predecessors. Education especially to
girl in developing country lags behind. In order attract to girl children to
school the educational head decided to launch a program that every girl
child would be awarded a bicycle if they attended school regularly. As
result more parent s sent their girl children to school.
Our country’s economy lies in business. It is important to have stable
business and the person who heads business at particular area has to
step down after few years. New CEO’s have to be appointed to get new
ideas and who is always creative to establish their business. As the new
generation is approaching the particular head has to know the mind-set
of the present and develop marketing strategy. They should have
acumen about the investment they make to get a better profit.
In the profession of business it is sometimes necessary to have people
in power for long terms. There are many family owned business
companies where the successor has always been successful. Since it is
their business and the reputation the family also matter they are more
careful than appointing a new member. For example business magnate
in India like Mukesh Ambani who is successor of his father’s reliance
company has been head of the company for long time and has
established well. Also, Steve Jobs who brought computer to palms of
every person severed as CEO of Apple Inc., for long time until he was
diagnosed with Cancer.
The American politics, when President Obama took office the country’s
economy was in recession and he made few policies to recuperate the
ailing economy and it is recovering slowly and within his first term he
could not make much difference which he promised. So he is given
another term to make the changes he promised. Sometime the time
constraint lacks people to do great things.
To sum up, many professions like politics, education and business
requires people to head them and those who head them are required to
be changed every few years. Power makes people greed and it is
dangerous to have people in power for longtime.

Requiring university students to take a variety of courses outside their major


fields of study is the best way to ensure that students become truly educated.
Write a response in which you discuss the extent to which you agree or
disagree with the statement and explain your reasoning for the position you
take. In developing and supporting your position, you should consider ways in
which the statement might or might not hold true and explain how these
considerations shape your position.

Learning is a lifetime opportunity and students have to make the best


use of their learning period. Even if a student decides to do a particular
major, his course selection must include a variety of courses, other than
those from his field of study. But, it is implied that, choosing a variety of
courses outside his field, must not hinder his concentration on core
subjects relevant to his field. Instead, a fair distribution of courses, both
from his field and other fields, must be followed for each semester. It is
the responsibility of the universities, to give the students freedom to
select the elective courses of their choice. Consequently, this will
contribute to a full-fledged learning in a university.
First of all, every field in this field is dependent on every other field.
There is no single field, which can work on its own. For instance,
computer science has few of its courses from electronics - which form
the base for understanding computer architecture; for instance, basics of
electronics and digital signal processing are prerequisite for a computer
science major, to delve deep into the interior of computer architecture.
Also, in the workplace, an IT professional cannot expect all of his peers
to be from a computer science background; he has to collaborate with
those from other fields like electronics, electrical engineering etc...
Hence to have a basic understanding of other fields is required for
guaranteed success in our chosen field.
Moreover, taking up courses outside ones field of study kindles one's
interest in other fields. Additionally, it exposes a student to a variety of
courses, from which a student can identify his field of interest. For
instance, a computer science graduate can take up few courses from
economics and management. Consequently, if he happens to be driven
more towards this economics course, then he can decide to an MBA and
become a successful businessman. Thus, course selection can go even
to the extent of diverting one's career path, and assuring success in the
chosen path. Only if a student is exposed to few courses outside his
major, will he know the gamut of career oppurtunities from which he
could branch out, after his graduation.
Furthermore, research is of utmost importance to the development of
any university. A university which produces successful research
scholars, is deemed in the academia as the best university. Living
examples include those universities in the United States such as
Carnegie Mellon, University of California-Berkeley etc... These U.S
universities are special for a single cause - they allow the students to
choose a variety of courses to complete his degree; there is no
restriction on the type of courses to be chosen to complete a degree.
Because of this freedom, students are able to devote more concentration
to their fields of study. This ends up in fruitful results when researches
are being conducted. Research in any field, requires a sound knowledge
of the related fields too. For instance, a research scholar in artificial
intelligence, might require some help from the mechanical engineering
department, in order to validate the mechanical parameters and
construction of the robot. The programming in robots is done by the
computer science majors; but the ultimate working of the robot and its
manufacturing, requires assistance from the fields of mechanical and
electronics engineering. Thus, any research field, requires atleast little
assistance, from the related fields. Hence, taking up courses outside
one's field, will go a long way in researches to be conducted in the long
run.
Taking up additional courses, has one another advantage: it enables a
student to do part-time job; for instance, if an engineering graduate is
damn interested in English literature, he can take up a course in English
literature. Ultimately he can turn out to be an engineer cum story-writer.
Thus, he has a possibility of always keeping him busy. Hence, additional
courses, can even contribute to the increased earning potential of an
individual.
Some may state that taking up courses outside one's field, deviates his
attention. But, this is possible, only if the courses are very much
unrelated to a chosen field and they are large in number. Courses, if
taken outside one's major, must be limited in number and they must not
be completely unrelated to one's major. They have to possess some
motive of helping an individual, in reaching his desired goal. Also, the
universities must be in a position, to provide experienced staff members,
to handle these courses, kindling interest in the students. Except for
these conditions, these additional courses, do not deviate a student from
his chosen major.
Thus, choosing a multitude of courses, outside one's field of study offers
various advantages like: flexibility in choosing the courses, chance for
potential research, increased earning potential, second chance to divert
one's career to a more relevant choice. While some may argue that
these courses seem of no use to the student, the courses actually come
in handy to a student in the long run. "Known is a drop; Unknown is an
ocean" goes a saying. Hence, there is no limit to learning and learning
continues throughout one's life. Thus, there is really no harm in choosing
a variety of courses outside one's field of study.
Claim: The surest indicator of a great nation is not the achievements of its
rulers, artists, or scientists.
Reason: The surest indicator of a great nation is actually the welfare of all its
people.
Write a response in which you discuss the extent to which you agree or
disagree with the claim and the reason on which that claim is based.

The phenomenal achievements obtained by the rulers, artists and


scientists are of great significance as they bring pride, fame and laurels
to the country. But it is difficult to ascertain whether these achievements
ensure a good life for the majority of the citizens of the nation. This is
quite a contentious issue. However, when observed factually the
countries fame and laurels may not ensure a good life of the common
citizen.
The factors that ensure a good life of the citizens cannot be generalized
and attributed to the achievements of popular It is based on a variety of
factors.Some of these factors could be a growing economy, low
unemployment rates, health and food security. These factors will truly
ensure that even the common man is able to keep his family satisfied.
The achievements of the rulers, artists and scientists in no possible way
generate revenue for the common man. A popular painting made by a
renowned artist will make the nation proud, but in no way does it help the
common man overcome the difficulties in life. An invention by a scientist
may bring laurels to a nation, but it will not help the people financially or
socially. Unless, the invention is implemented by an entrepreneur to
build a company, thereby creating jobs and employment for the people
of the nation.
The true unsung heroes of the nation are its Entrepreneurs.
Entrepreneurs with their innovative thinking help a nation in a variety of
ways. First they create jobs, these jobs are taken by the people of the
nation. Hence, it reduces the unemployment. With jobs in hand the
common man can strive hard to ensure growth of the company and earn
money for the satisfaction of his family. Thus it can be observed that the
work done by the entrepreneurs improve both the economy and the
standard of living of the citizens of a nation.
It can be argued that the work done by these popular figures does
influence the people of the nation positively to pursue their dreams with
passion and zeal. But this positive influence is not enough as the person
should have the determination and motivation to succeed within himself.
To ensure a good life is in ones hand and should be willing to pursue
success.
Considering all the above factors of the need of a growing economy, low
unemployment rates, health and food security necessary for a
satisfactory life it can be concluded that the achievements of the rulers,
scientists and artists may bring fame, pride and laurels to the nation but
do not ensure a good and satisfactory life for the majority of the nations
citizens.

Any leader who is quickly and easily influenced by shifts in popular opinion will
accomplish little.
Write a response in which you discuss the extent to which you agree or
disagree with the statement and explain your reasoning for the position you
take. In developing and supporting your position, you should consider ways in
which the statement might or might not hold true and explain how these
considerations shape your position.

The leaders, be it in the field of politics, socialist reforms, freedom


movements , art , business firm or other institutions, have always shaped
and molded the course of history. In advocating social reforms,for
example Kofia Anan raised his voice against apartheid and the
discrimination against the blacks in South Africa. The "popular
opinion",then, being the practising of slavery by the White people. He did
not give in to the popular opinion but ,instead, went against it. Looking
back into the past, we can see that most of the leaders have challenged
the then accepted popular opinion and achieved huge reverence and
renown for eradicating those practises. Like in India, during the freedom
movements when everyone resorted to violence for attaining freedom
from the oppresive British Rule, Mahatma Gandhi propagated the rarely
used method of Non-Violence and Truth. This not only helped India
attain Independence but also resulted in a government and constitution
with truth and Non-violence as foundations. Similarly, Mao Zendong led
to the establishment of Communism in China and so did Lenin when
these were not the populist ideas. The leaders, in these cases, have
achieved success by advocating against the populist methods.
Everytime when changes need to made , it always occur by going
against the populist opinion and not along with it. The same can be
extended to the field of business and manufacturing. What
pandemonium will occur if the business men kept changing their policies
with the flicker in market and did not rely on the potential of the
strategies he has implemented? If the Governor of a Central Bank kept
changing rates on the flicker in market prices or rise in inflation or
recession without properly analyzing the cause. The key phrase here is
"quickly and easily influenced". On not being influenced by the populist
opinions, it doesn’t mean that the leader should be adamant on his stand
come what may the circumstances. Since, leader being a human, is
equally erring and fallible, he should not rule out the possibility that
change in populist opinions might have come from a reliable and
constant source and is not evanescent. Hence, the same demands a
change in policies , ideas or strategy. Failing to recognize these changes
might lead to resentment in public if it concerns social , political or
economic stances or the employees in case of a business firm. Like for
example, the native practices of the monarchs in middle-east were
acceptable to people to a extent but when people demanded changes,
the leaders, be it of Libya, Syria or Egypt, did not pay heed to it and this
led to the Arab spring and huge loss to life and property. In firms and
organisations, the same leads to strikes by labor unions and employees.
Succinctly, it is very true that the leader should not submit to the frequent
change in populist opinions since the same will lead to confusion and will
prevent the accomplishment of the set goals. Nevertheless, leaders
should always analyse the cause and sources of such opinions to
ensure efficacy.

A nation should require all of its students to study the same national
curriculum until they enter college.
Write a response in which you discuss the extent to which you agree or
disagree with the statement and explain your reasoning for the position you
take. In developing and supporting your position, you should consider ways in
which the statement might or might not hold true and explain how these
considerations shape your position.

Already for years there are speculations about reforming the school
system in a lot of countries including the United States. The leading
example for all nations is of course Finland which consistently strands on
the top spot in all global educational rankings, from math to grammar.
They have a national curriculum that every student needs to follow until
they enter college. Should other nations adopt this approach to
education or isn't everything as bright as it seems? A scrutiny regarding
this topic will be necessary to give a decent answer.
One the one hand, one can say that a national curriculum certainly has
its plus sides. It provides a whole nation with the same knowledge, which
creates a coherent pack of students with the same educational
background. This makes it easier for colleges to adjust their classes,
since they now exactly know with what previous knowledge students
arrive. For the same reaon, this approach makes it easier to compare
schools with each other. Having the same curriculum will make clear
which schools offer a better education by comparing graduating and
drop out percentages (although other factors influence these
percentages as well). For students who need to transfer schools there is
also a benefit. Instead of comparing all courses they had with the ones
the new school offers, they will immediately fit in and be able to pick up
exactly where they left. For comparing purposes, the national curriculum
has without a doubt a benefit, ranging from cmparing individual students
to whole schools.
On the other hand, a national curriculum may limit certain students in
what they are able to achieve. By not offering the classes they would like
to see, they might become demotivated and drop out of school. After all
the point of education is to make students warm for studying, to trigger
their interest in certain subjects. If a school fails to achieve this goal due
to a forced national curriculum, that would be a pity. The student's
willingness and eagerness to learn should come in the first place.
However this is a valid argument, this approach suits more the one of a
college or university, where a student can study what they want. With the
necessary flexibility, a national curriculum might thus do the job for high
school students.
What should not be forgotten is that a national curriculum alone will not
get countries to the same international standards as Finland. However it
may help, the true heart of education lies with the teachers. If the
teachers are good and can get their point across, students will benefit
greatly. Therefore one should take a look how teachers are looking
against this national curriculum. If they agree, they will give their heart
and soul for this program, resulting in an overall better education. If they
oppose the reform, their teaching will most certainly deteriorate, leaving
us with a possibly better school system, but with less educational
classes due to demoralised teachers. Therefore the implementation of
such a reform of the educational system should be agreed upon by the
teaching community before government requires it.
In sum we can say that a national curriculum certainly has its benefits,
enabling us to easily compare students and schools with each other.
However there should be some flexibility in the program leaving the
possibility to make minor adjustments to suit the learning interests of the
student so as to keep him/her interested in learning. But after all the big
question is if the teaching community agrees with the proposal, because
they will determine the level of classes. A national curriculum will
therefore be a succcess if the teachers are standing behind this
reformation of education.

It is primarily in cities that a nation's cultural traditions are generated and


preserved.
Write a response in which you discuss the extent to which you agree or
disagree with the statement and explain your reasoning for the position you
take. In developing and supporting your position, you should consider ways in
which the statement might or might not hold true and explain how these
considerations shape your position.

It is popularly believed that nation's cultural traditions are mainly


generated and preserved in metropolises. However, may we concur with
this claim? Perhaps, only partly owing to the fact that big cities indeed
accumulate nation's conveniences; at the same time, the traditions are
well preserved and generated in regional towns and villages. The
reasons of my stance will be discussed in details below.
To begin with, nation's conveniences are accumulated in metropolises in
particular capitals. Those major cities represent the country and tend to
collect and display its' national ceremonies and traditions. For instance,
we may take a look at Kyoto which is a former capital of Japan. The city
is well-known for its festival of fire which takes place each winter. In fact,
this megalopolis is embodiment of Japanese culture, nation's attitude to
life which is demonstrated via nation's ceremonies and conveniences.
You may assume that Japan is an exemption but you may take a look at
Paris, Berlin or Oslo and you will see that all of those capitals proudly
display their nation's features and traditions.
However, it is mistaken to assume that nation's traditions are preserved
and created exclusively in those metropolises. In fact, regional towns do
the job as well in particular when we have a deal with countries which
include many nations. For instance, Russia and China were created by
conquering new territories; people who live in those territories have
become part of the country but they still live in their historical locations
and stick to their nations’ conveniences. Consequently, each local town
represents local nation and its traditions. In fact, almost each region of
Russia has its own holidays which reflect the conveniences of the nation
which inhabit it. For instance, people of Tatarstan regularly organize
competitions which reflect their traditional nomadic life-style, other
nations do the same. From this viewpoint, capitals represent averaged
culture and regional cities and town preserve and accumulate
conveniences and traditions of local demos.
Furthermore, we ought to devote our attention to rural areas and villages
where people have still led a traditional life style. In those areas, mores,
ceremonies and traditions are interwoven with everyday life and
preserved untouched. For example, we may find villages in Siberia
which still do not have electricity and the internet as well as other
contemporary conveniences. People who live in those areas stick to
traditions which for instance they wear national clothes, cook local
cuisine etc. For them nation’s traditions and ceremonies are a part of
routine. For example, wedding ceremonies are preserved and full of
meanings for those people. Those traditions play significant and
important role in their life. From this prospect, such villages play a role of
storage of nation’s conveniences. Such villages may be found in China,
Brazil and even the USA.
In conclusion, although many of us believe that nation's conveniences
are generated and preserved primarily in major cities, it is only partly true
owing to the fact that regional towns and rural villages do the same job
as well.

We can learn much more from people whose views we share than from
people whose views contradict our own.
Write a response in which you discuss the extent to which you agree or
disagree with the statement and explain your reasoning for the position you
take. In developing and supporting your position, you should consider ways in
which the statement might or might not hold true and explain how these
considerations shape your position.

To best learn one should glean from eclectic information. Yet, this does
still leave open the question of whether or not those who you agree with,
or disagree with, are most effective a having information imbibed into
you. The prompt suggests that it is those who you have the most in
common that can more effectively teach you, however I would dispute
that. I would say that those who you disagree with in general can teach
you more than those you agree with, and for this I present 3 reasons.
To begin consider the problem of information overlap. For those who you
agree with you by definition share the same views; views for which are
the result of postulates, or ideals, you believe to be true. When
ideologically similar people tend to know the same set of facts, because
no coincidently those facts are what shaped their belief in the first place.
For instance, consider two people who are both say socialists. Two
socialists would generally believe that wealth inequality is a systematic
flaw in capitalism that is the result of class oppression, rather than some
inherent human inequity of skill. They could potentially base this view on
children being born to wealthy families tending to have more successful
futures than those born to poor families. The problem with this, is it
ignores one critical fact, namely that those who are smarter, tend to be
wealther and tend to have smarter kids. So, due to their beliefs being so
similar they are missing information, that if they had someone from with
another view would not have been overlooked. Moveover they actually
gain little by talking to eachother about the topic, since they both already
know the facts that support their point of view.
Furthermore, listening only to people who agree with you can make you
complacent. We as humans require constant mental challenge to keep
us sharp. The most effective way of doing this is through challenging our
held beliefs. Only by having our assumptions and ideals questioned can
we really understand why they are true so, and understand their finer
details. Consider a man who has all his life believed that eating in public
is wrong. He believes it to be wrong, because his mother taught him it
was wrong, and her mother before her. This man can go his entire life
never understanding, or even thinking about why he thinks it is
disrespectful, if he is never challenged. Moreover, if he was challenged
he would be required to think why he believes it is disrespectful, why it
ought to be discouraged. Only by being questioned on his belief can a
induced need to truely understand the things one believes come into
being, thus allowing them to learn at a more indepth level.
Adding on to this, only through challenge can we even know when are
beliefs are wrong. It was assumed with the last example that the man
would come to the conclusion that eating in public was rude, however
that isn't necessarily true. I would say most people in the west do not
consider eating in public rude, but without a challenge the man very
could go his entire life never even questioning what he had been taught.
A challenge from opposition can give arguments that the man has never
heard of, ones that someone who agrees with would not know.
Arguments that in particular can show a man he is wrong, which is
something that can only be taught from those whom you disagree with.
In conclusion, it can be generally said that a mixture of both Pro and Con
points of view is the superior way to learn, if you only could have one, a
view contrary to your own is better for learning. This is because with
people you disagree with, you can be taught where you are wrong, they
induce you to understand why you believe the things you do, and finally
you are kept sharp.

When old buildings stand on ground that modern planners feel could be better
used for modern purposes, modern development should be given precedence
over the preservation of historic buildings.
Write a response in which you discuss the extent to which you agree or
disagree with the statement and explain your reasoning for the position you
take. In developing and supporting your position, you should consider ways in
which the statement might or might not hold true and explain how these
considerations shape your position.

One of the most important rights of the people is the accommodation.


But now a day it is becoming scarce because of the rapid growth of
population. But unfortunately the population is not uniformly distributed
throughout the world. Some of the countries are highly dense compared
to others. Even within the same country some places or cities are denser
than the others. So accommodation is a problem in those areas. In those
cases the difficult decisions have to make regarding the fate of the
historic buildings. Here the author speaks in favor of preserving the
historic buildings rather than to demolish them to build modern ones. I do
agree with the suggestions for the following reasons.
It is an era of modern technology. People are building skyscrapers. A lot
of technologies are available to build those. What we need is to make
the best use of those technologies and build some skyscrapers.
Because when new building are made demolishing the historic one then
it might happen that the accommodation is still inadequate. Then the
ultimate plan would be to build skyscrapers. So better to start that trend
now rather destroy the historic buildings. That’s why large cities have
multistoried buildings to meet the need of accommodation.
It is also possible to make a city plan preserving the historic buildings. In
that case it will increase the aesthetic value of the city. For example
Rome still has a lot of historic places that have not only increased the
beauty of the city but also has attracted a lot of tourists. This strategy will
make a contribution to the economy of the nations. Dhaka, the most
populated city in the world, has still maintained their historic places and
the establishment even though there is a severe accommodation
problem. The city engineers have decided to go for multistoried building
without destroying those.
Historic buildings are not so frequent. So question arises regarding the
contribution to the solution of the accommodation problems if those are
replaced by the modern one. I think it will be a very minor percentage.
So it is better to preserve those establishments. Apart from that those
buildings also represent the history of the nation. Sometimes they
represent the pride of the nation or a civilization that has been extinct. In
that sense they might be very good educational materials.
In conclusion I would like to say there are so many reasons to claim that
the historic buildings should be grounded and should be replaced by the
modern one. But the fact is that it is going to destroy the history of the
nation, the history that is cherished by the mass people. At the same
time the replacement of the historic building might not solve the problem
permanently. So we need to look for a permanent solution. And in that
case it is important to make use of other unutilized places and to
preserve the historic buildings.
.

Claim: The surest indicator of a great nation must be the achievements of its
rulers, artists, or scientists.
Reason: Great achievements by a nation's rulers, artists, or scientists will
ensure a good life for the majority of that nation's people.
Write a response in which you discuss the extent to which you agree or
disagree with the claim and the reason on which that claim is based.

The phenomenal achievements obtained by the rulers, artists and


scientists are of great significance as they bring pride, fame and laurels
to the country. But it is difficult to ascertain whether these achievements
ensure a good life for the majority of the citizens of the nation. This is
quite a contentious issue. However, when observed factually the
countries fame and laurels may not ensure a good life of the common
citizen.
The factors that ensure a good life of the citizens cannot be generalized
and attributed to the achievements of popular It is based on a variety of
factors.Some of these factors could be a growing economy, low
unemployment rates, health and food security. These factors will truly
ensure that even the common man is able to keep his family satisfied.
The achievements of the rulers, artists and scientists in no possible way
generate revenue for the common man. A popular painting made by a
renowned artist will make the nation proud, but in no way does it help the
common man overcome the difficulties in life. An invention by a scientist
may bring laurels to a nation, but it will not help the people financially or
socially. Unless, the invention is implemented by an entrepreneur to
build a company, thereby creating jobs and employment for the people
of the nation.
The true unsung heroes of the nation are its Entrepreneurs.
Entrepreneurs with their innovative thinking help a nation in a variety of
ways. First they create jobs, these jobs are taken by the people of the
nation. Hence, it reduces the unemployment. With jobs in hand the
common man can strive hard to ensure growth of the company and earn
money for the satisfaction of his family. Thus it can be observed that the
work done by the entrepreneurs improve both the economy and the
standard of living of the citizens of a nation.
It can be argued that the work done by these popular figures does
influence the people of the nation positively to pursue their dreams with
passion and zeal. But this positive influence is not enough as the person
should have the determination and motivation to succeed within himself.
To ensure a good life is in ones hand and should be willing to pursue
success.
Considering all the above factors of the need of a growing economy, low
unemployment rates, health and food security necessary for a
satisfactory life it can be concluded that the achievements of the rulers,
scientists and artists may bring fame, pride and laurels to the nation but
do not ensure a good and satisfactory life for the majority of the nations
citizens.

Some people claim that you can tell whether a nation is great by looking at the
achievements of its rulers, artists, or scientists. Others argue that the surest
indicator of a great nation is, in fact, the general welfare of all its people.
Write a response in which you discuss which view more closely aligns with
your own position and explain your reasoning for the position you take. In
developing and supporting your position, you should address both of the
views presented

What is the most ideal and effective standard to evaluate the greatness
of a nation? Some may argue that it is achievements made by individual
experts, such as rulers, artists, or scientists, as the statement contends,
while others argue that it should be the general welfare of the populace. I
believe that a nation should be assessed based on the general well-
being of is people for the following reasons.
Firstly, though individuals can contribute to a nation by their
achievements, it may not successfully improve the greatness of the
country. To say a nation is great, the people living in the country should
be at least provided by decent welfare system, which enables them to
maintain healthy and safe daily life as well as to be educated as much as
they wish. Unfortunately, achievements of individual specialists are not
always relevant to the welfare of the public. In Olympic, for example,
where individuals participate representing one’s nation, winning a medal
could be such a fame of not only for the player but also for the country.
Such amazing achievement, however, cannot feed children suffer from
famine, provide education for illiterate people, or help people who lost
their houses from flood. If a nation cannot take care of the most basic
needs of its citizenry, however great achievements individuals made, it
cannot be considered great.
Secondly, although the achievements are relevant to the welfare issue of
the public, the government may fail to distribute the benefit of the
achievements. When public system is corrupted, it becomes very difficult
to reach and provide support to the mid and lower class people. For
example, Korea is known for its advanced Information Technology,
which was led by a few renowned scientists back in 1990s. However,
because of the corrupted system, the e-governance system, which was
established by those skillful scientists to benefit the public, is failing to
serve the public, after consuming the uncountable research and
development funding. As this shows, a nation cannot become great only
with the achievements of its experts, but it requires a prudent system,
which can distribute the benefit.
Nevertheless, some may still argue that it is still possible to properly
evaluate whether a nation is praiseworthy based on the outcome of a
few specialists. However, when we think about what would require a
nation to serve its people adequately, it would be too naïve to say that
the individual’s achievements are enough. To assess a nation, more
holistic and comprehensive approach is required, which includes well-
being of public in education, medical services, security, and political and
religious freedom.
In conclusion, while accomplishments of individual experts in diverse
fields may contribute to the greatness of a nation, there are other more
critical factors that can tell us about the nation. Thus, it is also vital to
consider a nation as a whole, especially focusing on the well-being of its
citizenry when discussing about how great a nation is.

The best way to understand the character of a society is to examine the


character of the men and women that the society chooses as its heroes or its
role models.
Write a response in which you discuss the extent to which you agree or
disagree with the claim. In developing and supporting your position, be sure to
address the most compelling reasons and/or examples that could be used to
challenge your position.

The claim emphasizes the leading role of heroes corresponding to a


certain society in studying the character of it. Admittedly, their characters
can illustrate part of the values of the society, social study, in my point of
view, should rest its weight on mass people and other idiosyncrasies of
the society.
According to the claim, we can concede that there must be some
reasons requiring our notice to account for the selection of role models
and heroes by the society. Since they are acclaimed by the society, they
must carry on some qualities that are regarded lofty, highly respected or
of great significance of the society. On that sense, it can be inferred that
the values of the society are in favor of these qualities, that is why the
examination of its role models can reveal part of the characters of a
society.
However, the characters of these models are after all personal, far from
being epitomized for a society. The society consists of a diversity of
classes while each of them bears their own specific qualities and needs.
Suppose, how are several role models representative of all those
classes? In some cases, they are praised for precious qualities that are
short of in that society; in other cases, those qualities will achieve
rewards from a large range of societies. Having born those possibilities
in mind, we will find that the study towards them become ambiguous and
granted.
To deal with the issue, it is suggested that scholars transfer their
attention from these representatives to common people. All in all, it is
mass people who create the history. I am sure that you have heard of
Watt, who invented steam engine, but do you know the names of the first
people who discovered the cultivation of grains? You may argue that
they are not reported by documentaries. Nevertheless, it is the
agriculture that ends up the era of people's nomadic lives and it is
extremely hard for those people to find their way to create rise corn and
wheat for survive. To some extent, we owe our life today to them, whom
share a single name-"people". Hence, history study can't ignore them in
any society.
From what has been discussed above, we can see that common people
play an indisputable role in society, which is irreplaceable by heroes and
role models.

All college and university students would benefit from spending at least one
semester studying in a foreign country.
Write a response in which you discuss the extent to which you agree or
disagree with the statement and explain your reasoning for the position you
take. In developing and supporting your position, you should consider ways in
which the statement might or might not hold true and explain how these
considerations shape your position.

Education is the important aspects of a persons' life. A college and


university student cannot be truly educated unless he takes a broad
range of education. As the trend of globalization is increasingly
expanding, a large number of college and university are seeking
cooperation with ones in a foreign country by changing their students.
Would all college and university would benefit from spending at least one
semester. Admittedly, such a policy are favorable as a whole. In my
perspective, however, we should not overlook some potentially adverse
effects.
As is know to all, culture exposure is a integrated part of education.
While academical knowledge equips a students the basic skills, culture
exposure not only helps a student expand his perspective but also
enable him to learn tolerance,respect of other culture and appreciation of
disparity. Thus to learn at least a semester in a foreign country provides
undoubtedly the best chance with a student to knowledge the culture in
another country which is completely different from his own. Indeed, the
real culture is far different from what you are taught by textbooks or told
by others.For instance, after I live in United Stated for three months, I
love it in many respects including the religious culture and democratic
principle.In light of this , a study in a foreign country is benefit.
In the other hand, a study in foreign country enable a student to improve
his academic ability. Few will disagree that the duty to study in foreign
country is to focus on his study field regardless in science field or
societal field. Indeed, to study in another country means that a student
will cooperate with other professor and partners who will exhibit their
research results as well as their thinking ways and research
means.Indeed, what can a student learn in a short time is always the
means. Every years, the National Education Foundation funds many
Chinese students go to study in the United States in hope that these
students can exert advanced scientific means in Unites States into the
research in China. Therefore, such fact further demonstrates it is
necessary to study in a foreign language.
However, to study in a foreign country ineluctably have some problems
that the students must overcome, such as financial problem and
language problem. While it is clear that not all can afford to study in a
foreign language, the funding from the government is limited. In addition,
the most important thing that is disadvantageous is that some students
spend study time on travelling.Indeed, it is uncommon that many
Chinese visiting students travel around the United Stated with the
funding.
In conclusion, the college and university students will benefit from at
least a semester in a foreign country. However, the potentially adverse
effects should not be overlooked. Guaranteed by an specific funding
policy, a nation should support and encourage its college and university
students to study at least one semester in a foreign country.
Some people claim that a nation's government should preserve its wilderness
areas in their natural state. Others argue that these areas should be
developed for potential economic gain.
Write a response in which you discuss which view more closely aligns with
your own position and explain your reasoning for the position you take. In
developing and supporting your position, you should address both of the
views presented.

Organisations like WWF (World Wide Fund for Nature) and GAIA (Global
Action in the Interest of Animals, a Belgian organization) don't always
agree, but one point they can't stress enough is that a nation's
government should preserve its wilderness areas in their natural states.
However this belief is generally regarded as containing a lot of truth, it
evokes some criticism by economical minded people. But a thriving
economy can not outweigh the pluspoints a good wildlife can provide.
To begin with, a preservation of wilderness areas means that a diversity
in fauna and flora is kept alive. This diversity is important for keeping up
a florishing ecosystem. If a few species or plants become extinct, it may
cause a butterfly effect, affecting a small region near the area or even a
large percentage of living species.
Another point of criticism is that a wilderness area also has economical
functions. First of all, the roots keep the earth together and prevent the
region from erosion. As a surplus these roots also suck up water, which
keeps the region from flooding. Secondly, plants help to filter
carbondioxide out of the air and replace it by oxygen which is what we
need to keep ourselves alive. The countries that want to develop these
areas for economical gain, most of the time are industrialised nations
who have still a lot of work to do to reach their Kyoto-norm. However by
leaving wilderness areas intact, one is again a step closer to reaching
this notorious norm.
Economic gain on the other hand could be advantageous for a nation as
well, helping people to get jobs and providing a strong economy to the
country. In times where the economical crisis is still freshly on our mind,
this option is a tempting one. Although the advantages can easiliy be
seen, one should however resist this temptation and look at the potential
financial and ecological consequences this might make.
Therefore the unified protest against the developing of wilderness areas
for economical gain by WWF and GAIA should get more support to
persuade the government to undertake action and keep the wilderness
alive.

In most professions and academic fields, imagination is more important than


knowledge.
Write a response in which you discuss the extent to which you agree or
disagree with the claim. In developing and supporting your position, be sure to
address the most compelling reasons and/or examples that could be used to
challenge your position.

Knowledge and imagination have propelled society in all fields - since


the dawn of man. It is through imagination man has invented the wheel
and learned to cook meat with fire - and it is through the learning
process - knowledge has been passed on. Knowledge marks an
accumulative process through which society better itself and it is
imagination that allows individuals to impact society - in a symbiotic
process that nourishes all. Society is composed of a population divided,
albeit artificially, to professions - be proper daily professions or academic
ones - all embarked on a journey that requires both knowledge and
imagination.
Most professions and academic fields hold, by definition. the
accumulation of knowledge that has been reshaped into either practical ,
theoretical and academic forms. For a man to pursue a career, be it any
of the former, he cannot exclude himself from the world, as to presume
one could create all the knowledge of a profession all by himself. True,
as in the case for the lone Indian mathematician who found a
mathematical book and has reinvented numerous mathematical
branches, people could live a life of solitude and create all by
themselves - but even within the academic society, these are the
exceptions - not the common. It is through knowledge that can be learnt,
one can gain the experience of tens, if not hundred of years - if not even
more - and to stand on the frontier of his profession - be the best
detective one can be, the best scientist - or a great humanitarian.
Nonetheless, we must distinguish between proper,dull professions in
which little imagination is required, such as insurance agents - opposed
to the works of policemen, detectives and firemen on one hand - and
scientists or engineers on the other. It is not practical to gain all
knowledge of a profession, nor is it scalable with the ever expanding
knowledge of humanity. Those wishing to explore new realms of physics,
invent new gadgets, explore fields for which previous knowledge is
lacking - must employ deep thought processes, exhibit ingenuity, all the
remarks of the great minds, such as Albert Einstein - it is the works of an
alternate thinker, a cerebral thinker, through the use of the left, artistic
side of our brain, imagination can lead us to realms previous explorers
never dreamed of.
Imagination and knowledge are both distinctive marks of professions -
be it academic or not - and to go as to the extreme of stating which is
more important - is a tedious task. All professions could be thought of
requiring a mixture of the both. From the mundane salesman, who must
show creativity in order to support his sales rate - to the common
scientist - for whom imagination is an impeccable trait - his gained
knowledge of many years of study are not less that mandatory for proper
research. One must ask, what professions require no imagination or
pure knowledge alone - and who, in a sane mind, would be keen to work
or participate in such professions? A profession without the use
imagination can be described as robotic - the same very jobs in
manufacturer lines in which actual robots replace human beings. On the
other hand, the realm of pure imagination, arguably, can be assigned to
children alone - since any profession in imagination is used - the product
of such a process must be manifested - a play, a book or a publication -
even for the sheer share of knowledge.
In sum, most professions use both imagination and knowledge - yet in
any profession - the use of either exists on a continuous spectrum.

The surest indicator of a great nation is not the achievements of its rulers,
artists, or scientists, but the general well-being of all its people.
Write a response in which you discuss the extent to which you agree or
disagree with the claim. In developing and supporting your position, be sure to
address the most compelling reasons and/or examples that could be used to
challenge your position.

What is the most ideal and effective standard to evaluate the greatness
of a nation? Some may argue that it is achievements made by individual
experts, such as rulers, artists, or scientists, as the statement contends,
while others argue that it should be the general welfare of the populace. I
believe that a nation should be assessed based on the general well-
being of is people for the following reasons.
Firstly, though individuals can contribute to a nation by their
achievements, it may not successfully improve the greatness of the
country. To say a nation is great, the people living in the country should
be at least provided by decent welfare system, which enables them to
maintain healthy and safe daily life as well as to be educated as much as
they wish. Unfortunately, achievements of individual specialists are not
always relevant to the welfare of the public. In Olympic, for example,
where individuals participate representing one’s nation, winning a medal
could be such a fame of not only for the player but also for the country.
Such amazing achievement, however, cannot feed children suffer from
famine, provide education for illiterate people, or help people who lost
their houses from flood. If a nation cannot take care of the most basic
needs of its citizenry, however great achievements individuals made, it
cannot be considered great.
Secondly, although the achievements are relevant to the welfare issue of
the public, the government may fail to distribute the benefit of the
achievements. When public system is corrupted, it becomes very difficult
to reach and provide support to the mid and lower class people. For
example, Korea is known for its advanced Information Technology,
which was led by a few renowned scientists back in 1990s. However,
because of the corrupted system, the e-governance system, which was
established by those skillful scientists to benefit the public, is failing to
serve the public, after consuming the uncountable research and
development funding. As this shows, a nation cannot become great only
with the achievements of its experts, but it requires a prudent system,
which can distribute the benefit.
Nevertheless, some may still argue that it is still possible to properly
evaluate whether a nation is praiseworthy based on the outcome of a
few specialists. However, when we think about what would require a
nation to serve its people adequately, it would be too naïve to say that
the individual’s achievements are enough. To assess a nation, more
holistic and comprehensive approach is required, which includes well-
being of public in education, medical services, security, and political and
religious freedom.
In conclusion, while accomplishments of individual experts in diverse
fields may contribute to the greatness of a nation, there are other more
critical factors that can tell us about the nation. Thus, it is also vital to
consider a nation as a whole, especially focusing on the well-being of its
citizenry when discussing about how great a nation is.
. College students should base their choice of a field of study on the
availability of jobs in that field.
Write a response in which you discuss the extent to which you agree or
disagree with the recommendation and explain your reasoning for the position
you take. In developing and supporting your position, describe specific
circumstances in which adopting the recommendation would or would not be
advantageous and explain how these examples shape your position.

It is commonly said that one who loves his job, excels at his job. The
mere availability of open employment positions in the workforce should
not compel a job seeker to concentrate on that particular field of study.
While some may argue that one should search for jobs based on the
number of available positions, it is clear that the employee will not find
utmost satisfaction and consequently, fail to have a great job experience
throughout their working life. I believe that the aforementioned statement
above does not fare well and is inaccurate.
Most people have heard of Sanjay Gupta, the chief medical
correspondent for CNN. He pursued a degree in biomedical sciences
before completing his medical school education at the University of
Michigan. Gupta is currently practicing as an esteemed neurosurgeon
and also works as a professor of neurosurgery at Emory University. The
amount of neurosurgeons is quite limited in comparison to other
specializations and jobs in the healthcare force. The need for a
neurosurgeon is usually quite rare in the general population, and
therefore, the availability of jobs for that field are few, in comparison to
perhaps, general practitioners. However, Gupta put in years and years of
hard work to balance the multiple jobs he currently possesses. Despite
his knowledge of the limited availability of neurosurgeons, he studied the
field of neuroscience to prove his worth to the medical community.
Consider Indra Nooyi, the Chief Executive Office of the multinational
corporation, Pepsi Co. During her early years, she was studying at a
relatively small liberal arts college and then received her MBA in another
Indian University. She got admitted to Yale School of Management and
was then employed into Boston Consulting Group. After working in a few
more companies, she was able to break through the door and after
numerous accomplishments, sit on her throne as CEO of Pepsi Co. Her
dream all along was to manage a massive company. Of course, the
probability of being a CEO is very small and the job openings do not
come so easily at all in the professional field. However, with her
immense drive and dedication, Nooyi efficiently climbed up the ladder of
business to achieve her incredible goal. Nooyi serves as a model for
other college students who aspire big and strive to secure a job that has
very few openings in the workforce. As long as one is interested and
willing to put in copious amounts of effort, the availability of jobs should
not prevent one from choosing their field of specialization.
Gupta and Nooyi only represent a fraction of what other college students
have to offer. The positions of these jobs with less availability are still
looking to be filled. Students should not be discouraged from aspiring big
and looking to lock down the rarest of all professions. When a university
student appears zealous and ready to take on whatever the grueling
professional world has to offer, he or she can certainly conquer incoming
obstacles and further succeed in the workforce if they can devote their
time and interests to their chosen field of study.
Some people believe that corporations have a responsibility to promote the
well-being of the societies and environments in which they operate. Others
believe that the only responsibility of corporations, provided they operate
within the law, is to make as much money as possible.
Write a response in which you discuss which view more closely aligns with
your own position and explain your reasoning for the position you take. In
developing and supporting your position, you should address both of the
views presented.

Business and society are not two distinct quantity. They are highly
correlated. No business can survive with out the active cooperation of
the society. It is true that the cooperation should make profit but it can be
achieved with a responsibility to promote well-being of the societies also.
So in my opinion if any corporation want to give back something to the
society it will not decrease their profit but actually it will ensure the
sustainability of the corporation.
Corporate social responsibility comes into picture when corporation
actively tries to promote the well being of the society. The concept of
social responsibility is relatively new but many giant corporation followed
it. There are two ways of benefit from it. One is that when the corporation
agreed to follow the social responsibility then their tax margin is
somewhat lowered. It is because the corporation are doing the same job
as the tax money does. On the other hand their help to society will repay
back in their long term run.
Take some example of corporate social responsibility to understand
clearly. Giant shoe making corporation BATA hired many unskilled labor
at their adjoin places of factory. Train and make them skilled worker. In
this way the social and economic condition of the adjoin area of that
factory is changed. The society get benefited as the lifestyle of some
underprivileged people is lifted up and the corporation get profited as
they get some cheap skilled worker .
The repay back to the society may effect little region or it may effect a
large part of the society. For example Darjeeling Tea was previously
packed in paper container. But it may be packed in jute bag also. Now
the management of that corporation decided that instead of paper they
should use jute bag for packing of Tea. The jute industry before that was
seriously endangered. But after that use of jute is increased . The jute
industry which was previously about to close regain its profit again. So in
this way the Tea corporation commit a social responsibility.
In conclusion society and business all are interconnected. When a
society is able to buy then only the giant corporation able to make profit.
So it is the responsibility to the society that they should take care of the
society for their own sake. Profit should be a goal but it can be achieved
with social responsibility also. It has also been seen that when any
corporation is commit their social responsibility they can sustain in long
run.
Claim: Researchers should not limit their investigations to only those areas in
which they expect to discover something that has an immediate, practical
application.
Reason: It is impossible to predict the outcome of a line of research with any
certainty.
Write a response in which you discuss the extent to which you agree or
disagree with the claim and the reason on which that claim is based.

Research is of vital importance in this competitive world of ours. We


cannot undergo developments without serious research works being
conducted. People who pursue research are regarded with high prestige
and are considered equivalent to gods for they gift the society with mind-
boggling findings and inventions. Research work requires lots of
patience and diligent efforts to achieve success. In today's world, many
people just want to make as much money as possible in the shortest
possible time by doing a cozy job or involving in a business. In such a
scenario, it is highly contentious to limit the research works to areas that
have immediate practical application. A stand can be taken on this issue
only after a careful analysis.
Firstly, the ultimate aim of research is to firmly establish a new concept.
It seems so that concepts are useless unless they apply in some manner
fulfilling the needs of the society. On the contrary, in reality, all fruits of
renowned scientists which we enjoy now were not highly regarded
during those scientists' period. For instance, we cannot even imagine
traveling to another country without the help of aeroplanes. But in the
olden days, long journeys were carried out by ships or through walks
with periodic rests. We now realize that Wright brothers have gone a
long way in inventing aeroplanes for making our long journeys easier
and enjoyable. The application of the discovered results of a research is
analysed on-the-go and not as and when the results are produced.
Therefore, even if the research results does not result in an immediate
practical application, they may sometime in future turn out to be the most
important discoveries. Newton's theory of gravitation and Einstein's
theory of relativity seemed to be of trivial importance when they were put
forth before the society. On the other hand, today, most of the concepts
in physics could not establish their results without considering the
implications of these theories. Research done in any field will always
have utility. But it might not be relevant in the initial stages after research
work is being completed.
Secondly, the time and effort required to carry out a research work is
tremendous, to the extent of not justifying the compulsion of any
practical application. Huge amounts of money is also spent by
governments of countries like the United States of America in order to
make their nations forefront in the area of research. Unlike other
projects, a research project's consequences cannot be determined
beforehand. For instance, installation of nuclear power-plant in an area
in order to compensate for the electricity shortage might be a good
move. The scientists and Civil Engineers might also undergo an
extensive research about the targeted place in order to effectively design
a nuclear power-plant. But there exists a possibility of the untoward
incidents happening in case of leakage in the nuclear plant. The damage
that it would cause cannot be estimated and pre-cautionary measures
can be taken only to an extent so as to assuage the severity of the
purported damages. Therefore, uncertainty in research works should not
prevent one from carrying out them. Risks are meant to be taken in most
cases to come out with flying colors in the field of research. For example,
'Artificial Intelligence' in the field of computer science was touted to be
the active research field in the 1990's. After a decade, the research
works became dull in this area due to the fear of failure. But, now this
field has again gained momentum and countries are competing to bring
out a perfect humanoid robot. Therefore, considering the complexity of
research work, too much of thinking must not happen prior to the start of
a research.
Everyone today wants to do their activities in a jiffy. Subsequently, they
look forward to exciting gadgets and appliances to make their life easier.
However, there exists another part of the society beyond the common
people. The teachers, doctors, engineers etc... also require help in their
profession. Hence, research works must be evenly carried out across all
the fields. Some might be of the opinion that a research-work is not
worth pursuing, if it does not cater to the uses of majority of the people.
However, all the professions are interlinked. For example, if a serious
disease rocks the world tomorrow, then the medical researchers have to
come up with a promising treatment or a medication; Otherwise, the
disease will make a huge impact and affect people from all walks of life.
Same is the case with computers which are operated by almost all
professionals. As a result, all fields are interlinked and necessary to
ensure healthy and comfortable survival of the humanity.
Thus, I completely agree with the claim that researchers should not
restrict themselves to areas which have potential applications. As quoted
in the reason, the output of research works cannot be determined with
accuracy. Hence research works are to be carried out by the individuals
out of their own interest. Researchers ought to devote their time in
finding new concepts and leaving the application-part of it to the society.
For example, Ramnujam's mathematical derivations were not of much
use in his days. But today, they are highly regarded and researchers are
still working on his theorems to make new implications. Research works
take a humongous amount of time and sincere effort to establish
something worthwhile and research in all fields is necessary to ensure a
better living.
Some people believe that our ever-increasing use of technology significantly
reduces our opportunities for human interaction. Other people believe that
technology provides us with new and better ways to communicate and
connect with one another.
Write a response in which you discuss which view more closely aligns with
your own position and explain your reasoning for the position you take. In
developing and supporting your position, you should address both of the
views presented.

Technology is the sole requirement for human existence in today's


competitive world. Without technical advancements, it will be near to
impossible to survive in today's modern world. Each activity of ours is
mandated by technology which creeps into all of our lives in some form
or the other. Whether techology enhances or depreciates the ways in
which we communicate is a contentious issue. Technological
advancements have gone a long way in creating new forms of
communication, which fare far better than the traditional face to face
communication.
First of all, technology helps us in connecting people located at various
parts of the world. Without social networking sites like Facebook, today's
youth will be at a standstill; they will be no different than those who
existed a decade back. Since social networking does not require face-to-
face interaction, people from different parts of the world unite in a
common platform. Consequently, this form of communication helps in
students interacting with their potential employers and getting job offers;
these days, the whole phase of recruitment has changed to online,
primarily because of social networking. Because, they eliminate the need
to travel, this form of communication is mutually beneficial for the
interviewer as well as the interviewee. Moreover, these sites help
parents to connect to their wards who stay abroad for studies or other
reasons. Thus, technological advancements like social networking, bring
the whole world before us, in the form of social networking sites.
Moreover, use of technology is predominant in emergency cases in that,
it saves the lives of the people, who are at the brink of their death. For
instance, incase of accidents, mobile phones can be used to call the
nearby ambulance and do the necessary service. Similarly, when we
need to intimate an information to a person, and if it is an exigent
situation, then we resort to short text messages. These messages have
made the delivery of information to a person possible, even if he is
involved in a meeting or some important commitment. Thus, by
effectively utilizing technical wonders, we can deliver right information to
the right person at the right time. All these things happen in tandem,
solely attributed to the use of technology.
Furthermore, communication between teachers and students has grown
manifold, primarily because of the use of technology. Now students can
participate in online doubt-clearing sessions organised by their tutors.
Thus, the interaction between the tutors and students is facilitated and
encouraged. Moreover, education through video-conferencing is in
vogue today. Sites like coursera, provide the lectures of eminent
professors from Stanford, free of cost to students available to worldwide
and specialized sessions at subsidized costs. Therefore, standard
education can be guaranteed to a wide range of people and effective
learning through interaction is possible by effectively making use of
online features.
Above all, the developments in science have resulted in great
incorporations into the technical devices. For instance, we can decipher
whether a person is lying, by means of examining his voice modulation
through means of cellphones. These kinds of devices, take
communication to another extent, wherein application of scientific
concepts will enrich our communication and make them more reliable
and effective.
Some may argue that the opportunities for human interaction are
curtailed because of advancements in technology. They attribute their
statement to the fact that, humans do not involve in face-to-face
interaction. But, this claim is shattered by the new upcoming
developments, some of them including: video conferencing, answering
calls in mobile phones by means of 4G technology etc... As technology
evolves, it will even create a form of communication in future, which
exactly resembles direct interaction with other people. When it happens,
communication will transcend boundaries and people will not at all need
to move out of their seats.
All in all, technology connects people over distance, help in rendering
proper student-teacher interaction, instant communication during
emergencies and leads to saving time needed to travel. While there is a
view that direct interaction is not possible, they are indirectly made
possible through developing advancements. Hence, technology plays a
vital role in the communication process and it is a prerequisite for
effective communication.

Claim: Knowing about the past cannot help people to make important
decisions today.
Reason: The world today is significantly more complex than it was even in the
relatively recent past.
Write a response in which you discuss the extent to which you agree or
disagree with the claim and the reason on which that claim is based.

Due to technological progress in several fields, things have become


simpler to execute, but at the same time complex to develop. If one
compares today's high-pace life with the much simpler existence of our
ancestors, it seems valid to claim that things which happened before are
trivial to know as they are no longer helpful. However, the reason that as
things are getting more compact and complex, doesn't underestimates
the importance of our past developments as they continue to be the
cornerstone of many existing developments in diverse fields and will
continue to be in future.
Understanding the nature of a particular event in past has helped
historians and economist to analyze current trends and make important
decisions for today. Stock markets and traders used past trends of a
business to predict future loss/profit. Understanding the significance of a
particular culture or system of a country through the nature of past
happenings, gives historians insight into the reason due to which such
system exist and its importance for the society. This knowledge is vital
as any changes in the current trend shouldn't hurt people's sentiments.
Even in the field of scientific research, knowledge of past results of a
particular finding has guided scientist to take the same research to the
next level. Many advancements in the field of science are derivates of
past developments. Computers, mobile gadgets etc are enhanced
versions of past findings. Similarly, developments in fields of medical are
stimulated by several years of research. Improvements in diagnostic
methods and treatment, eradication of disease like small pox wouldn't
have been possible without the guidance of the study that happened in
the past for the same.
Further, past information helps correct fallacy and gives direction. It
becomes the basis for any progress or invention. Information about an
analogous event from past has helped several research, equipping them
with significant data and results from the past. This information is helpful
as researchers can analyze past results and make necessary changes
for any error, it also makes the process of finding faster by giving it a
right direction.
Hence, past knowledge is vital for any study as it gives insight into the
consequences that happened before, also provides guidance and
direction for future research.

Educational institutions should actively encourage their students to choose


fields of study that will prepare them for lucrative careers.
Write a response in which you discuss your views on the policy and explain
your reasoning for the position you take. In developing and supporting your
position, you should consider the possible consequences of implementing the
policy and explain how these consequences shape your position.

Students should have the freedom to choose, and explore fields of study
that are appealing to them. Thus, the role of educational institutions is to
train students in fields of their choice, and not actively encourage them
to choose fields on the basis of opportunities at lucrative careers. The
reasons for these views are as follows: having a lucrative career
depends on the individual, and not the field of study; also, changes in
government policies and economic stability may render these so called
lucrative jobs unavailable.
First, and foremost, such a statement creates the notion that some fields
of study do not lead students to lucrative careers. Having a successful
career depends on the student, what the student makes of the field, and
how the student effectively applies learning in the classroom to the real
world. There are numerous lucrative areas available within any field of
study. For instance, in Nigerian universities, there is a cachet placed on
courses like medicine, engineering, and law. However, agricultural
students are viewed ludicrously, but many of them are able to start
lucrative poultry, and fish rearing businesses before their counterparts
get jobs.
Furthermore, the continuous availability of these so called lucrative jobs
or careers in particular fields of study is not certain. Changes in
government policies and economic stability may reduce the presence of
these jobs. For example, the effect of the economic recession in
countries like Greece, and Portugal has been a lack of jobs, even high
paying jobs. In addition, in the Nigerian banking sector, the apex bank
implemented a recapitalization policy for banks, these lead to bankers
taking pay-cuts and in other cases been let go. Thus, a banking sector
once considered lucrative is now less benign.
Opponents against my idea argue that educators are helping students
make the right decisions for their future careers. However, students
should make the decision of what kind of career paths or options they
plan to take; if they fancy careers in academic research or want to be
lawyers that take pro bono cases or work in the oil industry, it is their
choice, a career choice should be one were an individual feels fulfilled.
Conclusively, lucrative opportunities are inherent in all fields of study,
educators should help students meet their aspirations and not advice
them to take particular fields of study because the chances at lucrative
careers in such areas are apparent.

Claim: Major policy decisions should always be left to politicians and other
government experts.
Reason: Politicians and other government experts are more informed and
thus have better judgment and perspective than do members of the general
public.
Write a response in which you discuss the extent to which you agree or
disagree with the claim and the reason on which that claim is based.

The assertion that major policy decisions should always be left to


politicians and other government experts is a controversial one. The
author has justified his claim by reasoning that politicians and other
government experts are more informed and thus have better judgement
and perspective than do members of the general public. On one hand, it
is in the best interest of all that major policy decisions should be left to
politicians and other government experts. On the other hand, public
judgement is also crucial in making decisions of policy implementation to
invoke proper transparency in the system. However, in the final analysis,
it is more beneficial is major policy decisions are taken by politicians and
general public mutually.
Firstly, it should be well known that democratic system has replaced the
aristocratic system and individual leaders are bygone by now. The
democratic system is by the people, of the people and for the people.
So, each individual in the society has right to get involved in decisions
which would have fairly good impact on their own future and livelihood.
Also, a great deal of turmoil can be avoided if people are consulted
before announcing any major policy decision taken by government
officials alone.
Secondly, the politicians can also get involved in their personal benefits
by implementing certain policies instead of looking for better good of
large amount of people in the society. The politicians can resort to
partiality in terms of policy decisions with respect to leading economic
companies in the society. For example, in India, some politicians had
given away large amount of land to one big automobile giant TATA
Motors for constructions of the manufacturing camp for much anticipated
TATA Nano car and for harbingering good relations with such a big
market leader. But most of this land given away belonged to poor
farmers whose livelihood depended on it. There was a big public ruckus
regarding this decision and the camp base of that automobile company
had to be shifted to another state resulting in loss of time and money for
people as well company. If people had been consulted before, such
wastage could have been prevented forehand.
Moreover, politicians and other government officials and politicians don’t
have much feel of problems which are faced by people in real life. Their
lavish life doesn’t let them feel the problems of consensus. For example,
people who have undergone a natural disaster like flood or earthquake
know what problems they have actually faced and what problems are
needed to be addressed by the government. In such case, opinion of
common people can prove to be much more beneficial. However, there
exists some section in society which is least interested in participating in
decision make. Also, the decision making process in itself is much more
individualistic. So, instead of wasting time in comprehending such variety
of decisions, expert opinions of politicians and government officials can
come handy.
To sum up, it can be said that politicians and government officials
implementing policies by themselves can be advantageous as well as
disadvantageous. So, proper and smooth functioning of society involves
mutual understanding and decision making by government officials and
public constructively.

Some people believe that universities should require every student to take a
variety of courses outside the student's field of study. Others believe that
universities should not force students to take any courses other than those
that will help prepare them for jobs in their chosen fields.
Write a response in which you discuss which view more closely aligns with
your own position and explain your reasoning for the position you take. In
developing and supporting your position, you should address both of the
views presented.

Curricula are the scenes of constant changes. The materials and the
methods changes= each period. Regarding these changes, there are
those who emphasize on directing students to a variety courses outside
the student’s field of study, and there are those who think a curriculum
should be narrowed down to only core courses of the field which are
preparing the individual for the jobs accordingly.
To begin with, many are those who think that students should only study
the core courses related to their field.Society needs professions, and
professionals are those who are experts in one specific field. For a
society to wait for the fruition of the students’ education, it takes at least
16 years from primary school to graduation in the bachelor degree. As
this period is a significant amount of time, these people believe that
burdening the students with the courses outside their field prolong the
way of becoming specialists in their own fields unjustifiably. Also, they
believe that accumulation of the variety of courses can dilute the focus of
the individual on the main courses. They reason that the students can
learn those fields outside the academia and there is no need to
incorporate them into their core courses.
Conversely, on the other side of the spectrum of opinions people believe
that students should be needed to study multitude courses. An
education, these people believe, is not a mere workshop to just focus on
attaining the skills and knowledge of a specific field. Students need to
study a comprehensive set of courses because of two main reasons.
First, to become a well-rounded individual. Consider the students of
literature, for instance. They need to study history to have an
understanding about their national identity. Also, they need to learn the
arts, as it soothes the characteristics of them. Otherwise, a one-
dimensional student would have not been evolved comprehensively and
intellectually.
Finally, as far as I’m concerned, I agree with the both of views. Having
weighed the two views against each other, we see that both have their
own reasons; we need engineers, doctors, scientists and artists, as we
need them to develop a multidimensional characteristics. To strike a
balance between two views the best way is that we offer the students
variety of courses, and as the level of education becomes more specific,
we then limit the courses to the core courses not to dilute the
concentration of the students on their major field. By this approach nor
we burden them in their becoming professionals, nor do we leave them
in becoming a well-rounded characteristic.
In short, as discussed, we clearly see that both core courses and non-
core academic subjects are needed to develop a student. By a balanced
curriculum, students can have the benefits of the non-core courses and
also focus on the main field.
It is more harmful to compromise one's own beliefs than to adhere to them.
Write a response in which you discuss the extent to which you agree or
disagree with the statement and explain your reasoning for the position you
take. In developing and supporting your position, you should consider ways in
which the statement might or might not hold true and explain how these
considerations shape your position.

Compromising with one's belief implies a person let go off his morals just
because of some petty selfish gains, or he don't have guts to stand up
against what he considers wrong, or he considers it to be simple enough
than opposing and taking some pain.All the above are indications of a
person with low morals and inner strength. Thus, I completely agree with
the author's assertion that it is harmful to compromise with one's belief.I
have substantial reasons to support my position. A few includes:
First, a person will lose one's individuality and self belief.If one work in a
company whose ideals and policies he don't agree with, or he don't
agree with his boss unethical business practices and still don't have guts
to stand against it. That person will lose respect for himself and it's a
vicious circle that keeps on going.
Additionally,it's a loss of integrity, mutual co-operation and reliance. For
instance, what if people like Mahatma Gandhi and Nelson Mandela
would have adjusted with their belief against foreign atrocities just
because it was tough to stick to their own ideals. Nothing good can be
achieved without hard-work, dedication and commitment. People will
lose faith in one another.Furthermore, think of situation if our politicians,
military officials start compromising with their principles.
Some may claim, that it is more harmful to stick with one's
beliefs.Because, they might be pointing at short term benefits such as
sticking to a bad job, giving in to corruption just because it seems
simpler.Ones belief should be flexible, coherent with the time. For
instance, when Galileo came up his heliocentric theory people adhered
to their belief of suncentric theory,which is not at all appropriate. With
changing times may be situations and circumstances change. One
should be open to innovations but it cannot be attributed that since, one
is open to change compromising with one's beliefs is also appropriate.
In the light of above points it can be justified that, compromising with
one's beliefs is a slippery slope.Once you let it go, it's hard to return.
Thus, it can be concluded that compromising with own belief make the
person lose his or her confidence, identity, self respect and individuality
and is thus deleterious in long run.

Claim: Colleges and universities should specify all required courses and
eliminate elective courses in order to provide clear guidance for students.
Reason: College students — like people in general — prefer to follow
directions rather than make their own decisions.
Write a response in which you discuss the extent to which you agree or
disagree with the claim and the reason on which that claim is based.
While colleges and universities are proven to guide students in their
interests along with preparing them for future opportunities, having them
follow one fixed course does more than good. While some sort of
guidance, in the form of required subjects, is needed for the students as
most of the students entering college are not aware of the problems of
the real world, the guidance should not be imposed on the young minds.
Such impositions can only lead to distress. Moreover, claiming that
college students prefer to follow directions rather than making their own
decisions is a very broad statement. Some of the students might prefer
studying subjects that are not necessary but the students have an
interest in those subjects. Courses can't be compiled on the principle of
one size fits all.
When students enter colleges or universities, most of them are unaware
of the challenges that they will face once they are out of college and
have to face the real world. For instance, most of the doctors have to
start operating on actual living people. This requires a carefully regulated
and planned curriculum that has fixed courses for such students so that
they have enough arsenal in order to take these problems head-on.
Thus, there is a need for a fixed set of subjects that all the students must
study.
On the other hand, eliminating all the elective subjects will be a very
extreme step. Elective subjects give the students an opportunity to
pursue their varied interests. All the students are not the same. All of
them have diverse interests. For instance, some engineering students
have an interest in Cloud Computing whereas others may have an
interest in Data Management. Both of these subjects have their own
importance. Thus, forcing a student to study particular subjets can cause
distress among them. It may even lead them to cause interest in the
field. Ergo, elective subjects should not be eliminated.
While this passage propounds a curriculum where some of the subjects
are chosen by the college or the university and some of the subjects are
chosen by the students, the author's reason that college students prefer
to follow directions rather than chasing their own goals couldn't be
further from the truth. A lot of eminent entrepreneurs of the world are
college dropouts e.g. Mark Zuckerburg dropped out of Stanford to build
Facebook. He didn't want to follow the curriculum laid out by the college.
Thus, saying that all college students prefer to follow directions is wrong.
In a nutshell, some form of guidance in the form of fixed courses is
needed for the students so that they are aware of the problems in the
actual world and have the right set of tools to solve them such as
working on a real-life person in the case of medical students. But, they
shouldn't be disallowed from taking electives as elective subjects help
students to follow varied interests and not be bound to the courses set
by the college or the university.
No field of study can advance significantly unless it incorporates knowledge
and experience from outside that field.
Write a response in which you discuss the extent to which you agree or
disagree with the statement and explain your reasoning for the position you
take. In developing and supporting your position, you should consider ways in
which the statement might or might not hold true and explain how these
considerations shape your position.

In this era of rapid technological development, it is very important to


possess knowledge in every other field of study and this is also helpful in
walking hand -in-hand with the rising competition.Individuals are now
evaluated based on the level of knowledge they have in their field of
study as well in other areas of study.The statement made above is
partially in sync with the points mentioned by me. The statement
however does not hold true from the perspective of experience. Hence I
partially agree with the statement.
I concede to the fact that for any study to be complete, one must
possess knowledge from outside that field. One such example would be
the core subjects that we study in our universities. For example, subjects
like Mathematics, Physics and Electronics, are symbiotic. Each subject
requires knowledge of the other to excel in the subject. In order to excel
in Electronics, we need to have sufficient knowledge of Physics to
understand various laws and knowledge of Mathematics to perform
calculations. Hence, failure to understand one subject affects the
knowledge of the other subject automatically.
Now considering the experience point of view, well its not a mandate to
be experienced in another field of study. Mere knowledge of the subjects
outside a field would serve the purpose. For example, a fitness trainer
possesses knowledge about various medical related problems and the
cure for those. The profession is such that, without having knowledge
about the various body mechanisms, reaction to various kinds of
exercises, how to treat a person when he experiences cramps, all these
are of utmost importance and would help a trainer to be complete. But
this does not necessitate the fact that he must possess experience as a
doctor to treat all the medical problems. Mere knowledge about the field
of medical will help him in guiding the people. His knowledge as a fitness
trainer is what comes into consideration when people enroll to the fitness
club.
Another example is of a physician and a specialist. A physician
possesses knowledge of every other field of medicine but in bits and
pieces. This knowledge is sufficient to treat a patient in the initial level.
When the problem does not alleviate, he suggests the patients to visit a
specialist. In order to guide the patients in the initial stages, the
physician need not have an experience in the respective fields.
Thus, it is true that having knowledge from outside a particular field of
study definitely helps in shaping a better future and expands the ken and
makes you more cognizant. But at the same time, it is not mandatory to
have an experience in other areas of study.

True success can be measured primarily in terms of the goals one sets for
oneself.
Write a response in which you discuss the extent to which you agree or
disagree with the statement and explain your reasoning for the position you
take. In developing and supporting your position, you should consider ways in
which the statement might or might not hold true and explain how these
considerations shape your position.

Success in any endeavor is not only determined by the final result, but
the path through which on arrives the result. Goal planning is an
important part of any work. A 'goal' is defined as the deadline that one
must achieve in due course of his work, towards satisfying his ultimate
desire. Also, the final goal is achieved, primarily be setting up short term
goals and striving hard to achieve them. The worth of a success can be
measured only, in terms of the goals that one sets for oneself and not
the acclaim that success brings to a person.
First of all, goals can be divided into two major categories - short term
goals and long term goals. People, often must strive hard, to meet the
short term goals. By splitting up the work, into subtasks, we can easily
complete the final work. Consequently, short term goals, often have
deadlines that have to be met in few days. Thus, the concept of short
term goals motivates the person, and keeps him in track. Because, goals
are set, it is highly unlikely that the person will fall out of track. Thus, this
systematic process, obviously results in success. For instance, freshers
who opt for employment, are often questioned about their short term and
long term goals. By having an idea of what would a person like to be,
five years from that point of time, we can have an idea of the goals of
that person.
Additionally, success can be measured, only by the type of goals one
sets for himself. There are many ways, to attain the same goal. For
instance, if a person wants to become a billionaire, he can do so in
multiple ways: he can loot a bank and become rich in a day; he can toil
for days together and become rich gradually. Hence, the path which a
person takes, in due course of achieving his goal, plays an important
role in measuring its effectiveness. Though, both kinds of people - the
one who toils arduously in a moral way and the one who tries to reach
his goal illegally - achieve success, at the end, it is the nature of goal
that differentiates one person from the other. "True" success is said to
be achieved, only when it is obtained through rightful means.
Moreover, success obtained through illegal means, do not stay with a
person for ever. The person's image slowly tends to get tarnished, as
and when his misdeeds are brought into picture. For instance, if a
politician indulges in corruption and comes to power. Later, if his acts of
corruption, is brought into picture, it will cause serious consequences for
the politican and it will even mar his possibilities of winning in the
forthcoming elections. Only true success, though obtained through
arduous goals, spanning over years of time will remain with the person
forever and speak of his glory.
Setting goals saves time and avoids any kind of deviation that people
can encounter. Because goals are there to be fulfilled at each stage,
people will hardly be distracted and this focus will contribute a lot
towards their success. For instance, if a child aspires to become a
Computer Scientist - he must first complete his schooling with high
scores; then, he ought to do his undergraduate degree in a reputed
institute; after which, he must enroll in one of the top graduate schools,
pursue his research and subsequently fulfill his dreams. At each stage,
he must remain motivated and the final goal must never be forgotten.
Because the path taken is more important to achieve success, setting up
goals, makes us understand the important of hardwork towards success.
Some may argue, that setting up goals, limits our creativity and makes
us live in a mechanized world. But, if no goals are set, people will not
have any idea of what to do; they will just go doing things in a haphazard
manner and end up in nothing. Hence, to streamline our actions, goals
are necessary. If goals are set properly, we can even try to excel in our
desired field with extraordinary skills; if there is time left for leisure, they
can be used constructively in carrying out tasks, that contribute to
achieving our ultimate goal.
Therefore, setting up goals makes us take the right path; saves time;
avoids any kind of distraction and ensures focus; creates more room for
improvement. Goals are those which define our life. Without planning out
goals, nothing can be achieved. "Plan out your work and work out your
plan" goes a saying. If a work is well begun, it is almost half done.
Hence, before plunging into any task, setting up goals, guarantees
success, provided we chart out a plan listing the goals to be achieved at
each point of time.
The general welfare of a nation's people is a better indication of that nation's
greatness than are the achievements of its rulers, artists, or scientists.
Write a response in which you discuss the extent to which you agree or
disagree with the claim. In developing and supporting your position, be sure to
address the most compelling reasons and/or examples that could be used to
challenge your position.

What makes a nation great? I personally disagree with the statement


that the general welfare of its people is a better indicator than the
achievements of its rulers, artists, or scientists. The general welfare can
be a fundamental standard but it is not a sufficient factor of the
judgment.
Some consider the general welfare as a better indication since it is the
primary responsibility of the government. Without people's general
welfare, a nation could hardly be great. No one treat North Korea as
great because millions of its people are still living under starvation while
the nation put most of its finance on its military defense and its so-called
research of nuclear weapons. It is their people that make the country but
the leaders never take care of them. Consequently, the general welfare
is usually our first concern when discuss this issue.
However, if the general welfare of a nation's people is a better indicator,
we can still find that there are some countries whose people actually live
under wealthy welfare while can hardly be considered as a great nation.
For example, Saudi Arabia actually makes large fortunes from its natural
petrol resources and serves its people well. But hardly do we consider it
great because it depends on its natural resources rather than its human
progression. We deny Saudi Arabia as great for its lack of contribution to
the human society--none of scientific breakthroughs and artistic works.
Politicians, scientists and artists, though conceptually only consist such a
little part of a nation, actually make most considerable contribution to the
greatness of a nation. It is these social elites who set up the
constitutional fundamental for United States that guaranteed the political
institution of the great country. We respect the scientific development of
America because its amazing scientific creations significantly promote
and speed up the procession of social development. Most of us are now
enjoying the convenience of life brought by American Internet genius in
silicon-valley.
In brief, the general welfare of a nation's people is a basic factor of the
judgment of a great nation, but it is not sufficient and cannot be claimed
to be a better indicator than the achievements of its social elites.
Greatness can be more than the general condition of people.
The best test of an argument is the argument's ability to convince someone
with an opposing viewpoint.
Write a response in which you discuss the extent to which you agree or
disagree with the statement and explain your reasoning for the position you
take. In developing and supporting your position, you should consider ways in
which the statement might or might not hold true and explain how these
considerations shape your position.

The tested and proven method to gain someone's side is to provide a


convincing argumentation.Undoubtedly, if an argument is able to
convince someone with an opposing viewpoint, it can be considered as
powerful. Providing that the thinking of the person with different opinion
is following the logic. Oftentimes, people's opinions are so ossified, that it
is impossible, even for the most sound argument, to change people's
mind.
Firstly, different religious views are mostly inculcated to people's minds
form their very early childhood. It must be very challenging to convert a
person from one denunciation to another. Moreover, religion is mostly
based on beliefs and feeling rather than facts. Here, the most
reasonable arguments on one side can be completely unsound on
another side.
Secondly, turning to political views, it is very hard to gain a disciple from
another political option. In many countries political opposition finds the
ideas of a leading party incongruous, just because (by definition) they
should be in opposition to them. In such situations, reaching consensus
in any field, is available only when the process of exchanging
argumentation is on very high cultural level. Often, everything ends with
quarrels that leads to lack of agreement. Here, not only argumentation,
but also clever representation of argumentation will have decisive role.
At least, there are numerous techniques that advertising companies use
to gain potential customers and win their approval. It is psychology of an
argument that has important role in the dispute. Leaving the best
argument for last in order for the opponent to remember it, is one of
psychological approaches. Additionally, conceding the argument of
opposition also influence the opposition's inclination to our own views. In
the end who does not like to be praised?
To summarize, failure in convincing someone with an opposing
viewpoint does not necessarily mean that the argument was weak and
poorly reasoned. Some areas are unarguable. It is the way we serve the
argument that matters the most to gain someone's side.

The effectiveness of a country's leaders is best measured by examining the


well-being of that country's citizens.
Write a response in which you discuss the extent to which you agree or
disagree with the statement and explain your reasoning for the position you
take. In developing and supporting your position, you should consider ways in
which the statement might or might not hold true and explain how these
considerations shape your position.
Whether a country's citizens well being should be directly linked with the
efficacy of the leader is a moot question. There are many countries in
the world where it is crystal clear that the citizens are not living well just
because the leaders are not effective in maintaining good relations with
other countries or because these leaders don't have the required
knowledge that is required to run a nation which makes them quite
ineffective. While, in general, the efficacy of the leaders has an impact
on its citizens, it is also the case that there are many other countries
where the citizens of the country believe that their leaders are effective
but still there are a lot of people who are living in a penurious state in
those countries.
In countries like North Korea, where dictatorship is the form of
government, most of the people are indigent. They don't have enough
food to eat. Their children are dying of starvation and the only thing
responsible for this is the ineffectiveness of its leaders. The supreme
leader of North Korea doesn't have good relations with most of the
countries. Instead of providing food to the people of the country, he is
focussing more on the military arsenal and wants to prove to the world
that his nation is an exemplar of a perfect nation. Because of this most of
the countries don't have good relations with North Korea and have
imposed sanctions. Thus, due to the ineffectiveness of the leader of
North Korea, the citizens of North Korea are living in a pitiful state.
On the other hand, there are some countries, where the leaders of the
country are believed to be effective but there is a mix of both classes of
people: those who are living luxurious lives and those who are living
below the poverty line. For instance, in India, there are a lot of people
who think that the current leader is a very effective leader. He has
maintained good relations with other countries. He is focussing on
Foreign Direct Investment to help the citizens of the country and provide
them with jobs so that they can provide for themselves and their families.
Still, there are more people in India, who go to sleep without a meal than
the number of people who are benefitting from these policies.
While the essay propounds that leaders effectiveness is linked to the
well being of the citizens in some sense but not completely, some might
argue that if the leader is so effective, then he should be able to take
care of all the people of the country by making policies that benefit all of
them. But, there is no silver bullet that can resolve all the issues of a
country's citizens. Even if an effective leader makes policies that help a
cohort of the nation, it is highly likely that it may not be beneficial to a
niche of the nation. For instance, the introduction of GST in India helped
a lot of middle class and upper-class businessmen of the country, the
lower class and SMEs were burdened because now they had to hire
CAs to help them file taxes.
Does the efficacy of a leader directly impact the well being of the
citizens? The answer in some sense is yes. Ineffectiveness of a leader
can be seen in some countries where the citizens are living in an
indigent state. However, there are some countries where the leaders are
effective and they try to help the people of the country, there are still lots
of people who are living poorly.
Nations should pass laws to preserve any remaining wilderness areas in their
natural state.
Write a response in which you discuss the extent to which you agree or
disagree with the claim. In developing and supporting your position, be sure to
address the most compelling reasons and/or examples that could be used to
challenge your position.
While the natural environments are seriously devastated and more and
more narrowed by human activities, a strong mean is needed to protect
ecological diversity. Although several non-goverment organizations and
environmentalist takes their actions, it is likely their achievements are
overwhelmed by the speed of environmental destruction. Thus I totally
agree with the passing of laws to preserve any remaining wilderness
areas in their natural state. This visionary protection not only helps men
correct their mistakes in the past, but also limits our nature-damaging
activities at the present and benefits the future of human kinds.
During the twentieth century, we have witnessed the significant decrease
of ecological diversity due to the human needs of natural resources and
expansion. As a result, several species, both animal and plant, became
completely extinct and many others are seriously threatened regardless
of the environment-saving efforts. The effective way to protect the
endengered species, therefore, is to keep the natural environments out
of the human presence by forming the preservation of any remaining
wilderness which.
Besides, many business companies running after profits contribute much
to the damages to nature. Although there are interferences by non-profit
environmental organizations, these efforts may be obscured by money
and power, typically in developing countries. Thus it is crucial to address
the problem at national level, which put the remaining natural areas
under the protection of law.
Finally, this policy will recover the ecological diversity which benefits our
future generation. It is no dout that we are paying the penalty for the
environmental damages by suffering from polution, climate changes and
other natural disasters. By preserving the remaining natural areas, the
ecological balance will be re-established according to the law of natural
development. It is expected that the global environment will be returned
to its integrity and our future will not suffer from the consequences of our
own activities.
In conclusion, it is essential to address the environmental issue by
passing the law to protect any remaining natural areas in our planet. It is
not only the way we save the damaged environments but also the
effective mean to limit the harmful human activities and to recover the
green planet for our future.

In any field — business, politics, education, government — those in power


should be required to step down after five years.
Write a response in which you discuss your views on the policy and explain
your reasoning for the position you take. In developing and supporting your
position, you should consider the possible consequences of implementing the
policy and explain how these consequences shape your position.

The young generation is more and more proving their excellent ability in
a wide variety of fields. Thus it is not surprised that the old one is
yielding their places to young people after a time. When and in what
context they should do that, however, is the question. In my opinion, it is
inflexible and unreasonable to pass the policy that those in power should
step down after five years for several reasons, among which are the
matter of time, experiences and knowledge.
First, five years is not a long period of time for a person to be in power. It
is, conversely, just sufficient time for him to be mature in his new
position. When one is promoted, he needs a time to adapt, learn,
improve himself and then fully becomes a leader with adequate skills. In
many cases, five years is just enough time for a new leader’s changes or
policies proved to be effective. If he is replaced immediately at the point
of his career maturity, it will be a waste of time and effort to the company
to find someone new. Bill Clinton, George Bush or Barack Obama, for
example, were the U.S. Presidents who won the election again for their
recognized leadership after one tenure of five years. This effectively
demonstrates that the short time is not necessarily a sufficient period of
time in a politics.
Second, somebody who has proved his excellent knowledge and skills in
leading an organization is deserved to hold power more than five years.
It is not easy to choose a leader, and even far more difficult to have
talented one. While we recognize that our company is prospering and
obtaining more and more profits, or reaching closer to its purpose, the
leader’s ability has been attested and is it a good idea to replace him by
another one? Certainly not, as nobody ensures that the new leader is
talented enough to make the same success as the old one. Bill Gate and
Steve Jobs, for example, held the CEO position for far more than five
years. But with their innovations that changed the whole world, I do not
think they should be replaced by anyone else.
Last, although new knowledge and technologies are highly regarded and
usually introduced to change old ideas, experiences are sometimes
overwelming. In these cases, the person in core position, and certainly
having power, should be retained for an enough time to give traing to the
young generation. This is the so-called transition period and that it is less
or more than five years is essentially dependent on specific contexts,
one of which is the concern if young people are sufficiently qualified for
new position. If not, the old leader should still be in power, the transition
time may be longer and certainly cannot determined by a specific
interval of time.
In conclusion, the world is flexible enough for us not to use a number of
years to designate a person in power. It essencially depends on a range
of reasons and particular cases, for which a person should be replaced
or not.

Some people claim that the goal of politics should be the pursuit of an ideal.
Others argue that the goal should be finding common ground and reaching
reasonable consensus.
Write a response in which you discuss which view more closely aligns with
your own position and explain your reasoning for the position you take. In
developing and supporting your position, you should address both of the
views presented.
Politics being one of the most fundamental and exacting leadership
disciplines, yet debated, is the crucial necessity for any country. There
has been reforms and changes in the ways politics is being practised
across the world. Almost all political parties and its leaders, as well as
members, have their ideology and basic principles to which they are
abided. The above prompt suggests two views of people: some claim
politics should be targeted to pursuit the ideal, whereas others argue that
the goal should be to find common ground and reach reasonable
consensus. In my opinion, I strongly agree with the latter and argue to as
its proponent for three reasons.
Its leaders and members may entirely accept the ideology of a particular
political party, but it may severely contrast with that of other parties and
associates. The political ideology of Republic, Democratic, and Green
Parties in the U.S., or Communists and Democratic parties in Nepal -- all
have different principles and ideology, and in some cases have differing
beliefs. If the party in the government intends to pursue its ideal, then
definitely it is not accepted by all citizens and remaining parties. This
may elicit fights within the house of parliament, as in Japan some years
back, riots and protests.
Finding common ground and reaching logical consensus is the
indispensable need for all political leaders of any country to have mutual
understanding and to peddle forward. A country is not a personal
belonging of a particular party of a specific ideology. It is a melting pot of
people of different views and ideals. In such circumstances, only if the
politics intend to have common understanding and consent in any public
work or bill in the parliament, it will create an amicable and pragmatic
environment in the country. Else, the chances are such that people will
start to voice against the ruling government, scath its way handling the
country, which may bring riots, violence and instability in the country.
Being a zealot or partisan with a particular ideal of a political party brings
extreme awful consequences and chances are that the regime becomes
tyrannical. For example, in North Korea, there is only one ruling party,
and all the citizens are required to abide by its principle and ideology.
Any different opinion from a citizen is taken as bigotry. Consequently,
neither the citizen have been able to practice freedom of speech nor
able to do anything according to their free will due to stringent rules of
the ruling party. This has made North Korea’s ruling image as a
dictatorship, and many countries have united against it.
Ideals of political parties are not necessarily similar. They are different
and also may contrast severely. Hence, only reaching the common
ground to act and finding the common consensus should be the goal for
the betterment and win-win situation of all parties and citizen.
The best way to solve environmental problems caused by consumer-
generated waste is for towns and cities to impose strict limits on the amount of
trash they will accept from each household.
Write a response in which you discuss the extent to which you agree or
disagree with the claim. In developing and supporting your position, be sure to
address the most compelling reasons and/or examples that could be used to
challenge your position.

The claim outlines the method to be implemented to reduce


environmental problems caused by consumer-generated waste. The
waste generated by customers range from plastic products to vegetables
and fruits. The claim suggests that towns and cities should impose strict
limits on the amount of garbage collected from each household. The
claim provides an extreme solution for controlling customer-generated
waste. The limiting of waste generated would restrict the activities of the
citizens thereby, resulting in illegal disposing of waste. Instead of
imposing a limit, encouragement of recyclable products would in turn
result into the same benefit of environmental protection without the
added cost of public dissatisfaction.
Products, which can be recycled do not damage the environment and
would reduce the overall cost of disposal of waste. Even with the
restriction on waste, pernicious and indecomposable waste ,would still
be obtained in a slightly less quantity. The use of recyclable products,
would help in overall reduction of harmful customer generated waste.
The limiting of waste, would result into people having to limit their waste
generation, which after a limit would be difficult to maintain. Resulting in
the illegal disposal of waste in quiescent places or storage of waste
leading to unhygienic environment.
For example, Sweden has incorporated the use of recyclable products
for majority of public products resulting in a holistic reduction in
generated waste. The country has such an efficient recycling mechanism
that they generate less amount of waste compared to the quantity they
can recycle. The model implemented by Sweden follows a snowball
effect, reduction of products using environment harming substances
results in less production by companies of these products. Reduction of
customer generated harmful waste and waste generated by companies
reduces the overall impact on the environment. There are many other
countries trying to implement the same model as Sweden. If Sweden
had tried to implement the model of strict limiting in waste, it would have
resulted in slight reduction in customer generated waste if it was
successful. The implementation of recyclable product use has resulted in
reduction in both customer generated and industry generated wastes.
Alternatively, some might suggest that implementing of recyclable
product use, would result in increase in cost for companies and
customers. For some products, the implementation of recyclable
materials, would require change in factory machinery and thereby,
increase product cost. The reason is warranted, but considering the
majority of products for household customers, most are available in
recyclable variants. The product cost would increase but only for a
certain section of products such as plastic toys, polythene bags and
other indecomposable products. But majority of products such as
wrappers of edibles, glass wares, etc. can easily be replaced.
Nevertheless, initial cost would have to be incurred to implement the
model of use of recyclable products.
In conclusion, The implementation of the claim, would result in
dissatisfaction among the citizens and would not create a major impact
on reducing impact on environment. On the other hand, the use of
recyclable products helps to reduce the quantity of total waste generated
by both individuals of a town or city and industries. To reiterate, recycling
and reusing different types of products can seriously reduce the impact
of humans on the environment.`

We learn our most valuable lessons in life from struggling with our limitations
rather than from enjoying our successes.
Write a response in which you discuss the extent to which you agree or
disagree with the claim. In developing and supporting your position, be sure to
address the most compelling reasons and/or examples that could be used to
challenge your position.

Many have argued that success can not be enjoyed with out the turmoil
that struggles bring. The question as to whether the most valuable
lessons are learned in life from struggling or enjoying success is deep
rooted and complex. During the struggle to succeed, a person can learn
a lot about themselves. Limitations become clear and the ability to
overcome those limitations can augment a person's feeling of self worth
and accomplishment. Therefore, I believe that more can be learned from
the struggle than from enjoying success.
It is during a struggle that a person is faced with the most challenges. In
the act of overcoming these challenges, a lot can be revealed about a
person and their character. For example, Liz Hartel was raised in an
affluent family and begun horseback riding at an early age. She was very
successful in her professional dressage career and did not face many
barriers to success until she was diagnosed with polio. Liz became so
weak that it was impossible for her to hold herself upright or walk on her
own. Determined to return to her horseback riding career, she worked
intensively day in and day out. Eventually, against all odds, Liz was able
to compete in dressage again. Not only did Liz return to dressage after
her paralysis, she also won numerous awards and served as a role
model and inspiration to other young people with polio. Although Liz was
extraordinary prior to her diagnosis, it was not until the struggle with
polio that her admirable character was uncovered. Liz Hartel's struggle
has many more valuable lessons to offer people around the world than
does her easily obtained success in dressage.
Additionally, it is during times of struggle that heroes most often emerge.
Heroes such as Martin Luther King, who until this day is renowned for
his unparalleled bravery and pristine moral character. Martin Luther King
was a preacher prior to the travesties of slavery and segregation. While
his life as a pastor did offer some life lessons, it was the struggle for
equality that made Martin Luther King's character clear and that taught
the entire nation lessons about equality and peace. The struggle
endured by Mr. King offered valuable lessons about heroism and
standing up for your beliefs, no matter what the cost. Lessons of heroism
and valor can not be learned during unchallenging times of success.
During the revolutionary war, the American colonies struggled for their
freedom from British oppression. It was during the revolution that the
worth of our great nation was realized. In the absence of such struggles
greatness can not be realized and the most valuable lessons can not be
learned. If it was not the revolutionary war, America would not be what it
is today; it was during that struggle that American values were
established. Such revolutions teach the value of fair treatment and
independence; values that can not be learned during times of success.
In conclusion, without facing and overcoming limitations, the most
important parts of our character can not be realized and the most
valuable lessons can not be learned. When reminiscing, it is apparent
that it was the the revolutions and great achievements in history that are
rich in life lessons, not the moments of prosperity. Therefore, I firmly
believe that the most valuable lessons in life stem from struggling with
our limitations and not enjoying successes.
.
Claim: While boredom is often expressed with a sense of self-satisfaction, it
should really be a source of embarrassment.
Reason: Boredom arises from a lack of imagination and self-motivation.
Write a response in which you discuss the extent to which you agree or
disagree with the claim and the reason on which that claim is based.

This topic raises the contentious issue that boredom arises from a lack
of imagination and self-motivation. Also, the opening statement claims
that though boredom is often expressed with a sense of self-satisfaction,
it should really be a source of embarrassment. Indisputably, there are
phases in an individual's life where a lack of imagination and self-
motivation to find the next big thing of their growth trajectory creeps in.
Perhaps one of the reasons for this complacency is a feeling developed
due to the abundance of advancements in technology with multiple
products available for each service unlike the past. In short such level of
convenience inhibits many individuals to try something new and leave it
to the rest of the world to solve.
Products like Whats-app for instant communication, E-commerce
platforms like Amazon.com for online shopping and none other than
Google for discovery of the most unthinkable at the click of a button.
Nevertheless, every individual should strive to improve and excel, and
make life meaningful with whatever the resources available besides
being apathetic to the embarrassment which author thinks by and large
for being bored. Thus, I generally disagree with the opinion that boredom
is a source of embarrassment and would argue that it arises from a lack
of imagination and self-motivation.
Firstly, boredom arises in an individual largely when a task is done with a
sense of lack of purpose over a prolonged duration mostly. Imagination
takes back seat when one turns apathetic to the role they are playing. I
would like to point that not all people are self-motivated till the time they
find a meaning in the job they have to accomplish. It is human nature
that one wants to succeed and get approbation for the same. For
example - a batsman might feel bored due to a change in its batting
order, he/she may be performing outstandingly at one position but the
change in position may result in an ephemeral boredom. The player can
still definitely imagine for his contribution at that position and feel
motivated despite the transition from the coveted position.
Secondly, an individual might feel self-satisfied at some age and be
bored due to a lack of social circle as well. For example - a retired officer
may feel self-satisfied for his decades long service, must have displayed
all sorts of imagination and motivation at work but no more feels the
need to do it. It is no way that he should feel embarrassed and his
boredom is quiet justified due to a lack of social circle which perhaps
might exist for other reasons. People often feel better alone till the time
they find congenital peers with whom they can be candid and do away
with their monotony in their style or preference. Hence, these evidences
demonstrate that lack of imagination and self-motivation can still co-exist
without boredom.
Today's competitive world seems to be heading in a direction where the
survival of the fittest looks apt. Admittedly,a constant innovation,
unparalleled motivation and the courage to dare the unthinkable is
required to sustain and grow at-least in a few trades. For example - It is
inevitable for a creative director of an advertising company to stay
abreast in all these arenas. Their is no room for boredom, amidst the
jungle of creative thinking, as one learns and unlearns at an
unprecedented pace. However, the above argument does not constitute
a sufficient claim that dearth in imagination and self-motivation begets
boredom. Right to live is a fundamental right of each human and one
shouldn't feel embarrassed for his/her choices that suits his/her desired
needs. Both a teacher and a scientist find their ways of imagination in
their respective professions and stay motivated. There are phases where
they revitalize themselves by taking breaks and bring forth new ideas.
To conclude, although the world will embrace success marked by
constant momentum of innovations and an unending expectations of a
hyper-competitive race. To some, expression of boredom as a self-
satisfaction might appear a source of embarrassment but that does not
hold true in all cases. There is a diminishing marginal utility for each
activity as per economics which leads to phase of a lack of imagination
and self-motivation for every individual. I disagree partially basis these
evidences shared that lack of imagination and motivation account for
boredom. There is an opportunity cost of being motivated and full of
imaginative ideas every-time which not all are ready to pay and they
better feel content to live with their peace.

Some people believe that the most important qualities of an effective teacher
are understanding and empathy. Others believe that it is more important for
teachers to be rigorous and demanding in their expectations for students.
Write a response in which you discuss which view more closely aligns with
your own position and explain your reasoning for the position you take. In
developing and supporting your position, you should address both of the
views presented.

The role a teacher plays in building the future of his or her students is
crucial. By certain views, this role is even more pivotal than that of the
parents fo the students. For this reason, it is fundamental to discuss
what are the best means for teachers to achieve the best possible
preparation for their students. In particular, there is a hot debate about
whether it is more important for a teacher to be understanding or, on the
contrary, to be rigorous and demanding. It is the case to spend some
words about what truly is the role of a teacher, namely to guide and
motivate students throughout their studies, and what the students
themselves need to get to their goals. This way, it will be evident that
teachers must be first of all empathic, and, only after this, also rigorous.
How can we specifically mark some qualities as the most important for
an educational leader? That is, what is that teachers mean for students,
what is their true role? It seems legit to talk widely and to consider every
aspect of the preparation of a student: by this point of view, teacher have
to teach how to love the entire otherwise-stressing studying process.
Simone Weil, a French writer, expressed this beautifully by the sentence:
"The joy of learning is as essential to students as breath to runners". In
fact, to effectively succeed in our studies, what we basically need is,
before anything else, passion. Hence, the qualities of the witty teacher:
empathy, enough to build up a potive relationship with his students and
be able to push them toward a passionated and loving studying activity.
This is, incontrovertibly, the main feature of a great teacher.
On the other hand, it is also undoubted that students are, in particular in
the earliest stages of their study careers, demanding a bit of authority. It
is not something so misterious or mythical: everyone knows very well
that when his teacher demands little or nothing, no study is really
requested. In other words, people need to be challengedby the school
system and even a bit scared by their teacher. As a matter of fact,
indeed, people who are not expected to study a lot ar not going to study
as much as would be correct to. The reason is simple: studying is a
tough process, a demanding activity, almost a sacrifice. Thus, no good
teacher can really exist if no rigour and authority are presented in front of
the students.
Nonetheless, it seems to me that the above presented views are not
totally mutually exclusive. If we closely look at both the positions on the
issue, we can recognize that they are in fact not that much of a
dicotomy. Why would be a teacher that is, at the same time, rigorous
and empathic, or highly expecting and understanding? As "the best is in
the middle", like ancient Latin philosophers recomended, it sounds
reasonable to infer from what has been analyzed in the above
paragraphs that a teacher has to be both emphatic and distant. The
instance that can prove this is quite simple: imagine a teacher who is
capable of capturing the trust of his students, to almost make them feel
comfortable with them. Well, imagine now that the same teacher looks
with askance to one of his students: the latter would be furious with
himself, because he disappointed his trust-worthy teacher. This is, of
course, the best possible teaching system. As base of everything,
anyway, there are again positive and nice attitudes of the teacher toward
the students.
To conclude, it seems wise to deem empathy and apprehensiveness as
the most fundamental features of the perfect teacher. In fact, to truly
strenghten and engender the pleasure of study in their students,
teachers need to make them love what they do. This notwithstanding,
some rigorousness and slightly high expectations do also have their
positive effects on students, as far as these attitudes do not overcome
the above mentioned virtues.

ARGUMENT TOPICS:
The following appeared in a memorandum from the manager of WWAC radio
station.
"To reverse a decline in listener numbers, our owners have decided that
WWAC must change from its current rock-music format. The decline has
occurred despite population growth in our listening area, but that growth has
resulted mainly from people moving here after their retirement. We must make
listeners of these new residents. We could switch to a music format tailored to
their tastes, but a continuing decline in local sales of recorded music suggests
limited interest in music. Instead we should change to a news and talk format,
a form of radio that is increasingly popular in our area."
Write a response in which you discuss what specific evidence is needed to
evaluate the argument and explain how the evidence would weaken or
strengthen the argument.

In his argument, the author's recommends that inorder to reverse the


trend of decline in its listener numbers, WWAC radio station needs to
switch to news and talk format from its already existing rock-music
format. To support his argument, the author has stated that there has
been a decline in the number of listeners of the WWAC radio station
espite population growth in its listening area. The author also cites that a
decline in local music sales suggests a decreasing interest in music. The
author has pointed out that the news and talk format is a popular radio
format in the concerned area, so it would be profitable for WWAC radio
to switch to this format. However, the author has made some
assumptions for which there is no supporting evidence.
Firstly, the author assumes that a decrease in the listener numbers for
WWAC radio station which specialises in rock-format proves that people
in the listener area, are losing interest in rock-format music. The author
remains silent regarding the trend of listenere numbers of other radio
channels which specialise in rock-format in the same listener area. The
real reason for decline of listner numers of WWAC may be the
decreasing quality of the rock music provided by them, while people may
be preferring other rock-format radio channels or internet channels which
provide better quality music. If the author were to state conclusively that
either WWAC radio station is the only major radio channel in the area
which provides rock-format or all the radio stations providing rock-format
in the area have reported a decline in number of listners, it would have
provided considerable suport to his position.
Secondly, the author has assumed that there is a general trend showing
a lack of interest in music. To support his assumption he has pointed out
that there has been a decline in local sales of recorded music. However,
he has not quoted any definite figure or survey to support his
assumption. If the trend of decline of salesa is very recent and
percentage small, the statistical data will be insufficient to draw any
reasonable conclusion. The people of the listener area may have
resorted to other means of procuring music. For example people may be
going for online purchase of new albums to listen on their gadgets like i-
pods rather than going to local stores for buying music records. The
author has not examined any such alternative reason which could be
responsible for decline in local sales of recorded music.
Thirdly, the author has asumed that a change to news and talk format
will be profitable for the WWAC radio station based as acording to him
this is a popular format in the listener area. Here, the author has again
failed to provide any reliable statistical data to prove that news and talk
is indeed the preferred radio format in the listerner area. He has also
failed to examine some other important areas. For example, in the
WWAC listner area there already may be bigger players in news and talk
format who have a much larger listner base. Consequently, any new
player in news and talk format may have to face a lot of stif competition
inorder to make a name for it. In such a case, switching to news and talk
format may incur losses for WWAC radio station.
Summing up, the author may be correct in his recommendation that
inorder to reverse the trend of decline in its listener numbers, WWAC
radio station needs to siwtch to news and talk format from its already
existing rock-music format. However, in arriving at his conclusion, the
author has made some assumptions and has not examined many other
key areas which could affect his recommendation. If the author's
recommendation is to have the desired effect, he needs to provide
suprting evidence for his assumptions and examine all the other key
areas.

The following appeared in an article written by Dr. Karp, an anthropologist.


"Twenty years ago, Dr. Field, a noted anthropologist, visited the island of
Tertia and concluded from his observations that children in Tertia were reared
by an entire village rather than by their own biological parents. However, my
recent interviews with children living in the group of islands that includes
Tertia show that these children spend much more time talking about their
biological parents than about other adults in the village. This research of mine
proves that Dr. Field's conclusion about Tertian village culture is invalid and
thus that the observation-centered approach to studying cultures is invalid as
well. The interview-centered method that my team of graduate students is
currently using in Tertia will establish a much more accurate understanding of
child-rearing traditions there and in other island cultures."
Write a response in which you discuss what specific evidence is needed to
evaluate the argument and explain how the evidence would weaken or
strengthen the argument.
The author of this passage states that an interview-centered method of
studying child-rearing traditions is much more accurate than the
observation-centered method used by Dr. Field in his study of the
children of Tertia. In order to do this, the author, Dr. Karp builds up the
argument based on the recent interviews conducted with the children of
Tertia. However, the arguments laid down by Dr. Karp are not conclusive
enough. His argument is not properly substantiated and in order to reach
the conclusion given by Dr. Karp, a lot more evidence. Only then can this
argument be properly evaluated.
Firstly, Dr. Karp mentions that in his interviews with the children of
Tertia, he saw that these children talked mostly about their biological
parents instead of other adults of the village. However, this statement
alone is not enough to corroborate that the children were not raised by
the entire village. In order to prove his hypothesis, the author must
provide evidence that talking about one's biological parents insinuates
that one was reared only by his biological parents and not by the entire
village. If the author cannot provide such evidence, then this whole
argument falls apart.
Secondly, no information is provided regarding the number of children
interviewed. Even if we assume that talking about one's parents implies
that one is raised by one's parents and not by the entire village, it's still
not clear if these results will hold true for all the children. If the number of
children interviewed was very small, then it might be possible that those
children were raised by their biological parents but this doesn't mean that
all the children of Tertia were raised by their biological parents. The
author extrapolates the interview results further to say that these will
provide an accurate understanding of child rearing traditions in other
cultures as well. The author needs to provide whether this result can be
used for the entire population of Tertia and other islands as well. In case
the author is not able to provide evidence bolstering this claim, it cannot
be concluded that interview-centered approach is better than
observation-ceneterd approach.
However, if we assume that the author can provide evidence supporting
the claims made, is it true to say that one failure refutes a theory? It is
entirely plausible that the children of village of Tertia act an outlier to
observation-centered theory. The author needs to provide evidence that
this failure successfully refutes the observation-centered theory. Also, it
might be the case that Dr. Karp is using this opportunity to denigrate Dr.
Field. Only if the author can provide evidence against these arguments
can he/she conclusively say that interview-centered approach of
studying child rearing traditions is better than observation-centered
approach.
It is not implausible that interview-centered approach is more accurate
than observation-centered approach but the argument and evidence
provided by the author is rife with loopholes. More evidence is needed to
prove this hypothesis. Whether talking about parents implies that the
children were reared by their parents only is a moot point. More
evidence is needed to prove that the study conducted by Dr. Karp is
enough to extrapolate the results for the entire population. Also, Dr. Karp
needs to substantiate his claim with more evidence so that it can be
proved that one failure of the observation-centered approach implies that
interview-centered approach is much more accurate. Without all this, the
author cannot conclusively prove that his approach is better than Dr.
Field's approach.

According to a recent report, cheating among college and university students


is on the rise. However, Groveton College has successfully reduced student
cheating by adopting an honor code, which calls for students to agree not to
cheat in their academic endeavors and to notify a faculty member if they
suspect that others have cheated. Groveton's honor code replaced a system
in which teachers closely monitored students; under that system, teachers
reported an average of thirty cases of cheating per year. In the first year the
honor code was in place, students reported twenty-one cases of cheating; five
years later, this figure had dropped to fourteen. Moreover, in a recent survey,
a majority of Groveton students said that they would be less likely to cheat
with an honor code in place than without. Thus, all colleges and universities
should adopt honor codes similar to Groveton's in order to decrease cheating
among students.
Write a response in which you discuss what questions would need to be
answered in order to decide whether the recommendation and the argument
on which it is based are reasonable. Be sure to explain how the answers to
these questions would help to evaluate the recommendation.

The author recommends a measure to reduce the number of cheating


cases among students which has been on the rise in recent times. He
urges all the colleges and universities to adopt honor code smililar to that
of Groveton College in order to decrease the forgery among students.
He may be correct and the implementation might prove fruitful, however
there are certain questions that need to be answered to figure out the
success of the implementation.
The very first question that will is arise is regarding the genuine and trust
worthy nature of the students. Where is the assurance of the students
not tending to cheat even though they have agreed not to do so. Even
doctors and police officers take the pledge of not getting diverted from
their main task, however some of them do get corrupted. A similar
situation may be applied on the students too. By adopting a honor code,
a student will have the power to notify the faculty member about any
copying case. Now this reporting might be true most of the times,
nevertheless a student can take an advantage of the same and report a
false complaint just to put his class mate in trouble. This might be due to
any sort of past issue which he might be having with that candidate.

Also, the report quotes certain statistics to show how effective the
implementation of the honor code has been. The author says that in the
absence of the honor code, thirty cases were reported, in the first year of
implementing it, the figure went down to twenty one and at the end of
five years it was fourteen. Such a kind of data would had been reliable if
the total number of students would have been mentioned. Moreover,
isn’t it possible that the students must have formed groups and each
group would copy among themselves without reporting the matter to the
higher authority? Also, in a span of five years, the reputation of the
institute might have gone down and the number of candidates taking
admission must have reduced. As a result, the question regarding the
total number of students prior to the implementation, in the first year of
implementation and also in the fifth year of honor code must be
mentioned.
Finally from the conclusion one can easily question regarding the
similarity between Groveton’s College and other universities and
colleges. It is possible that the suggested measure could help in curbing
the number of cheating cases only if various aspects like number of
copying cases, sincerity of the students, total number of students
enrolling, the command of the institute over the students, etc of the
various institutions match with Groveton’s. In a school where students
are not at all emotional and don’t realize the importance of an honor
code, such a measure could turn out to be a disaster. Moreover, the
heads of the other institutes must also welcome this idea of adopting an
honor code. It is only then we can try to figure out if the recommended
measure will be successful or not.
Thus, the answers of the above question need to be answered in order
to evaluate the recommendations. However, the report fails to answer
these questions and as a result, the out come of the suggestion is
dubious and as of now the authenticity of the argument falls flat.

The following appeared in an article written by Dr. Karp, an anthropologist.


"Twenty years ago, Dr. Field, a noted anthropologist, visited the island of
Tertia and concluded from his observations that children in Tertia were reared
by an entire village rather than by their own biological parents. However, my
recent interviews with children living in the group of islands that includes
Tertia show that these children spend much more time talking about their
biological parents than about other adults in the village. This research of mine
proves that Dr. Field's conclusion about Tertian village culture is invalid and
thus that the observation-centered approach to studying cultures is invalid as
well. The interview-centered method that my team of graduate students is
currently using in Tertia will establish a much more accurate understanding of
child-rearing traditions there and in other island cultures."
Write a response in which you examine the stated and/or unstated
assumptions of the argument. Be sure to explain how the argument depends
on these assumptions and what the implications are for the argument if the
assumptions prove unwarranted.

It is not implausible that interview-centered approach is more accurate


than observation-centered approach but the argument and evidence
provided by the author is rife with loopholes. More evidence is needed to
prove this hypothesis. Whether talking about parents implies that the
children were reared by their parents only is a moot point. More
evidence is needed to prove that the study conducted by Dr. Karp is
enough to extrapolate the results for the entire population. Also, Dr. Karp
needs to substantiate his claim with more evidence so that it can be
proved that one failure of the observation-centered approach implies that
interview-centered approach is much more accurate. Without all this, the
author cannot conclusively prove that his approach is better than Dr.
Field's approach.

A recently issued twenty-year study on headaches suffered by the residents of


Mentia investigated the possible therapeutic effect of consuming salicylates.
Salicylates are members of the same chemical family as aspirin, a medicine
used to treat headaches. Although many foods are naturally rich in salicylates,
food-processing companies also add salicylates to foods as preservatives.
The twenty-year study found a correlation between the rise in the commercial
use of salicylates and a steady decline in the average number of headaches
reported by study participants. At the time when the study concluded, food-
processing companies had just discovered that salicylates can also be used
as flavor additives for foods, and, as a result, many companies plan to do so.
Based on these study results, some health experts predict that residents of
Mentia will suffer even fewer headaches in the future.
Write a response in which you discuss what questions would need to be
answered in order to decide whether the prediction and the argument on
which it is based are reasonable. Be sure to explain how the answers to these
questions would help to evaluate the prediction.
In the given passage, the outcomes of the investigations, which have
been done on headaches suffered by the residents of Mentia, are
presented. The consequences of consumption of salicylates are
mentioned. The food companies are using salicylates as preservatives
and flavor additives. It is reported that due to rise in use of salicylates,
there is a decrease in average number of headaches. At the end, health
experts have predicted that in future average number of headaches will
further decline.
It is stated that salicylates chemical is helpful to treat headache
problems. But the side effects of this chemical on human health is not
reported. It would have been possible that although it can cure the
headache problem, it can be harmful for other body parts. Food
companies are using salicylates as preservatives. What are the other
attributes of this salicylates?, are these preservative foods are safe to
use or not, for how long we can use it, these type of questionable
premises should be answered clearly to strengthen the argument.
The correlation between the rise in the commercial use of salicylates and
a steady decline in the average number of headaches is presented by
the author. Here statics is not given to justify the facts. The food
companies have found the another use of salicylates as flavor additives
for foods. Is artificial flavor additives is not harmful for health, what are
their features, in which type of food they can be used, such questions
should be answered to justify the argument.
The food companies are planning to use salicylates in future as flavor
additives. Based on this information health experts have predicted that
by the use of these food Mentia will suffer even fewer headaches in the
future. How many companies are going to use salicylates as flavor
additives and in which type of food, and how many people are going to
use that food, these all information should be mentioned to make the
prediction reasonable.
As conclusion, the effect of consumption of salicylates on human body,
its safe limit which can human body can take, should be stated. More
detail survey of food companies and their products which are using
salicylates, should be done in order to make argument more stronger. To
support the prediction, more justification about the statics of amount and
consumption of salicylates should be provided.

The following was written as a part of an application for a small-business loan


by a group of developers in the city of Monroe.
"A jazz music club in Monroe would be a tremendously profitable enterprise.
Currently, the nearest jazz club is 65 miles away; thus, the proposed new jazz
club in Monroe, the C-Note, would have the local market all to itself. Plus, jazz
is extremely popular in Monroe: over 100,000 people attended Monroe's
annual jazz festival last summer; several well-known jazz musicians live in
Monroe; and the highest-rated radio program in Monroe is 'Jazz Nightly,'
which airs every weeknight at 7 P.M. Finally, a nationwide study indicates that
the typical jazz fan spends close to $1,000 per year on jazz entertainment."
Write a response in which you discuss what specific evidence is needed to
evaluate the argument and explain how the evidence would weaken or
strengthen the argument.

The author argues that the proposed new jazz club in the city of Monroe
would be successful business idea because there’s no club nearby in the
city. Several musicians live in the city; and jazz festival last summer
were successful as more than 100000 people visited. However, in the
evidences provided by the author, there are several flaws, assumptions,
and illogical rationales he made to support the arguments. In order to
strengthen the arguments, he should go through several discussions
below.
First, the author argues that the proposed new jazz club will be
successful because of the successful jazz festival last summer in which
over 100,000 people visited. Even though these evidences sound
relevant to the argument, these reports cannot guarantee the success of
the new jazz club opening in the city because only one year of jazz
festival success does not represent that local people in Monroe will visit
the new opening jazz club. There are many variables that can affect and
predict the popularity of the genre of music, jazz, in the city. Especially,
music festivals are a huge event that usually visitors who live outside of
the city can visit the city for the festival. Therefore, the author cannot
assume that all the people in the festival were from Monroe city. In order
to strengthen the argument and avoid unnecessary assumption, he
should survey how many people from the city in order to study whether
most people in the city like jazz music.
Second, according to the author, the city highest rated radio program in
Monroe is jazz program. However, all the listeners would not prefer to
visit the jazz club with several reasons. Also, there is several reasons
that the radio rating cannot possibly be a representative information of
music preference of people in the city. Even though the ratio is the
highest radio channel, people could select the one because the other
channels are terribly designed programs. Depending on preference,
listeners would not like jazz as much as they visit jazz club in the city.
Therefore, the author should specifically investigate how radio channel
surveyed the rating and the reason for why listeners chose the jazz radio
channel as the number one.
Third, nationwide study has good advantages and information to know.
However, the average amount of money spent for jazz entertainment
would necessarily represent the specific city in Monroe about the
expenditure of jazz entertainment. Some of cities can spend much
higher than the city of Monroe which weight up the average amount of
money for jazz entertainment in Monroe. In this case, the author should
refer the average of the city itself rather than the average of the
nationwide. With that, since jazz club is one of major jazz entertainment,
if the money spends are higher in the city, it is likely that people spend
more money on jazz club which increase the credence of the argument
as well.
In order to strengthen the evidences and argument, the author should
consider the listed suggestions thoroughly and discuss those
assumptions he made in the previous arguments about opening a jazz
club in the city.

The following appeared in the summary of a study on headaches suffered by


the residents of Mentia.
"Salicylates are members of the same chemical family as aspirin, a medicine
used to treat headaches. Although many foods are naturally rich in salicylates,
for the past several decades, food-processing companies have also been
adding salicylates to foods as preservatives. This rise in the commercial use
of salicylates has been found to correlate with a steady decline in the average
number of headaches reported by participants in our twenty-year study.
Recently, food-processing companies have found that salicylates can also be
used as flavor additives for foods. With this new use for salicylates, we can
expect a continued steady decline in the number of headaches suffered by the
average citizen of Mentia."
Write a response in which you discuss what specific evidence is needed to
evaluate the argument and explain how the evidence would weaken or
strengthen the argument.

The given summary of the study on headaches suffered by the residents


of Mentia, tends to propose the use of Salicylates as flavor additives for
foods. This conclusion is based on the belief that the use will lead to
decrease in the number of headaches encountered in the city of Mentia.
It relies on the assumption that, the same increase in commercial use of
salicylates in food processing companies, will be encountered in the new
use of salicylates too. There is a lot of evidence needed in order to
effectively evaluate this argument. Without a mention of the required
evidence, the argument will prove to be fallacious.
First of all, Salicylates are quoted to be the members of the chemical
family as aspirin. But how much of the property does Salicylates have, to
treat headaches must be ascertained. Just because, Saliclylates is a
member of aspirin family, cannot guarantee that it will possess the same
properties.
The foods which are naturally rich in salicylates must be further
elaborated and whether they are being purchased by the people must
also be made clear. Furthermore, the author quotes that, during the past
'several' decades, food processing companies havestarted to use
salicylates as preservatives.The number of decades must be clearly
presented, to solve the conflict, which is to follow in the argument. The
author quotes that the commercial use of salicylates is rising on par with
the steady decline in the averge number of headaches. This, is said to
be a result of the study. But the number of participants taken for the
study, their age group etc... must be mentioned to get a clear view of the
study. Moreover, since the study was conducted for twenty years, we
cannot exactly predict whether the decrease in headaches is due to the
salicylates added to foods as preservatives, or those salicylates which
naturally occur in foods.
Moreover, the author commits an error of correlation-causation. The rise
in commercial use of salicylates happened the same time around, when
the decrease in average number of headaches were reported. However,
the decline in headches, can be due to a number of other reasons such
as: the people becoming aware of their health, the incidence of
headaches having been decreased in the past few years etc... Also,
evidence has to be provided, whether all the people who have
headaches, resort to foods containing salicylates. Because, there can be
multiple ways to tackle headaches which have to be weighed against the
use of salicylates.
The summary ends up in a hasty conclusion, basing its decision, on the
recent findings of food-processing companies. These companies have
found that salicylates can be used as flavor additives for foods. But
even, if they are used, will those foods be readily accepted by the
people? Also, the average number of headaches occuring are already
reported to have dwindled. Then, there arises a doubt whether, the
demand for foods containing salicylates in any form, will continue. If the
demand decreases, then the recommendation will not hold good for
implementation. When the link between headaches and foods containing
salicylates, is not proved in the first hand, it will be too early to conclude
that, introducing salicylates as flavor additives, will increase the sales
and reduce the number of headaches.
Thus, in order to evaluate the given argument, lots of evidence needs to
be furnished. Some of them include: Salicylates having the property of
reducing the occurence of headaches, elaborate details of the study
conducted, details regarding the persons interviewed for the study,
relation between the use of salicylates in foods and decrease in the
occurence of headaches, the effectiveness of the decision to use
salicylates as flavor additives. These details must be provided, in order
to evaluate the argument without any skepticism. Unless the argument is
backed up with substantial evidence, it will remain fallible and unsuitable
to be taken into consideration.

The following appeared in a letter to the editor of a local newspaper.


"Commuters complain that increased rush-hour traffic on Blue Highway
between the suburbs and the city center has doubled their commuting time.
The favored proposal of the motorists' lobby is to widen the highway, adding
an additional lane of traffic. But last year's addition of a lane to the nearby
Green Highway was followed by a worsening of traffic jams on it. A better
alternative is to add a bicycle lane to Blue Highway. Many area residents are
keen bicyclists. A bicycle lane would encourage them to use bicycles to
commute, and so would reduce rush-hour traffic rather than fostering an
increase."
Write a response in which you discuss what specific evidence is needed to
evaluate the argument and explain how the evidence would weaken or
strengthen the argument.
The letter to the editor suggests that adding a bicycle lane to Blue
Highway would be a good solution to reduce traffic jam and commuting
time for the community. However, the argument is not quite sound
because it lacks of evidences to support the idea suggested.
According to the letter, there are two proposals suggested to solve the
problem: widen the highway with additional lane and adding the bicycle
lane. The writer says that widening Blue Highway would not be helpful
since Green Highway, which has been added a new lane, actually
worsen the traffic jams. It assumed that two Highways are on the same
situations, and share the same conditions over all. If not, the result of
Blue Highway adding a new lane would not be the same with one of
Green Highway. Without any specific details of Green Highway, it cannot
assume that the solution suggested would not be helpful to decrease the
traffic.
On top of that, it should have more specific information to support the
alternative proposal adding a bicycle lane to the highway. How many
people are willing to participate to commute by bicycle? Even though
many residents like to ride a bicycle, it does not mean they are willing to
use it to commute since using bicycle usually takes longer than using a
car. Moreover, bicyclists also need to be educated to ride a bicycle on
the highway since highway is the place vehicles drive very fast.
So far, it suggested that changing the highway would solve the problem
of traffic jam during rush-hours. However, the traffic itself might have
fundamental problems. For instance, there are too many lights on the
high way, there are too many accidents on the highway, or there is not
much public transportation from suburban to the city. Unless it finds out
the fundamental problem on the highway, any proposals might not be
useful solutions that ease the highway.
The letter presents the ideas to reduce the traffic jam during rush-hours,
and commuting time: adding a new lane for vehicles or bicycles. The first
proposal was discarded because it did not succeed in Green Highway.
But there is no evidence to both highways share the same problem and
condition. The idea of adding a new lane is not reliable because it does
not guarantee that people use more bicycles. Over all, it only gives a few
superficial solutions without concerning of any fundamental problems
that cause the traffic jams. Therefore, the arguments need more specific
information regarding Blue Highway before making decisions

The following appeared in the summary of a study on headaches suffered by


the residents of Mentia.
"Salicylates are members of the same chemical family as aspirin, a medicine
used to treat headaches. Although many foods are naturally rich in salicylates,
for the past several decades, food-processing companies have also been
adding salicylates to foods as preservatives. This rise in the commercial use
of salicylates has been found to correlate with a steady decline in the average
number of headaches reported by participants in our twenty-year study.
Recently, food-processing companies have found that salicylates can also be
used as flavor additives for foods. With this new use for salicylates, we can
expect a continued steady decline in the number of headaches suffered by the
average citizen of Mentia."
Write a response in which you examine the stated and/or unstated
assumptions of the argument. Be sure to explain how the argument depends
on these assumptions and what the implications are for the argument if the
assumptions prove unwarranted.

The given summary of the study on headaches suffered by the residents


of Mentia, tends to propose the use of Salicylates as flavor additives for
foods. This conclusion is based on the belief that the use will lead to
decrease in the number of headaches encountered in the city of Mentia.
It relies on the assumption that, the same increase in commercial use of
salicylates in food processing companies, will be encountered in the new
use of salicylates too. There is a lot of evidence needed in order to
effectively evaluate this argument. Without a mention of the required
evidence, the argument will prove to be fallacious.
First of all, Salicylates are quoted to be the members of the chemical
family as aspirin. But how much of the property does Salicylates have, to
treat headaches must be ascertained. Just because, Saliclylates is a
member of aspirin family, cannot guarantee that it will possess the same
properties.
The foods which are naturally rich in salicylates must be further
elaborated and whether they are being purchased by the people must
also be made clear. Furthermore, the author quotes that, during the past
'several' decades, food processing companies havestarted to use
salicylates as preservatives.The number of decades must be clearly
presented, to solve the conflict, which is to follow in the argument. The
author quotes that the commercial use of salicylates is rising on par with
the steady decline in the averge number of headaches. This, is said to
be a result of the study. But the number of participants taken for the
study, their age group etc... must be mentioned to get a clear view of the
study. Moreover, since the study was conducted for twenty years, we
cannot exactly predict whether the decrease in headaches is due to the
salicylates added to foods as preservatives, or those salicylates which
naturally occur in foods.
Moreover, the author commits an error of correlation-causation. The rise
in commercial use of salicylates happened the same time around, when
the decrease in average number of headaches were reported. However,
the decline in headches, can be due to a number of other reasons such
as: the people becoming aware of their health, the incidence of
headaches having been decreased in the past few years etc... Also,
evidence has to be provided, whether all the people who have
headaches, resort to foods containing salicylates. Because, there can be
multiple ways to tackle headaches which have to be weighed against the
use of salicylates.
The summary ends up in a hasty conclusion, basing its decision, on the
recent findings of food-processing companies. These companies have
found that salicylates can be used as flavor additives for foods. But
even, if they are used, will those foods be readily accepted by the
people? Also, the average number of headaches occuring are already
reported to have dwindled. Then, there arises a doubt whether, the
demand for foods containing salicylates in any form, will continue. If the
demand decreases, then the recommendation will not hold good for
implementation. When the link between headaches and foods containing
salicylates, is not proved in the first hand, it will be too early to conclude
that, introducing salicylates as flavor additives, will increase the sales
and reduce the number of headaches.
Thus, in order to evaluate the given argument, lots of evidence needs to
be furnished. Some of them include: Salicylates having the property of
reducing the occurence of headaches, elaborate details of the study
conducted, details regarding the persons interviewed for the study,
relation between the use of salicylates in foods and decrease in the
occurence of headaches, the effectiveness of the decision to use
salicylates as flavor additives. These details must be provided, in order
to evaluate the argument without any skepticism. Unless the argument is
backed up with substantial evidence, it will remain fallible and unsuitable
to be taken into consideration.

The following appeared in an editorial in a local newspaper.


"Commuters complain that increased rush-hour traffic on Blue Highway
between the suburbs and the city center has doubled their commuting time.
The favored proposal of the motorists' lobby is to widen the highway, adding
an additional lane of traffic. Opponents note that last year's addition of a lane
to the nearby Green Highway was followed by a worsening of traffic jams on
it. Their suggested alternative proposal is adding a bicycle lane to Blue
Highway. Many area residents are keen bicyclists. A bicycle lane would
encourage them to use bicycles to commute, it is argued, thereby reducing
rush-hour traffic."
Write a response in which you discuss what questions would need to be
answered in order to decide whether the recommendation and the argument
on which it is based are reasonable. Be sure to explain how the answers to
these questions would help to evaluate the recommendation.

The author argues here that adding a bicycle lane to Blue highway is a
successful alternative to rush-hour traffic. Stated in this way, the
argument fails to mention several key factors on the basis it could be
evaluated. On the support of this suggestion, the author cites that adding
new bicycle lane would encourage residents to use bicycle to commute.
However, careful scrutiny of evidence reveals it provide little credible
support to the author's conclusion. Hence, the argument can be
considered incomplete and unsubstantiated.
First of all, the argument readily assumes that widening the highway the
year before causes more problems as opponents said. This is merely an
assumption made with much solid ground. For example, the author did
not mention how many people complaints about that. May one or two
individuals make the same complaints. The argument would have much
more convincing if there is an on-site survey to examine these
complaints. If the complaints are true, there may be some other reasons.
For example, the behavior of vehicles drives. There should be
Secondly, the author points out that many area residents are keen
bicyclists. This again a weak and unsupported claim.as it didn't mention
the exact number of residents who are keen bicyclists. The word "many"
is a vague word. How does the author know that many of them are keen
to use bicycles? If this is true, why they are using other vehicles while
this is a cheaper one? If the argument had provided evidence that they
are keen to use bicycles, it would have been much more convincing.
Finally, the author cites that adding a new bicycle lane to the highway
will encourage residents to use bicycles. This raises skeptical questions.
For example, is adding new lane only enough to encourage them to use
their bicycles? Is using their bicycles enough to solve the rush-hour
traffic? Without convincing answers to these questions, the reader is left
with the impression that the author's conclusion is only a wishful thinking.
In conclusion, the argument is unpersuasive as it stands. To bolster it
further, the author needs to provide concrete evidence. Perhaps by way
of detailed analysis. To evaluate it better, the author should provide
more information about the residents and their behavior.

The following appeared as a recommendation by a committee planning a ten-


year budget for the city of Calatrava.
"The birthrate in our city is declining: in fact, last year's birthrate was only one-
half that of five years ago. Thus the number of students enrolled in our public
schools will soon decrease dramatically, and we can safely reduce the funds
budgeted for education during the next decade. At the same time, we can
reduce funding for athletic playing fields and other recreational facilities. As a
result, we will have sufficient money to fund city facilities and programs used
primarily by adults, since we can expect the adult population of the city to
increase."
Write a response in which you discuss what specific evidence is needed to
evaluate the argument and explain how the evidence would weaken or
strengthen the argument.

The planning committee predicts increase in adult population in the


Calatrava city and thus suggests allocation of extra budget for adult
facilities and programs by cutting the budget allocated to the education
and children recreational facilities. The argument of the committee is
presumptuous and makes sweeping conclusion even when there is lack
of evidence to come to any solid conclusion.
To begin with, the committee cites the declining birth rate of the
Calatrava city and thus concludes that number of students enrolling to
schools will decrease dramatically in next ten years. It is completely
wrong on part of committee to consider that the trend of decline in birth
rate will continuously follow for upcoming ten years. The committe
should provide insightful scientific data that reports the population of the
Calatrava city for atleast next ten years. It may happen that from next
year, the birth of the city starts increasing and consequently more
number of students need education facilities than before. In such
scenario, If budget of the education is reduced, education system in the
city will collapse and create a chaos for students. Also, the committee
has completely overlooked many of the external factors like migration
while coming to a conclusion. Hence, committee should provide
information as such about the current influx rate of migration to the city
and also provide information whether this rate is increasing or
decreasing. Hence, without such evidence, any conclusion about future
enrollment in schools of city is flawed.
Next, the committee also suggests that the funding for athletic activities
and other recreational activites should be reduced due to expected
decline in number of students enrolled to school and instead diverted to
make adult facilities. First of all, it is not clear whether students
enrollment to school will decrease or not, and thus to base any
conclusion upon this faulty premise is a big mistake and thus makes
argument flawed.
At last the committee predicts that adult population in the city will
increase. This claim is not backed by any evidence. The committee must
provide stastics reporting total number of adult population predicted for
next ten years. But the evidence that it must furnish must contain
population of adults acoording to age groups. For it is possible that the
committees’ plan to increase funding for adult facilities will only be useful
if most of the adults are old. Because, if most of the adults are middle
aged it is quite possible that they will have children in near future and
thus number of students enrolling to schools will automatically go up and
more funding for education will be needed.
Hence, in conclusion, the committee must provide warranted data
predicting future birth rate of city for at least ten years, expected influx
migration population in next ten years, and projected population of adults
as per their age group for atleast next ten years. Because, without these
supporting evidence as of now, the argument of the committee if flawed
and not persuasive.

The following was written as a part of an application for a small-business loan


by a group of developers in the city of Monroe.
"A jazz music club in Monroe would be a tremendously profitable enterprise.
Currently, the nearest jazz club is 65 miles away; thus, the proposed new jazz
club in Monroe, the C-Note, would have the local market all to itself. Plus, jazz
is extremely popular in Monroe: over 100,000 people attended Monroe's
annual jazz festival last summer; several well-known jazz musicians live in
Monroe; and the highest-rated radio program in Monroe is 'Jazz Nightly,'
which airs every weeknight at 7 P.M. Finally, a nationwide study indicates that
the typical jazz fan spends close to $1,000 per year on jazz entertainment."
Write a response in which you discuss what specific evidence is needed to
evaluate the argument and explain how the evidence would weaken or
strengthen the argument.

The following application appeals to a group of developers that a jazz


music club in Monroe would be very good and profitable business
because jazz is very popular in the area. There is no jazz club around,
jazz is favorite genre in the area, and a study has shown that jazz fan
spends money on it. However, the applicant needs to provide more
information and evidences to support the argument.
Application explains that the closest jazz club is 65 miles away, so that a
new jazz club should attracts the local people in Monroe. But it misses a
few points that need to be looked at. First, the arguments assumed that
a new jazz club definitely attracts the people just because it is closer
than the other club. Perhaps, people are willing to travel far distance and
enjoy the music and the atmosphere. Or there are not enough people
who are interested in jazz music club. Moreover, it never mentions how
they would be able to bring people to a new club. Therefore, it does not
perfectly support the reason why it is profitable.
As a second reason, the applicant mentioned that jazz is very popular in
Monroe. However, the premises on the argument are not reliable.
100,000 people who attended the jazz festival do not only count on the
local people but also people from outside. In addition, it is not clear
correlations between number of musicians in the area and popularity.
Also, high-rated jazz radio program merely means that other programs
are boring, or that is the only music program available at 7 PM. Or, it
looks popular because it is commute time when many people are on the
road.
Furthermore, it mentions the nationwide research that shows how much
money jazz fans spend. This point assumes that nationwide sample
surely applies to the smaller group in the local, however, it only weaken
the argument. Since it is not the research subjected only the jazz fans in
Monroe, it does not guarantee that they are willing to spend $1,000 per
year on jazz entertainment.
The following application asserts that a new jazz club have great
potential to make profit in Monroe city because it jazz is very popular in
the area, however, it provides too little and unreliable information to
persuade the group of developers. Therefore, all the reasons that
supported the ideas are too superficial; it needs more market research
and detailed information.

The following appeared in the summary of a study on headaches suffered by


the residents of Mentia.
"Salicylates are members of the same chemical family as aspirin, a medicine
used to treat headaches. Although many foods are naturally rich in salicylates,
for the past several decades, food-processing companies have also been
adding salicylates to foods as preservatives. This rise in the commercial use
of salicylates has been found to correlate with a steady decline in the average
number of headaches reported by participants in our twenty-year study.
Recently, food-processing companies have found that salicylates can also be
used as flavor additives for foods. With this new use for salicylates, we can
expect a continued steady decline in the number of headaches suffered by the
average citizen of Mentia."
Write a response in which you discuss what specific evidence is needed to
evaluate the argument and explain how the evidence would weaken or
strengthen the argument.

The given summary of the study on headaches suffered by the residents


of Mentia, tends to propose the use of Salicylates as flavor additives for
foods. This conclusion is based on the belief that the use will lead to
decrease in the number of headaches encountered in the city of Mentia.
It relies on the assumption that, the same increase in commercial use of
salicylates in food processing companies, will be encountered in the new
use of salicylates too. There is a lot of evidence needed in order to
effectively evaluate this argument. Without a mention of the required
evidence, the argument will prove to be fallacious.
First of all, Salicylates are quoted to be the members of the chemical
family as aspirin. But how much of the property does Salicylates have, to
treat headaches must be ascertained. Just because, Saliclylates is a
member of aspirin family, cannot guarantee that it will possess the same
properties.
The foods which are naturally rich in salicylates must be further
elaborated and whether they are being purchased by the people must
also be made clear. Furthermore, the author quotes that, during the past
'several' decades, food processing companies havestarted to use
salicylates as preservatives.The number of decades must be clearly
presented, to solve the conflict, which is to follow in the argument. The
author quotes that the commercial use of salicylates is rising on par with
the steady decline in the averge number of headaches. This, is said to
be a result of the study. But the number of participants taken for the
study, their age group etc... must be mentioned to get a clear view of the
study. Moreover, since the study was conducted for twenty years, we
cannot exactly predict whether the decrease in headaches is due to the
salicylates added to foods as preservatives, or those salicylates which
naturally occur in foods.
Moreover, the author commits an error of correlation-causation. The rise
in commercial use of salicylates happened the same time around, when
the decrease in average number of headaches were reported. However,
the decline in headches, can be due to a number of other reasons such
as: the people becoming aware of their health, the incidence of
headaches having been decreased in the past few years etc... Also,
evidence has to be provided, whether all the people who have
headaches, resort to foods containing salicylates. Because, there can be
multiple ways to tackle headaches which have to be weighed against the
use of salicylates.
The summary ends up in a hasty conclusion, basing its decision, on the
recent findings of food-processing companies. These companies have
found that salicylates can be used as flavor additives for foods. But
even, if they are used, will those foods be readily accepted by the
people? Also, the average number of headaches occuring are already
reported to have dwindled. Then, there arises a doubt whether, the
demand for foods containing salicylates in any form, will continue. If the
demand decreases, then the recommendation will not hold good for
implementation. When the link between headaches and foods containing
salicylates, is not proved in the first hand, it will be too early to conclude
that, introducing salicylates as flavor additives, will increase the sales
and reduce the number of headaches.
Thus, in order to evaluate the given argument, lots of evidence needs to
be furnished. Some of them include: Salicylates having the property of
reducing the occurence of headaches, elaborate details of the study
conducted, details regarding the persons interviewed for the study,
relation between the use of salicylates in foods and decrease in the
occurence of headaches, the effectiveness of the decision to use
salicylates as flavor additives. These details must be provided, in order
to evaluate the argument without any skepticism. Unless the argument is
backed up with substantial evidence, it will remain fallible and unsuitable
to be taken into consideration.

The following appeared in a letter to the editor of Parson City's local


newspaper.
"In our region of Trillura, the majority of money spent on the schools that most
students attend — the city-run public schools — comes from taxes that each
city government collects. The region's cities differ, however, in the budgetary
priority they give to public education. For example, both as a proportion of its
overall tax revenues and in absolute terms, Parson City has recently spent
almost twice as much per year as Blue City has for its public schools — even
though both cities have about the same number of residents. Clearly, Parson
City residents place a higher value on providing a good education in public
schools than Blue City residents do."
Write a response in which you discuss what specific evidence is needed to
evaluate the argument and explain how the evidence would weaken or
strengthen the argument.

In Trillura region,the city of Parson spents more on providing good


education to the city residents than the Blue city.This conclusion has
been derived from the fact that recently Parson has spent twice the
amount expended by Blue city.However,this reasoning is based on
several unwarranted assumptions which highly undermine the validity of
the conclusion.
Both the cities have public schools where most of the city students take
education.No evidence has been provided about the number of such
schools in each city.Are they equal?.Does Parson city has more public
schools compared to Blue city?.If yes,than the comparison of fund
allocated for public education is fallible.Since public schools are more in
Parson city,it will eventually have to spent more for maintenance of each
school.Hence without knowing the number of schools in each city,it is
erroneous to deduce that Parson city values good education than Blue
city based on the amount of funds allocated for public schools.
As mentioned that Parson city has recently spent more on the public
schools than Blue city.What was the need of spending more
money?.Was it due to poor quality of education or infrastructure
earlier?.The argument has overlook these plausible reasons for recent
accretion in the money spent on public schools of Parson city.Parson
city might be receiving complaints from the city residents for the inferior
quality of education provided,and therefore might have decide to
upgrade the quality or infrastructure by allocating more money compared
to previous years.Blue city is not spending more has its public schools
already provide good quality education and therefore there is no need to
accrue more funds than usual to allocate for public schools.Hence the
argument is weakned due to lack of reasons for the increase in more
money spent on public schools in Parson city.
Further,the funds budgeted for public schools are collected from the tax
paid by the city residents in each city.Do the tax rates in each city are
equal?.Also is the average income earned by a person in each city
equal?.If the tax rates are more in Parson city compared to Blue city
given that each person average earnings are almost similar in each
city,than Parson city is bound to garner more money to utilize for public
school education in comparsion to Blue city.Even if the tax rates are
equal in each city,but the average earnings of people in Parson city is
more than Blue city and therefore more money gets allocated to provide
quality education.The argument needs to do further study to find out the
tax rate waged on residents of each city and the average income of each
city residents.Without these statistical data,the claim of the argument is
unjustified that Parson citys value good quality education more than Blue
city.
Hence the argument makes several assumptions in deducing that
Parson city’s residens value good education more than Blue city.Indeed
if these assumptions hold true,than the conclusion totally falls apart.

The following appeared in a memo from a vice president of Quiot


Manufacturing.
"During the past year, Quiot Manufacturing had 30 percent more on-the-job
accidents than at the nearby Panoply Industries plant, where the work shifts
are one hour shorter than ours. Experts say that significant contributing
factors in many on-the-job accidents are fatigue and sleep deprivation among
workers. Therefore, to reduce the number of on-the-job accidents at Quiot and
thereby increase productivity, we should shorten each of our three work shifts
by one hour so that employees will get adequate amounts of sleep."
Write a response in which you examine the stated and/or unstated
assumptions of the argument. Be sure to explain how the argument depends
on these assumptions and what the implications are for the argument if the
assumptions prove unwarranted.

The vice president of Quiot Manufacturing claims that by reducing the


work shifts of the workers, the number of on-the-job accidents at Quiot
will reduce and this will increase the productivity of the workers. The vice
president makes this claim based on the fact that Quiot Manufacturing
had 30 percent more on the job accidents than the nearby Panoply
Industries plant where the working hours are 1 hour less. However, to
evaluate the argument, it is necessary to consider the various
assumption this argument is based on -
The vice president assumes that the Panoply Industries and Quiot
Manufacturies are similar and have the same working conditions. This is
a major assumption since the two industries can be vastly different in
their operation. For instance, Quiot manufacturing might manufacture
products like blades or glass bottles that require a lot of care while being
dealt with. Whereas, Panoply industries could be dealing with vastly
different products like plush toys. In such a scenario, the workers at
Quiot Manufacturing are more exposed to dangerous machines and
materials compared to Panoply industries. Therefore, the workers at
Quiot are more prone to accidents compared to the workers at Panoply.
If this scenario is true, it highly undermines the argument that reducing
work shifts by one hour will reduce the accidents.
Furthermore, the vice president assumes that the accidents are caused
due to sleep deprivation and fatigue. He also assumes that the workers
are sleep deprived due to the extra working hours. The accidents could
be for a variety of reasons other than sleep deprivation. Perhaps Quiot
doesn’t have all the safety measurements in place to operate the
machinery or the workers are not trained well enough to deal with these
machines without getting hurt. Even if it can be proved that fatigue and
sleep deprivation cause the most number of accidents, it cannot be
proved that an extra working hour is the cause of sleep deprivation. For
instance, many of the workers might have night jobs at other industries
and that is the major reason for sleep deprivation and not the extra hour
at Quiot. If such a scenario exists, the argument will not hold water since
reducing the work hours will not affect reducing the number of accidents.
The vice president also assumes that by reducing the number of
accidents, the productivity of the workers is sure to increase. Even if the
accidents reduce, the productivity might remain the same. However, if
the working hours have reduced, the productivity is sure to decrease.
Since the loss in productivity due to a few workers who met with an
accident cannot be greater than the loss in productivity of all the workers
by one hour.
In conclusion, the argument depends on several unwarranted
assumptions that undermine the argument considerably. There needs to
be sufficient proof provided regarding the correlation between sleep
deprivation and accidents, the similarity of Q with P, and that reducing
the number of hours will ensure a rise in productivity.
The following appeared in a memorandum from the planning department of an
electric power company.
"Several recent surveys indicate that home owners are increasingly eager to
conserve energy. At the same time, manufacturers are now marketing many
home appliances, such as refrigerators and air conditioners, that are almost
twice as energy efficient as those sold a decade ago. Also, new technologies
for better home insulation and passive solar heating are readily available to
reduce the energy needed for home heating. Therefore, the total demand for
electricity in our area will not increase — and may decline slightly. Since our
three electric generating plants in operation for the past twenty years have
always met our needs, construction of new generating plants will not be
necessary."
Write a response in which you examine the stated and/or unstated
assumptions of the argument. Be sure to explain how the argument depends
on these assumptions and what the implications are for the argument if the
assumptions prove unwarranted.

The memorandum from the planning department of an electric power


company predicts that, the demand for electricity will decline shortly,
owing to the increase in production of energy efficient devices and
alternative technologies which are becoming prevalent. But, the
argument given here, is based on a number of stated and unstated
assumptions. The author draws a conclusion, primarily based on few
arguments which do not seem to hold considerable value. Hence, the
assumptions must be warranted for the conclusion to be effective.
First of all, the argument quotes the results of the recent survey. The
exact year during which the survey was conducted, must be mentioned.
If the survey has been conducted 4 years ago(which can be deemed
'recent' by the author), then the results will not hold good. Moreover,
people are eager to conserve energy. This leads to the unstated
assumption that, they will take several steps to conserve energy for sure.
While people can be interested to conserve energy, there is no
guarantee until they do so.
The manufacturers' marketing of home appliances which are twice as
energy efficient compared to those sold ten years ago, does not
necessarily imply that the people will like to purchase them. It is
assumed that, the cost of these appliances will be in the affordable
range. On the other hand, if the appliances are costly due to their high
energy efficient nature, then it will lead to turmoil and the desired sales
might not be achieved. Hence, more details regarding the appliances
need to be furnished. Also, the energy efficiency must be quantified, in
order to compare it with the value that existed ten years ago. For
instance, if the energy efficiency of the appliances was 4% ten years
back, now it will be 8% and it no great improvement. Thus, the authors
ideas have to be substantiated with quantified data.
Moreover, new technologies for home insulation and passive solar
heating are quoted to be readily available to decrease the energy
required for home heating. The applicability and the feasibility of these
technologies, must be mentioned in the argument. Unless, these
technologies are inbuilt in the appliances, there is no assurance that, it
will yield the desired benefits. Furthermore, whether heating will readily
replace, the energy produced by electricity has to be determined. If the
energy cannot be compensated, then there will be more demand for
electiricity and people will not prefer to use heating technology.
Moreover, some people can resort to electricity, despite the advantages
of solar heating being obvious. Hence, more information regarding the
nature of people and their income levels, must be specified in order to
arrive at the decision, regarding the demand for electricity.
Above all, the author assumes that the currently existing three of the
electric generating plants, will continue to function in a normal manner.
The state of the electric generating plants must be explicitly stated.
Otherwise, there arises a doubt, whether plants which have been
working for 20 years, will be able to sustain for a long period of time.
Also, the author assumes that the demand for electricity will almost
remain same in the years to come. On the other hand, if some
unforeseen circumstance, such as the population explosion occurs, then
more electricity plants will needed to be constructed.
Thus, the author assumes a lot of details in presenting his argument;
most of them remain unwarranted; some of them include: the survey
being conducted very recently, affordability of the appliances,
considerable energy efficiency of the new devices, applicability and
feasibility of the newly introduced technologies, the guaranteed normal
functioning of the electric generating plants. If these assumptions prove
warranted, then the argument given in the memorandum can be
assumed to be true, and we can anticipate decrease in the consumption
of electricity.

The vice president of human resources at Climpson Industries sent the


following recommendation to the company's president.
"In an effort to improve our employees' productivity, we should implement
electronic monitoring of employees' Internet use from their workstations.
Employees who use the Internet from their workstations need to be identified
and punished if we are to reduce the number of work hours spent on personal
or recreational activities, such as shopping or playing games. By installing
software to detect employees' Internet use on company computers, we can
prevent employees from wasting time, foster a better work ethic at Climpson,
and improve our overall profits."
Write a response in which you examine the stated and/or unstated
assumptions of the argument. Be sure to explain how the argument depends
on these assumptions and what the implications are for the argument if the
assumptions prove unwarranted.

The recommendation from the vice president of human resources to the


president of Climpson Industries is based on certain assumptions, and an overall
lack in logical reasoning and rationality. This insular argument stands on
unproved hypothesis, and is rife with glitches.
One major reason why the argument is flawed is the direct correlation it assumes
with authority between the employees’ productivity and their use of internet.
Without a previously developed model or an undertaken survey to understand
how productivity fluctuates with the employees’ use of internet in the
workstations, one cannot draw a logical conclusion about the effectiveness of the
introduction of the new monitoring system. Furthermore, it assumes people using
company computers to indulge only in gaming and shopping, and that it is not
utilized for personal constructive purposes.

An additional disadvantage of the argument is its assumption of the employees’


internet use limited only to workstation computers. What the argument fails to
portray is that people can equally waste the same time indulging in such
activities from their personal laptops or cell phones. Moreover,  it also fails to
delineate the extent to and the effectiveness with which the software will gather
data for employee analysis.

A further drawback of the argument lies in the fact that it views personal and
recreational activities as time waste and unnecessary. Quite often, work proves
strenuous, leading people to indulge into personal activities to regain their vigour
for further work. Stalling such small recreational activities between works may
only prove fatuous, hindering growth and bringing in boredom. Interest level can
plummet with continuous work pressure, thus bringing down the entire
productivity.

Another notable flaw the argument poses is an obvious and unwanted result:
intrusion of privacy, which results in uncalled hesitancy and suspicion on the
part of the employees, who might be skeptical of the monitoring of their internet
use. Nevertheless, the vice president does not provide an insight as to how the
curbing the employees’ free time will result in better work ethic and increased
productivity. A realization of the other major factors like employment motivation,
team management, effective administration, adoption of better technology and an
overall conducive ambiance play an important part in the productivity of a
company, which clearly has not been hinted at. 

Taking all the above things into consideration, it can be summarized that if the
recommended steps as outlined by the vice president are imposed, it might not
prove to be as effective as it had been forecasted.

The following appeared in a letter from the owner of the Sunnyside Towers
apartment complex to its manager.
"One month ago, all the showerheads in the first three buildings of the
Sunnyside Towers complex were modified to restrict maximum water flow to
one-third of what it used to be. Although actual readings of water usage
before and after the adjustment are not yet available, the change will
obviously result in a considerable savings for Sunnyside Corporation, since
the corporation must pay for water each month. Except for a few complaints
about low water pressure, no problems with showers have been reported
since the adjustment. I predict that modifying showerheads to restrict water
flow throughout all twelve buildings in the Sunnyside Towers complex will
increase our profits even more dramatically."
Write a response in which you discuss what questions would need to be
answered in order to decide whether the prediction and the argument on
which it is based are reasonable. Be sure to explain how the answers to these
questions would help to evaluate the prediction

The owner of the Sunnyside towers apartment complex predicts that


their profits will increase by modifying the showerheads of all the twelve
towers in the complex. However, the assumptions and the predictions of
the owner raises a lot of questions that need to be answered before the
argument seems plausible.
The first question thet needs to be addressed is that, what is the basis of
his assumption that the water use in the complex will be reduced just by
modifying the showerheads? It is not necessary that maximum water is
used by people under the shower. It is also not necessary that people
take bath under a shower. It can happen that people might use buckets
and tumblers while using water and the showers are lying idle. Also, the
prediction that water bills will reduce and profits will rise before the
results of water usage is availabe is questionable.
Another question that can be raised is: what is the contribution of water
bills in the overall expenses of the complex? If the water bills contribute
to just a fraction of the overall expenses of the complex, then this
assumption fails to gain any favor. This is because if efforts are made to
reduce costs in that part of the budget which already contribute
minimally in the expenses, it is highly unlikely that the profits will rise
dramatically. Thus, the owner may want to try and find other amenities
that are likely to rise their profits if used economically.
Also, it can be asked whether any polls were being conducted before
taking the decision of modifying the showerheads? These polls would
have asked the residents about their water usage such as whether they
use showers too much or whether their use of water is for other
purposes where showers are not used such as cooking, gardening etc. If
this step was taken, it could've given a better idea to the management
about the steps to be taken to save water rather than just doing trials like
these showerheads modification.
Thus, the argument put forward by the owner of Sunnyside Towers
apartment complex about modifying showerheads stands a lot of
scrutiny and answering. If those questions discussed above are not
answered, the assumptions and predictions of the owner are absolutely
implausible.

The following report appeared in the newsletter of the West Meria Public
Health Council.
"An innovative treatment has come to our attention that promises to
significantly reduce absenteeism in our schools and workplaces. A study
reports that in nearby East Meria, where fish consumption is very high, people
visit the doctor only once or twice per year for the treatment of colds. Clearly,
eating a substantial amount of fish can prevent colds. Since colds represent
the most frequently given reason for absences from school and work, we
recommend the daily use of Ichthaid — a nutritional supplement derived from
fish oil — as a good way to prevent colds and lower absenteeism."
Write a response in which you discuss what specific evidence is needed to
evaluate the argument and explain how the evidence would weaken or
strengthen the argument.

The argument above seems reasonable on the surface. However, the


premises and conclusion of the report differ. Thus weakening the overall
argument. The association drawn between colds and abseetism is also
weak. Discussed below are the flaws of the arguments and the
conclusion.
The first flaw of the argument is that it fails to provide evidence that East
Meria and West Meria are comparable. Do the two cities have
comparable life styles and employment demographics? If West Meria
and East Meria are comparable in terrms of above, the argument may
hold. However, the current generalization weakens the report’s
arguments. Also, the report assumes that fish consumption is the reason
why doctor visits for colds has reduced. This assumption would be valid
if the writers can prove that holding other factors- such as presence of
lack of a healthy lifestyle, good weather and incidence of diseases-
constant, fish consumption reduces colds. Otherwise, the argument is
weakened. The author also fails to clarify in what amounts and what
parts of fish cause the reductions in colds. If for instance, fish oil is the
substance responsible for reduction in colds, the conclusion may hold.
Furthermore, the writers state that colds are the major reason for
absenteeism. This assertion is only valid if the writers can prove that
people actually suffer from colds and do not just state it as a reason to
miss work or school. This is because people can say they have a cold
but do not. The lack of evidence such as doctors reports to show that
absentees have cold weakens the argument.
The report’s recommendation for the use of a supplement is a departure
from earlier arguments. The use of a supplement and the consumption
of the actual food are two different things. A supplement is processed
thereby losing certain nutrients. Therefore, the recommendation
weakens the earlier claim that high consumption of fish reduces colds.
In conclusion, the report the based on a generalization that may not be
applicable to West Meria, fails to support the stated claims and gives a
recommendation that differs from the premises of the argument. To
support the conclusion, the report should provide

The following appeared in a recommendation from the planning department of


the city of Transopolis.
"Ten years ago, as part of a comprehensive urban renewal program, the city
of Transopolis adapted for industrial use a large area of severely substandard
housing near the freeway. Subsequently, several factories were constructed
there, crime rates in the area declined, and property tax revenues for the
entire city increased. To further revitalize the city, we should now take similar
action in a declining residential area on the opposite side of the city. Since
some houses and apartments in existing nearby neighborhoods are currently
unoccupied, alternate housing for those displaced by this action will be readily
available."
Write a response in which you discuss what specific evidence is needed to
evaluate the argument and explain how the evidence would weaken or
strengthen the argument.

The arguer has recommended that in order to revitalize city Transopolis


further , it should resort to same action that proved fruitful ten years ago.
Although at face value, the argument seems convincing to readers, a
closer analysis reveals the argument is full of flaws which casts doubt on
the proposition of arguer.
In the first place, a period of ten years is a long time and circumstances
are bound to be changed since then. It is likely that ten years ago, the
city needed an industrial area and measures taken for adaptation of
industrial use proved beneficial at that time. The same measure may not
work for Transopolis at present as it is possible that the industry
requirement is almost saturated in the city. Also it is possible that this
city was a frontrunner to take such steps ten years ago and due to lack
of similar opportunities , industrialists preferred Transopolis at that time.
Since then most of nearby cities adapted similar measures and came up
with similar goals. Because of abundance of such towns nearby at
present , Transopolis will not be only choice for enthusiastic
entrepreneurs.
Secondly, sufficient proof haven’t been put forwarded to show the
reasons behind factors like built up of several factories, reduction in
crime rates, increase in property tax with the adaptation strategy that
was taken ten years ago. It may be possible that the city had taken few
other measures apart from only adapting substandard housing area like
awareness among residents that how city is providing them livelihood
and increase in tax will actually benefit them in many ways, or allocation
of more police forces. Unless all the measures are taken at present
combined, the strategy to take declining residential area may not prove
advantageous to the city.
Thirdly, the arguer cannot guarantee that the residents in that declining
neighbourhood are willing to shift to opposite side of the city. If residents
demand for excessive amount of money due to shift to another place ,
the whole strategy may crumble and would be unrealistic to implement.
The residents may want to check on the basic civic facilities like energy
and water availability, drainage system, security or communication
options in the city. Unless all these options are provided to them, they
will likely to refuse the offer.
In conclusion, the recommendations seems unconvincing as it does not
produce a cogent case that the strategy can be implemented and will be
favorable to the city. The arguments provided could have been
substantiated by providing additional information like the same
environment is present currently that proved advantageous ten years
ago or this adaptation is the only reason because of which urban
renewal program worked or the residents have agreed to move to other
places in the city. In absence of all these information, the renewal
strategy remains unconvincing and cannot be implemented at present
state.

The following appeared in a memo from the new vice president of Sartorian, a
company that manufactures men's clothing.
"Five years ago, at a time when we had difficulties in obtaining reliable
supplies of high quality wool fabric, we discontinued production of our alpaca
overcoat. Now that we have a new fabric supplier, we should resume
production. This coat should sell very well: since we have not offered an
alpaca overcoat for five years and since our major competitor no longer
makes an alpaca overcoat, there will be pent-up customer demand. Also,
since the price of most types of clothing has increased in each of the past five
years, customers should be willing to pay significantly higher prices for alpaca
overcoats than they did five years ago, and our company profits will increase."
Write a response in which you discuss what specific evidence is needed to
evaluate the argument and explain how the evidence would weaken or
strengthen the argument.

The vice-president makes an argument for reviving a discontinued


production of alpaca overcoat to make more profits. He provides
reasoning in the form of availability of new fabric supplier, no
comparable product by competitors, pent-up customer demand and
overall price increases. However the argument makes a number of
assumptions which can make it suspect.
Firstly, there is an assumption that new fabric supplier for alpa overcoat
is reliable. In light of difficulties faced by the company earlier may due to
rarity of the fabric, it is highly suspect that new fabric supplier is capable
of delivering supplies on a continuous basis. There is no evidence stated
in the argument that can reassure us regarding the same, such as any
long term contract which has clause for penalties in case of default.
Thus, the argument for renewing the production becomes feeble.
Secondly, there is an assumption that there is still demand for the alpa
overcoat presently. This assumption is made on feeble grounds that
there was outcry from customers when product was discontinued earlier
and no comparable product from the competitors. There is no evidence
that is presented regarding the pent-up demand. May be the customers
have moved on from the product by finding cheaper alternatives though
less quality from the competitors and they find them useful enough to
meet their requirements. Unless the company conducts a survey
regarding the demand, any assumptions can prove counter-productive
for the company's profits due to lack of demand. If the survey results are
favourable then they could go ahead.
Thirdly, there is an assumption that by introducing the alpa overcoat
again the company can earn more profits due to overall increase in
prices.However, there is no evidence regarding the cost of producing the
alpa overcoat presently. Just because the overall prices of clothes have
increased it does not mean it will result in more profits. Due to rarity of
fabric, it may mean more input costs finally effecting the bottomline.
Furthermore, there is assumption the demand will stay constant despite
increase in prices of clothes. May be because of downturn in the
economy customers may prefer cheaper clothes from competitors
despite fall in the quality. This makes the argument to start production in
hope for profits imprudent.
In conclusion, there are number of assumptions which are made which
lack sufficient evidence to be warranted. Unless the company obtains
additional information regarding reliability of supplies, conduct survey
regarding the existing demand and give facts regarding costs involved,
the argument rests on feeble grounds and is highly suspect.

A recent sales study indicates that consumption of seafood dishes in Bay City
restaurants has increased by 30 percent during the past five years. Yet there
are no currently operating city restaurants whose specialty is seafood.
Moreover, the majority of families in Bay City are two-income families, and a
nationwide study has shown that such families eat significantly fewer home-
cooked meals than they did a decade ago but at the same time express more
concern about healthful eating. Therefore, the new Captain Seafood
restaurant that specializes in seafood should be quite popular and profitable.
Write a response in which you discuss what specific evidence is needed to
evaluate the argument and explain how the evidence would weaken or
strengthen the argument.
In this argument, the author recommends that Bay City should open a
new seafood-specific restaurant which will lead to fame and profit. For
approving the conclusion, the author employs a variety of evidence such
as statistic data and nationwide study. At first glance, the
recommendation sounds reasonable, however, further examination
reveals that there maybe other evidence ignored by the author. Without
considering all the possibilities, the recommendation may not be
persuasive and invulnerable.
First, a 30% increase in seafood consumption at Bay City does not
necessarily illustrate that a high demand on seafood served by
restaurants is existing. Although 30% seems significant, the author may
ignore the fact that the actual level of consumption of seafood there is
rather low. If true, this data is too weak to prove that a new seafood
restaurant can bring to profit.
Second, a nationwide study showing a clear tendency that two-income
families toward dining out and eating healthily does not necessarily apply
to Bay City. Perhaps, families in here do not prefer this life-style. The
author needs to proceed a sound survey in two-income families in Bay
City to find out whether its result will support the initial idea.
Even if most of families in this city follow the nationwide trend mentioned
above, it is unfair to infer that these families will select seafood or even
seafood restaurant as their first choice. Because other than dining out,
ordering take-out food at home is relatively convenient for people who
dislike eating in a a noisy and crowed restaurant. Although people may
go out for dinner, there are still various kinds of food available for people
to pick. Facing all kinds of delicious food, it is hard to imagine people will
stick to only seafood. Furthermore, according to the common sense,
seafood can not be the only healthy diet, other nutrient food like milk,
eggs and vegetables are all can assist people to keep sound. Any of
these scenarios discussed above, if true, would cast considerable doubt
on the argument's conclusion that a new seafood restaurant in Bay City
would be popular and profitable.
Finally, even if Bay City families surge into the new seafood restaurant,
the restaurant may not be popular and profitable. In one thing, Captain
Seafood restaurant is rather unfamiliar for the citizens, and as a new
brand it may need a long time for people to accept it. Another thing is,
profit is a factor relating to not only revenue but also cost. The new
restaurant may suffer a deficit when it is establishing its positive images
at the beginning. For example, obtaining high-quality, healthy seafood,
or promoting the new restaurant both will cost a large fortune. At the
same time, if there are rare customers coming to eat, the restaurant will
lose a lot.
To sum up, the proposal lacks credibility to conclude that the new brand
seafood restaurant will be popular and profitable for the prediction is
based on several shaky and dubious surveys. To strengthen the
argument, the author would have to set a new survey which is thorough
and comprehensive enough to substantiate the feasibility of opening a
brand-new seafood restaurant in Bay City.

Milk and dairy products are rich in vitamin D and calcium — substances
essential for building and maintaining bones. Many people therefore say that a
diet rich in dairy products can help prevent osteoporosis, a disease that is
linked to both environmental and genetic factors and that causes the bones to
weaken significantly with age. But a long-term study of a large number of
people found that those who consistently consumed dairy products throughout
the years of the study have a higher rate of bone fractures than any other
participants in the study. Since bone fractures are symptomatic of
osteoporosis, this study result shows that a diet rich in dairy products may
actually increase, rather than decrease, the risk of osteoporosis.
Write a response in which you discuss what specific evidence is needed to
evaluate the argument and explain how the evidence would weaken or
strengthen the argument.
Milk and dairy products are rich in vitamin D and calcium—substances
essential for building and maintaining bones. Many people therefore say
that a diet rich in dairy products can help prevent osteoporosis, a
disease that is linked to both environmental and genetic factors and that
causes the bones to weaken significantly with age. But a long-term study
of a large number of people found that those who consistently consumed
dairy products throughout the years of the study have a higher rate of
bone fractures than any other participants in the study. Since bone
fractures are symptomatic of osteoporosis, this study result shows that a
diet rich in dairy products may actually increase, rather than decrease,
the risk of osteoporosis.
Write a response in which you discuss what specific evidence is needed
to evaluate the argument and explain how the evidence would weaken
or strengthen the argument.
Composition:
The group of people who have consistently consumed dairy products
throughout the years may have had fractures more than other people,
but the author of the article makes number of unsubstantiated
assumptions about the risk of disease. Based on these assumptions, the
author boldly claims that the consumption of the dairy products actually
increases the risk of osteoporosis (OS).Though the argument or study is
made for the betterment of the society, it may fail to achieve the goal
because it is flawed.
The author’s first mistake is to assume that many people think that the
disease OS can be prevented by consumption of dairy products and that
the disease is associated with number of environmental and genetic
factors. Although people may think that the dairy products may be good
for their bones on the whole, the author has no basis to assert that
people think that the disease is prevented by consuming such products.
It is not stated that how many people are of this opinion. May be only the
grandma living next door to the author thinks so or may be a very large
population is of this view.
Secondly, the author states that the study conducted led him to the
conclusion that the people who consume dairy products throughout their
lives have a higher rate of bone fracture. The author never states that
how many people were included in the study and for how long were they
studied. Also it is not stated that what kind of people were included in the
group and what were their environmental and genetic condition. May the
people that were under study were football players or may be hockey
players and there were no computer nerd included in the study group.
Obviously, the people involved in games tend to have a higher rate of
injury and may be fracture. Also it does not make sense that to consider
the consumption of dairy products, the cause of this disease when
author himself states that the disease is caused by many environmental
and genetic factors.
Lastly, the author states that bone fractures are symptoms of OS,
therefore the consumption of dairy products increase the risk of OS. The
author states this on the above flawed assumptions and also wrongly
assumes that any kind of fracture is an indication of OS. It may be true
that OS is accompanied with weak bones but with someone having a
fracture because of a car accident is not likely the case of OS. Also it is
not stated that in which conditions these fractures occurred. For instance
, if the person injured got a fracture by only a gentle kick of his best
buddy, may be a sign of poor bone condition. Neither is the body
structure or the genetic makeup is stated.
The author’s argument can be very powerful and persuasive if the author
were to include certain assumptions. For example if the author were to
include the type of study that was carried and by whom, the people that
participated in it, their daily activities and genetic make-up and also the
detail of why where and how these fractures occurred , can make the
authors argument very powerful. With these kind of assertions the
authors call might be more justifiable.

The following appeared in a health newsletter.


"A ten-year nationwide study of the effectiveness of wearing a helmet while
bicycling indicates that ten years ago, approximately 35 percent of all
bicyclists reported wearing helmets, whereas today that number is nearly 80
percent. Another study, however, suggests that during the same ten-year
period, the number of bicycle-related accidents has increased 200 percent.
These results demonstrate that bicyclists feel safer because they are wearing
helmets, and they take more risks as a result. Thus, to reduce the number of
serious injuries from bicycle accidents, the government should concentrate
more on educating people about bicycle safety and less on encouraging or
requiring bicyclists to wear helmets."
Write a response in which you examine the stated and/or unstated
assumptions of the argument. Be sure to explain how the argument depends
on these assumptions and what the implications are for the argument if the
assumptions prove unwarranted.

In the argument, the author comes to a conclusion that the government


should pay attention to the safety education instead of taking measures,
such as wearing a helmet, to protect bicyclists themselves. To illustrate
the effectiveness of the argument, the author provided several
assumptions, however, not convincing enough to persuade us.
In the beginning, the author assumes that a ten-year nationwide study
can prove that there are more people wearing helmets now than ten
years before. But the number 35 percent and 80 percent are just two
specific percentages of the report, rather than direct numbers. It is
entirely possible that nowadays few people wear helmets, only if the total
number bicyclists is much less than ten years ago. The author fails to
provide accurate numbers of the bicyclists, thus we can not have a
comparison of the bicyclist changes through the ten years.
In addition, the author assumes that another survey shows that the
bicycle-related accidents has increased dramatically in the same ten-
year period. However, the author doesn’t explain the agency who
conducted the survey, the total number of the sample and the location
where the researchers did the study. Without the details, we can not
trust in the representative, accuracy and the equality of the study. Even if
the study is convincible, the arguer fails to present an exact number of
the increase. Only with a percentage provided, the author can not make
an assumption about the accidents happened throughout the ten years.
Granted that there do exists more accidents, and there are more people
wearing the helmets, the author still can not assume that there have a
correlation between the helmets wearing and accidents. In fact, there
might be some other factors that cause the increase of the accidents, for
example, the less favorable conditions of the road, or the more often bad
weathers. Perhaps, the high speed cars which rush into the bicycle lane
might also be the reason of more accidents. Without ruling out the these
and other alternative explanations, the author fails to convince us that
there have a cause of effect between the helmets wearing and the
number of accidents.
In sum, the vague data, the skepticism of the authentic of the study and
the fault cause and effect relation all undermines the convincible of the
argument. Without providing more details about the study and ruling out
other possible explanations, the author cannot convince us that the
government should not encouraging or requiring bicyclists to wear
helmets.

The following is a letter to the head of the tourism bureau on the island of Tria.
"Erosion of beach sand along the shores of Tria Island is a serious threat to
our island and our tourist industry. In order to stop the erosion, we should
charge people for using the beaches. Although this solution may annoy a few
tourists in the short term, it will raise money for replenishing the sand.
Replenishing the sand, as was done to protect buildings on the nearby island
of Batia, will help protect buildings along our shores, thereby reducing these
buildings' risk of additional damage from severe storms. And since beaches
and buildings in the area will be preserved, Tria's tourist industry will improve
over the long term."
Write a response in which you discuss what specific evidence is needed to
evaluate the argument and explain how the evidence would weaken or
strengthen the argument.

In this letter, the author recommends that the administrators of the island
of Tria should charge people for using the beaches to stop the erosion of
beach sand. To support this recommendation he cites that this course of
action will reduce the number of people using the beaches and will raise
money for replenishing the sand. He also points out that reforming the
sand will protect buildings along the shores as was done on the nearby
island of Batia and consequently attract more tourists. I find this
recommendation unconvincing as it stands.
First, the author unfairly assumes that the erosion of the sand is due to
people’s extravagantly benefiting from the shores or much commuting.
There might be other environmental factors which has led to depletion of
the sand shores such as much industrial activities or even pollutant
wastes which has ruined and contaminated the sand beaches. Without
considering and ruling out this and other possible reasons the author
cannot substantially conclude that this erosion is merely because of
recreational activities of people.
Second, even assuming that people’s using up the beaches would be
the only reason of sand erosion, there is no guarantee that the charges
will limit the number of visiting people considerably to the extent that
inhibit the erosion. The charges might not be so high that hinder people
from visiting or people might be so eager and wealthy that this charges
are not good reasons to disappoint them. In addition, there is no clear
evidence demonstrating that the money gained from these charges
would be sufficient to compensate for the destructions.
Third, the author unsubstantially reasoned that this course of action will
contribute to protect the costal buildings as it was seen on the nearby
island of Batia. He provides no dear evidence that both islands have the
same environmental features such as visitors’ behaviors and weather
conditions to confirm that the result of the same actions in Batia can be
attributable to Tria. Also, thriving the tourist industry might not be merely
based on protecting the sand and buildings of the beaches, and it might
be highly dependent on other recreational facilities and decent
accommodation in the city which is not related to the beaches.
In sum, this argument fails to convince me. To bolster it the author must
points out other possible factors which might have caused the sand
erosion and also provide sufficient information about the amount of the
charges and its effectiveness in improving the conditions. To better
assess the strength of the argument I need more clear evidence showing
that whether the results in Batia can be attributed to Tria and whether
improving the sand and building of the beaches is the only requirement
to attract more tourists.

The following appeared in a memorandum written by the chairperson of the


West Egg Town Council.
"Two years ago, consultants predicted that West Egg's landfill, which is used
for garbage disposal, would be completely filled within five years. During the
past two years, however, the town's residents have been recycling twice as
much material as they did in previous years. Next month the amount of
recycled material — which includes paper, plastic, and metal — should further
increase, since charges for pickup of other household garbage will double.
Furthermore, over 90 percent of the respondents to a recent survey said that
they would do more recycling in the future. Because of our town's strong
commitment to recycling, the available space in our landfill should last for
considerably longer than predicted."
Write a response in which you discuss what specific evidence is needed to
evaluate the argument and explain how the evidence would weaken or
strengthen the argument.
In this memo, the chairperson of the West Egg Town Council asserts
that the available space of landfill would last for longer than predicted
five years, for the reasons that the residents have recycled more
materials during the past two years, as well as the charges for other
garbage will double in the next mouth. Although this argument seems
plausible at the first glance, a close scrutiny reveals its untenability.
In the first place, the author claims that during past two years, recycled
materials are as twice as they did in previous years. The author neglects
a strong possibility that more recycling mateirals are due to a larger
amount of garbage, which means it is probable that more other garbage
might be landfilled. If the amount of other garbage exceeds the
predicted, the landfill would be filled much faster than predicted.
Secondly, even though the charge of other household garbage will
double, it may attribute little to decrease the speed of filling the landfill.
On the one hand, since whether the other household garbage is the
major source of other garbage is unknown, the author cannot guarantee
the amount of other garbage would decrease. If the other factory
garbage is the most important part of other garbage, charging more
money for other household garbage is useless to reduce the other
garbage. On the other hand, the council may face more potential
problems if they charge more money for other household garbage.
Perhaps the residents do not care the more charge of garbage so that
the amount of recycled materials would not increase. Or perhaps, to
avoid expansive garbage charge, some residents put the other garbage
in the recycled garbage bins. The council would hire workers to pick out
these garbage. Thus charging more money for household garbage is
unjustifiable.
Finally, the statistics of recent survey is vague and unconvincing. The
survey cannot be seen as representative if most respondents are the
people who are more willing to protect environment. Furthermore, even
assume those repondents are representative, whether they would do
more recycling is unknown. Maybe they have recycled garbage as much
as they can, it is hard for them to recycle more. Hence this survey
cannot predict that most residents would recycle more garbage in the
future.
In conclusion, the author fails to prove that the available space of landfill
would last for a longer time, at least bases on this memo. To make the
argument more persuasive, the chairperson should provide more
information about the amount of garbage landfilled during the past two
years. More statistics about the ratio of other household garbage to other
garbage and specific condition of recycled garbage is also appreciated.

The following appeared in a letter to the editor of a journal on environmental


issues.
"Over the past year, the Crust Copper Company (CCC) has purchased over
10,000 square miles of land in the tropical nation of West Fredonia. Mining
copper on this land will inevitably result in pollution and, since West Fredonia
is the home of several endangered animal species, in environmental disaster.
But such disasters can be prevented if consumers simply refuse to purchase
products that are made with CCC's copper unless the company abandons its
mining plans."
Write a response in which you examine the stated and/or unstated
assumptions of the argument. Be sure to explain how the argument depends
on these assumptions and what the implications are for the argument if the
assumptions prove unwarranted.

The author of the statement above believes that the mining process of
CCC Company would result in an environmental disaster. Based on this
premise, the author concludes that in order to preclude any disaster, all
the customers should boycott the company. The author reaches the
conclusion via taking for granted several unsupported assumptions.
First, there is no evidence that the pollution and the following disaster
are inevitable. If the CCC observes the environmental criteria in
extracting copper from the mines in that area, there would not be a
serious concern about the pollution that the company would introduce to
the area. There is no evidence that such a significant pollution is on its
way to this area.
Secondly, since the CCC has purchased 10,000 square miles of the
region, the author assumes that mining copper on the land would
jeopardize the life of endangered species. Although the surface under
the control of CCC is vast, but it may not have significant overlap with
the inhabitants of the endangered species. The mining might take place
in a place quite far from the place that endangered species are living.
Hence there would not be a serious threat to the species.
Thirdly, even if the CCC’s work is a looming environmental disaster,
there may be other ways to preclude the company to release pollution
into nature. The author’s unspoken assumption is that if the customers
“simply” do not buy the products of CCC, then the problem would be
solved. It might not be that easy not to buy the CCC products. The CCC
might be the only supplier of certain products that customers cannot
readily provide them otherwise. Furthermore, the author did not assess
other ways. For instance, a protestation against the company might force
the government to inhibit any course of mining in there.
Finally, assuming that boycotting this company would preclude it from
working there and will preclude the environmental disasters. This
strategy works only when there is not any other company making
pollution in that area. Otherwise, the pollution would be released
continuously by the other companies and the disaster would come with
or without boycotting only one agent of this pollution.
In short, as discussed, there are several assumptions in the author’s
argument left unsupported. With each assumption potentially being
unwarranted, the whole conclusion is questionable until the argument is
buttressed by the firm evidence.
The following is part of a memorandum from the president of Humana
University.
"Last year the number of students who enrolled in online degree programs
offered by nearby Omni University increased by 50 percent. During the same
year, Omni showed a significant decrease from prior years in expenditures for
dormitory and classroom space, most likely because instruction in the online
programs takes place via the Internet. In contrast, over the past three years,
enrollment at Humana University has failed to grow, and the cost of
maintaining buildings has increased along with our budget deficit. To address
these problems, Humana University will begin immediately to create and
actively promote online degree programs like those at Omni. We predict that
instituting these online degree programs will help Humana both increase its
total enrollment and solve its budget problems."
Write a response in which you discuss what questions would need to be
answered in order to decide whether the prediction and the argument on
which it is based are reasonable. Be sure to explain how the answers to these
questions would help to evaluate the prediction.

This argument is well presented yet far-fetched. It lays a claim that the
increasing number of students and the decreasing expenditures for
dormitory and classroom in Omni University is because of the new
degree program - online program. Nevertheless, the argument is in
effect definitely unreasonable due to several flaws after a closer scrutiny,
though it may appear plausible at the first glance.
To begin with, the president of Humana University imputes the
decreasing number of enrollments to the lack of the online degree
program, like the nearby Omni University. Maybe the true cause of the
Omni University to implement the program is because it is inconvenient
traffic condition. It might locate at the barren area that impedes their
students to go to school and forces them to take the online degree
program. And the Humana University fails to increase the number of
students is due to its diminishing quality of teaching or its outmoded
facilities.
Secondly, the author states that the decreasing expenditures for
dormitory and classroom in the Omni University is due to the online
program which enable the students to take the courses online at home
or other places and don't need the dormitory or classroom. Maybe the
dormitory and classroom in the Omni University are much fresher than
those in the Humana University which cost the school to spend even
more money to maintain.
Thirdly, the president of the Humana University insists that in order to
attract more and more students to enroll, they should immediately
implement a new program just like Omni University. However, the
president fails to consider the opinion of the students. By replacing the
traditional method with the online program, the students who enjoy
hanging around with classmates and the lifestyle in the campus will
refuse to take the courses and leave. Finally, the Humana University will
not be able to solve its budget problems, even worsen.
In retrospect, it seems precipitous for the author to jump to the
conclusion based on a series of problematic premises. To bolster it the
president has to provide clear evidence that the online degree program
contributed to the increasing number of enrollment in the Omni
University and the program will attain the same effects in the Humana
University. After all, feckless attempts with a fallible method could be
nothing but a foolish errand. Thus, only by grasping the key point of this
argument could the author draw a convincible conclusion.

The following appeared in a memorandum from the owner of Movies Galore, a


chain of movie-rental stores.
"Because of declining profits, we must reduce operating expenses at Movies
Galore's ten movie-rental stores. Raising prices is not a good option, since we
are famous for our low prices. Instead, we should reduce our operating hours.
Last month our store in downtown Marston reduced its hours by closing at
6:00 p.m. rather than 9:00 p.m. and reduced its overall inventory by no longer
stocking any DVD released more than five years ago. Since we have received
very few customer complaints about these new policies, we should now adopt
them at all other Movies Galore stores as our best strategies for improving
profits."
Write a response in which you discuss what specific evidence is needed to
evaluate the argument and explain how the evidence would weaken or
strengthen the argument.

This argument is about the recent decline in the profits of Movies


Galore's ten video rental stores. The owner of the store is of the opinion
that they must reduce operating expense at the store as he points out
that thay are famous for their special bargains therefore raising the rental
prices is not a viable way to improve profits. Moreover, the author says
that last month their store in downtown Marston significantly decreased
its operating expenses by closing at 6:00 P.M. rather than 9:00 P.M. and
by reducing its stock by eliminating all movies released more than five
years ago. Furthermore the author is suggesting that in order to increase
profits without jeopardizing their reputaion for offering great movies at
lower prices, they must implement the similar changes in their other nine
Movies Galore stores. However in order to fully critique this argument,
we need to have a significant amount of additional evidence.
The first piece of evidence that we would need in order to evaluate this
agrument is why there is a decline in profits of Movies Galore's stores.
Clearly there is no mentioning of any other competitors. Whether the
competitors have found a new tactic to increase their profits or the
customers of Movies Galore's are now inclined towards other stores
because of more benifits. There is also a possibilty that the other stores
are providing thw videos in much lower prices as compared to Movies
Galore's store and this might be the reason in decline in thier profits.
Another piece of evidence that might help us evaluate this claims involve
the owner strategy about one store in downtown Marston which
significantly decreased its operating expenses by closing at 6:00 P.M.
rather than 9:00 P.M. and by reducing its stocks by eliminating all
movies released more than five years ago. But the author has not
provided the conclusion about the downtown Marston store whether it is
in profit or not. Clearly if this store is also not in making profit then the
owner opinion about descrsing the operating expenses to all other nine
stores will not be viable.
One problem with this argument is that the author has not provided the
details of the latest videos releases. It might be possible that the video
industry in going through a rough patch and no good movies or songs
are being released which is why there is a decline in business. Moreover
the citizen has now turned towards digital gadgets like MP3 player, iPod
etc which provides lots of songs, videos in a small device and is much
cheaper and easier to carry. Furthermore the increase of internet
websites like Youtube etc are providing all these stuffs for free which
might be the case in the decline in the profits of Movies Galore's stores.
The author has not offered convincing evidence to support his views. He
should have strengthened his argument by providing the concrete details
on how by decreasing the operating expense will increase their stores
profits. To conclude whether reduction in operating expenses and stocks
will increase their profits, different questions relating the relevant
evidence is required. The author has to work towards making his
assumption logically more correct.
The following appeared in a magazine article about planning for retirement.
"Clearview should be a top choice for anyone seeking a place to retire,
because it has spectacular natural beauty and a consistent climate. Another
advantage is that housing costs in Clearview have fallen significantly during
the past year, and taxes remain lower than those in neighboring towns.
Moreover, Clearview's mayor promises many new programs to improve
schools, streets, and public services. And best of all, retirees in Clearview can
also expect excellent health care as they grow older, since the number of
physicians in the area is far greater than the national average."
Write a response in which you discuss what specific evidence is needed to
evaluate the argument and explain how the evidence would weaken or
strengthen the argument.
The argument that encouraged people to choose a quiet place to retire
for the benefits of economy and health may seem logical at first glance.
The author makes a valid argument, one that would be correct if its
premises were true. However, his conclusion relies on assumptions for
which there is no clear evidence, and it uses terms that lack definition.
First, the writer assumes that a clearview place represents natural
beauty and suitable environment. In fact, there is not a concrete
connection among them. For example, a natural beautiful place may
include a beautiful lake, a spectacular mountain, and a wilderness area
where elderly can enjoy fresh air and quiet while a clearview place has
nothing around it. Furthermore, the author assumes that people want to
live in quiet place after they retire. Indeed, many of whom prefer to live in
big cities where the public transportations, restaurants, and other
services are very convenient, and their families and their friends live
near them.
Additionally, the writer provides no evidence that housing costs in
clearview place have decreased. In order to convince customers, he
should give the specific statistic of how many percent of housing costs
were reduced, and the rate of the decrease to compare with others
places.Moreover, which programs will benefit to elderly should be
showed from many new programs that clearview’s mayor promises. In
some cases, improving school systems and public services advantage to
young people than old people because retirees often spent a lot of time
at home, and they pay more attention for health care systems and their
pensions. The assumption is that if the number of doctors in this place is
greater than others, there is not anybody can trust that the quality of
health care systems in clearview place is better than others without
some evidences.
In conclusion, the writer would not necessarily be wrong to assert that
clearview place brings many advantage for retirees. But to support his
conclusion, the writer must first define the clearview place more clearly
and submit more conclusive evidence that housing costs, health care
systems, and public services are suitable for retirees.
The following is part of a memorandum from the president of Humana
University.
"Last year the number of students who enrolled in online degree programs
offered by nearby Omni University increased by 50 percent. During the same
year, Omni showed a significant decrease from prior years in expenditures for
dormitory and classroom space, most likely because online instruction takes
place via the Internet. In contrast, over the past three years, enrollment at
Humana University has failed to grow and the cost of maintaining buildings
has increased. Thus, to increase enrollment and solve the problem of budget
deficits at Humana University, we should initiate and actively promote online
degree programs like those at Omni."
Write a response in which you examine the stated and/or unstated
assumptions of the argument. Be sure to explain how the argument depends
on these assumptions and what the implications are for the argument if the
assumptions prove unwarranted.

The author of the argument purportedly highlights that Humana


University should enhance its educational system via internet in order to
encourage majorities of students to enroll at the university. However, the
premises upon which he puts his claim are fallacious. For the support of
which some critical, yet ignored question need to be addressed.
To commence with, the author contends that last year The University of
Omni improved its online programs and the number of enrollment
increased significantly. However, it does not lend credence to the
argument since, a question that might arise is whether this program
could be successful in long run. One point that should considered is that
the number of enrollment in every university fluctuate every year.
Indeed, maybe most of the students at Omni University have worked
along with their education and they preferred to take online classes
during past year. Perhaps it did not seems useful for students during a
year and they made a decision to attend classes afterward. It is also
important to say that maybe this university are making a long term plan
in order to offer just online degree programs. In such case, it is plausible
that the number of enrollment in online classes will rise significantly.
The author also asserts that during past years, development of online
degree programs have diminished expenses of dormitories and
classroom noticeably. Although it might seems tenable at face, it has
some defects since, you can always ask this question if internet could be
profitable for university of Omni. One of the main, if not the only, problem
with the premise is that online programs required high speed internet,
which need a substantial amount of budget. In fact, although price of
dormitory and classrooms have reduced, the expenses of internet will
increase significantly. Alongside that, there is possibility that those
building and classes did not need a hoard of money during past years.
Or perhaps students prefer to continue their education without online
programs in near future who knows?
Finally, as set forth by the author The University of human were not
successful to improve during past years and the cost of repairing an
maintaining building have raised significantly. Nevertheless, the rationale
behind this premise could be challenged owing to an unsettled question
if this university could develop through offering online programs as
University of Omni. One point that should not go unnoticed is that maybe
Humana University does not offer wide variety of courses and crowds of
students prefer to attend other universities or perhaps Humana is not a
high-ranked university. Moreover, comparison of these two university
could not be reliable owing to the fact that conditions and situations are
not identical in every given university.
Having scrutinized all the premises, a logical conclusion that can be
drawn is that there is a number of question, having been ignored by the
author while the answer of which could add to the logic of each premise.
An ancient, traditional remedy for insomnia — the scent of lavender flowers —
has now been proved effective. In a recent study, 30 volunteers with chronic
insomnia slept each night for three weeks on lavender-scented pillows in a
controlled room where their sleep was monitored electronically. During the
first week, volunteers continued to take their usual sleeping medication. They
slept soundly but wakened feeling tired. At the beginning of the second week,
the volunteers discontinued their sleeping medication. During that week, they
slept less soundly than the previous week and felt even more tired. During the
third week, the volunteers slept longer and more soundly than in the previous
two weeks. Therefore, the study proves that lavender cures insomnia within a
short period of time.
Write a response in which you discuss what specific evidence is needed to
evaluate the argument and explain how the evidence would weaken or
strengthen the argument.

It might seem logical, at first glance, to agree with the report that
lavender scented pillows helps to make feel better and causes sleep.
However, in order to fully agree with the argument, we need to have
significant amount of evidences. Though, the undrelying issue might
have merit, because of lack of complete evidences and unwarrented
assumptions, the report is unsubstainted and deeply flawed.
In order to fully evaluate the evidence, the first piece of evidence we
need to have is it’s sample size and category of selection. The report do
not mention about the age group, helth condition and sex about the
volunteers. The same result may not be applicable with other person.
Just conducting the experiment in thirty volunteer and concluding the
result is not enough to gestate conclusion.
Similarly, the monitoring process is also doubtful. It can not give full idea
about the sleeping ability about the volunteers. Like these people are
feeling safer in the experimental site and they develp their sleeping
habit. Perhaps the volunteers are just pretaining to sleep. In this case,
the conclusion is not logically convincing.
The reporter assumes that the scent of pillow remains constant
throughout the night. Regarding the specific case, the reporter assuems
that, once it is treated with levender leaves, it will work for whole night.
The assumption it self not compelling one because the scent migh have
loose it’s strength within few hours. Therefour the reason behing the
volunteers sleep is another like they feel relax during their experiment, or
it might be possible that, they feel safer than their own home. Also, the
dose is not mention in the report and we can not conclude any dose
work for the choronic patient.
As the report is engaged with an assumptions like the sleepness can
monitor through the elcectronic devices perfectly, scent power remains
constant, chornic sleep problem is similar for all group and sex, which do
a great role in reporter’s argument. If any one of these argument were
prove to wrong, the whole report will be thrown into question. Clearly, the
reporters required to take more diverse sample, monitor more effectively
for long time.

The following memorandum is from the business manager of Happy Pancake


House restaurants.
"Butter has now been replaced by margarine in Happy Pancake House
restaurants throughout the southwestern United States. Only about 2 percent
of customers have complained, indicating that 98 people out of 100 are happy
with the change. Furthermore, many servers have reported that a number of
customers who ask for butter do not complain when they are given margarine
instead. Clearly, either these customers cannot distinguish butter from
margarine or they use the term 'butter' to refer to either butter or margarine.
Thus, to avoid the expense of purchasing butter and to increase profitability,
the Happy Pancake House should extend this cost-saving change to its
restaurants in the southeast and northeast as well."
Write a response in which you discuss what questions would need to be
answered in order to decide whether the recommendation is likely to have the
predicted result. Be sure to explain how the answers to these questions would
help to evaluate the recommendation
The following appeared in a letter from the owner of the Sunnyside Towers
apartment building to its manager.
"One month ago, all the showerheads on the first five floors of Sunnyside
Towers were modified to restrict the water flow to approximately one-third of
its original flow. Although actual readings of water usage before and after the
adjustment are not yet available, the change will obviously result in a
considerable savings for Sunnyside Corporation, since the corporation must
pay for water each month. Except for a few complaints about low water
pressure, no problems with showers have been reported since the
adjustment. Clearly, restricting water flow throughout all the twenty floors of
Sunnyside Towers will increase our profits further."
Write a response in which you discuss what questions would need to be
answered in order to decide whether the recommendation is likely to have the
predicted result. Be sure to explain how the answers to these questions would
help to evaluate the recommendation

The following appeared in a health magazine.


"The citizens of Forsythe have adopted more healthful lifestyles. Their
responses to a recent survey show that in their eating habits they conform
more closely to government nutritional recommendations than they did ten
years ago. Furthermore, there has been a fourfold increase in sales of food
products containing kiran, a substance that a scientific study has shown
reduces cholesterol. This trend is also evident in reduced sales of sulia, a food
that few of the most healthy citizens regularly eat."
Write a response in which you discuss what specific evidence is needed to
evaluate the argument and explain how the evidence would weaken or
strengthen the argument.

The author concludes in the argument that the people of Forsythe are
following healthy lifestyle. The evidence given in the argument, are
assumptions made by author is not substantial.
The people of Forsythe may be adopting any healthy lifestyle. The
author provides evidence stating that the survey was carried out and
they followed eating habits closely recommended by the government
which they did not follow 10 years earlier. The statistics of the survey is
not shown in the argument. Here the argument shows no result from the
survey and the factors they followed for the survey like the number of
people surveyed, their age, the diet they followed the occupation and the
physical activities they carried out.
There was a fourfold increase in food product-kiran sales from the
survey.The author states that the scientific studies have shown that the
food product kiran reduces cholesterol and the author does not cite any
scientific journal or any scientific group which conducted this experiment.
Also no statistical evidence is shown that this product reduces
cholesterol in humans. The increase in sales of kiran product may be
due to many other factors like they have good marketing strategies,
inexpensive and etc. The residents of Forsythe are following government
approved diets that does not mean that kiran product is recommended
by government.
The sales of other product sulia were reduced noticeably from the
survey. Firstly the author does not give any details of the food product
sulia like it supplements or anything. The reduced sales may be because
of bad marketing strategies and it is too expensive, so very few healthy
people consumed .
The argument may be strengthened with further evidence of the survey
like the statistics of number of people, age, occupation and diet. The
number of people consumed Kiran in their diet versus Sulia, there cost
and more details of the scientific studies of both the products and
government approval. These evidences would support the fact the
increase in sales of Kiran and decrease of Sulia.

Humans arrived in the Kaliko Islands about 7,000 years ago, and within 3,000
years most of the large mammal species that had lived in the forests of the
Kaliko Islands had become extinct. Yet humans cannot have been a factor in
the species' extinctions, because there is no evidence that the humans had
any significant contact with the mammals. Further, archaeologists have
discovered numerous sites where the bones of fish had been discarded, but
they found no such areas containing the bones of large mammals, so the
humans cannot have hunted the mammals. Therefore, some climate change
or other environmental factor must have caused the species' extinctions.
Write a response in which you examine the stated and/or unstated
assumptions of the argument. Be sure to explain how the argument depends
on these assumptions and what the implications are for the argument if the
assumptions prove unwarranted.

The argument precludes humans being the cause of the extinction of the
large mammal species in the Kaliko Islands. It provides support in the
form of lack of evidence of contact between the two and no evidence of
bones of large mammals which could indicate hunting. However, the
argument makes a number of unwarranted assumptions which can make
the argument highly suspect. If the assumptions are not found to be true
then the argument falls apart.
Firstly, there is assumption that significant contact of humans with the
mammals is necessary to ensure extinction of the species. It may be that
humans can cause the species to become extinct without significant
contact. For instance, the humans would have extensively fed on the
most primary food resource of the mammals like some grasses or trees.
Since, their primary food source is depleted the mammals would have
gradually become extinct from the Island. Thus, humans are responsible
for the extinction even though there is no direct contact with the
mammals. This makes the assumption unwarranted and argument
feeble.
Secondly, there is assumption that hunting of mammals requires the
humans to discard the bones of the mammals. Just because no bones
were found does not mean mammals were not hunted. May be humans
found use for the bones of the mammals as well in some form of food
item for their dogs etc. May be they dug deeper pits to bury them as part
of their customs. Moreover, there is no evidence provided in the
argument that large mammals species do infact contain bones. Without
providing necessary evidence it is difficult to conclude that humans have
in fact not hunted the mammals. This undermines the conclusion that
humans are not responsible for extinction of the species.
Finally, there is an assumption that climate change or other
environmental factor will account for the extinction of the species.
However, there is no evidence presented in the argument that in fact
these factors are cause of extinction. The author makes a quantum leap
of faith to predict them as the cause and thus makes assumption
unwarranted. Moreover, even if these were factors it would be possible
that such an event would be likely caused by humans themselves. Thus,
even in that case humans are indirectly responsible for the extinction of
the species.
In conclusion, there are number of assumptions which are made in the
argument which need additional evidence to prove them to be
warranted. Otherwise, the argument becomes suspect and falls apart.

The following appeared in an editorial in a business magazine.


"Although the sales of Whirlwind video games have declined over the past two
years, a recent survey of video-game players suggests that this sales trend is
about to be reversed. The survey asked video-game players what features
they thought were most important in a video game. According to the survey,
players prefer games that provide lifelike graphics, which require the most up-
to-date computers. Whirlwind has just introduced several such games with an
extensive advertising campaign directed at people ten to twenty-five years old,
the age-group most likely to play video games. It follows, then, that the sales
of Whirlwind video games are likely to increase dramatically in the next few
months."
Write a response in which you examine the stated and/or unstated
assumptions of the argument. Be sure to explain how the argument depends
on these assumptions and what the implications are for the argument if the
assumptions prove unwarranted.
In this editorial the author concludes that the sales of Whirlwind video
games are likely to increase dramatically in the next few months. To
justify this conclusion, he cites a study showing that the declining sales
trend of these video games over the past two years is about to be
reversed. He also points out that survey reveals that players prefer
games with lifelike graphics, requiring the most up-to-date computers.
And he claims that Whirlwind has just introduced several such games
with an extensive advertising campaign directed at people 10 to 25 years
old. A careful scrutiny to this supporting evidence reveals that it lends
little credence to this argument for several respects.
First, the author provides no evidence to demonstrate that the recent
survey is statically reliable. In order to accept the strength of the result,
the survey’s sample must be sufficient in size and representative of the
overall population of the game players. Lacking evidence of a sufficiently
representative sample, the author can not authentically rely on the study
to draw any conclusion.
Second, even assuming that the study result are justifiably attributable to
the whole game players, the author unfairly assumes that the new
introduced games will satisfy the game player’s new needs and desires.
Although Whirlwind has just introduced several games with lifelike
graphics based on the current tastes of the player, without some
statistical evidence it can not be guaranteed that these games will be
accepted as satisfying games to encourage the players to buy more
games and that the sales of video games will increase. Or video games
might seem increasingly interesting or exciting at first, but this trend can
be weakened or even be reversed in long term. Without considering and
ruling out these and other possible factors, the author can not support
his claim about the significant success of Whirlwind video games sails.
Third, the author cites an extensive advertising campaign directed at 10
to 25 years old people to support its conclusion. However, he provides
no dear information about the performance of the advertising campaign
to credit its effective influence on the customer’s ideas about the new
games and on increasing the sails trend. In addition, just addressing
people 10 to 25 years old might not be enough to warrant the future sale
increase. There is the possibility that the population of games players
mostly includes younger children.
In sum, the author fails to convince me that the new video games of
Whirlwind seem to increase the sale trend dramatically. To bolster this
argument, he must assure me that the study results accurately reflect
the the whole population of games players’ taste and desires. To better
assess the strength of this argument I need more information about
whether the new measurement of Whirlwind will really satisfy the
customers and even this satisfaction will last in long term and will
increase the sails significantly. He also must provide clear information
about the advertising campaign performance in introducing the new
products of Whirlwind and in good coverage of the whole population of
the game players.

The following appeared in a memo from the vice president of marketing at


Dura-Sock, Inc.
"A recent study of our customers suggests that our company is wasting the
money it spends on its patented Endure manufacturing process, which
ensures that our socks are strong enough to last for two years. We have
always advertised our use of the Endure process, but the new study shows
that despite our socks' durability, our average customer actually purchases
new Dura-Socks every three months. Furthermore, our customers surveyed in
our largest market, northeastern United States cities, say that they most value
Dura-Socks' stylish appearance and availability in many colors. These findings
suggest that we can increase our profits by discontinuing use of the Endure
manufacturing process."
Write a response in which you examine the stated and/or unstated
assumptions of the argument. Be sure to explain how the argument depends
on these assumptions and what the implications are for the argument if the
assumptions prove unwarranted.
Here, the company's vice president cites that they eschew use of the
Endure manufacturing process. To support his claim, he also cites that
many customers who buy their socks, change socks after every three
months. Hence, he bolster his claim by saying that the Endure
manufacturing process which produces highly durable socks can be
used till two years, is not helpful for their company's economy. Although,
this kind of advantages, the vice president's claim made many
assumptions and did not provide proper numerical data. Therefore, I
refute this claim.
Initially, the author made very big assumption that the sample of people
whom he choose for test, can be considered as a representative of their
overall customers. If we say that this assumption is true, though it lacks
in numerical data. We can understand this point of view by one simple
example that there are 51 percent people who change his socks after
three months, still, there are 49 percent of people who use their socks
more than three months. So, we can't estimate exact number customers
who change their socks after 3 months. To bolster his claim, the author
reconcile his argument with numerical data of the people whom they use
as sample.
Secondly, the author says that the Endure manufacturing process is
costly and they can save money by replacing this process, though, he
had not given information about new process. The new process-they will
use in the place of the Endure process- has lower cost than Endure
process. Is this new process is enough cheap that it produces socks for
three month and can be used without any problem I just want to say that
this new process might produces socks which are rarely used more than
1 month. At the end, the cost of socks to customers is increase for three
months. Hence, the author should give information regarding new
process.
At the end, he has not given the numerical data that how many profit
they are going to get due to this modification? Is this profit is more than
the old process? How one can ignore the process durability? The author
would give information about relation between avoiding the Endure
process and profit.
In conclusion, I just want to tell that the author made claim without giving
information about new process, making assumptions and not providing
relative numerical data. Due to this all loopholes, one can easily disprove
this argument. If the author really wants to make a strong arguments, he
should remove this loopholes from the claim.

The following appeared in a memo from the vice president of marketing at


Dura-Sock, Inc.
"A recent study of our customers suggests that our company is wasting the
money it spends on its patented Endure manufacturing process, which
ensures that our socks are strong enough to last for two years. We have
always advertised our use of the Endure process, but the new study shows
that despite our socks' durability, our average customer actually purchases
new Dura-Socks every three months. Furthermore, our customers surveyed in
our largest market, northeastern United States cities, say that they most value
Dura-Socks' stylish appearance and availability in many colors. These findings
suggest that we can increase our profits by discontinuing use of the Endure
manufacturing process."
Write a response in which you discuss what specific evidence is needed to
evaluate the argument and explain how the evidence would weaken or
strengthen the argument.

In this argument, the arguer claims that the company should stop the
Endure manufacturing process to seek more profit. This conclusion is
based on the premises that customers prefer the appearance of product
rather than the endurance. The recommendation's logic is faulty,
however, since it relies on several shaky assumptions to bridge the gap
between it premises and conclusion.
According to the resent survey, the author assumes that they should
care more about the socks' look rather than their endurance which is
based on the purchasing frequency and preference of purchasers. There
are potential flaws. First, the author fails to provide the information of the
respondents whether they can be the representatives of the whole
buyers or even actual wearers, and whether they are selected randomly
in a large scope. Besides, the author mentions that the customers in the
survey are from northeast. Can those people represent buyers from the
whole country?
Secondly,several other things may also determine their choices. For
example, distinct career needs divers socks to match the working suite.
Usually, people work in an office will choose plain color to fit their suites
while people in fashion fields will consider style and color of socks more.
Gender and age, other factors may influence the choices. Females and
children may prefer light colors and fancy styles while males and the
older may prefer mono-color and simple patterns. Simply put, different
needs call for different choices.
Thirdly, even though the average customers will buy similar kinds of
socks, the arguer still can not assume customers care less about socks
quality based on their purchasing frequency. As the author mentions in
the survey, their customers know clearly that the socks are strong
enough to last quite a long time and this may be the requisite when
customers buy socks. Because,due to the common sense, people will
consider the quality of product first before buying.
Moreover, the author assumes that the profit will increase followed by
cutting the Endure manufacturing process. However, "Dura-Socks"-the
brand is famous for its feature of endurance. If people buy Dura-Socks
according to this, the move may cause the company losing its
customers. Another disadvantage of this advice may be that the
appearance of the product may be easily copied by their rivals. Then the
company may lose their competitive superiority in the market. At that
time, the company may be trapped into the financial problem let alone
the possibility of making profits.
To sum up, the argument that advises to cut the endure manufacturing
process is rather weak. If the author would have considered other hidden
factors behind purchasing preference and proceeded a more well-
rounded survey, the recommendation may seem more reasonable and
practical.

The vice president for human resources at Climpson Industries sent the
following recommendation to the company's president.
"In an effort to improve our employees' productivity, we should implement
electronic monitoring of employees' Internet use from their workstations.
Employees who use the Internet inappropriately from their workstations need
to be identified and punished if we are to reduce the number of work hours
spent on personal or recreational activities, such as shopping or playing
games. Installing software on company computers to detect employees'
Internet use is the best way to prevent employees from wasting time on the
job. It will foster a better work ethic at Climpson and improve our overall
profits."
Write a response in which you discuss what specific evidence is needed to
evaluate the argument and explain how the evidence would weaken or
strengthen the argument.

The recommendation from the vice president of human resources to the


president of Climpson Industries is based on certain assumptions, and an overall
lack in logical reasoning and rationality. This insular argument stands on
unproved hypothesis, and is rife with glitches.

One major reason why the argument is flawed is the direct correlation it assumes
with authority between the employees’ productivity and their use of internet.
Without a previously developed model or an undertaken survey to understand
how productivity fluctuates with the employees’ use of internet in the
workstations, one cannot draw a logical conclusion about the effectiveness of the
introduction of the new monitoring system. Furthermore, it assumes people using
company computers to indulge only in gaming and shopping, and that it is not
utilized for personal constructive purposes.

An additional disadvantage of the argument is its assumption of the employees’


internet use limited only to workstation computers. What the argument fails to
portray is that people can equally waste the same time indulging in such
activities from their personal laptops or cell phones. Moreover,  it also fails to
delineate the extent to and the effectiveness with which the software will gather
data for employee analysis.

A further drawback of the argument lies in the fact that it views personal and
recreational activities as time waste and unnecessary. Quite often, work proves
strenuous, leading people to indulge into personal activities to regain their vigour
for further work. Stalling such small recreational activities between works may
only prove fatuous, hindering growth and bringing in boredom. Interest level can
plummet with continuous work pressure, thus bringing down the entire
productivity.

Another notable flaw the argument poses is an obvious and unwanted result:
intrusion of privacy, which results in uncalled hesitancy and suspicion on the
part of the employees, who might be skeptical of the monitoring of their internet
use. Nevertheless, the vice president does not provide an insight as to how the
curbing the employees’ free time will result in better work ethic and increased
productivity. A realization of the other major factors like employment motivation,
team management, effective administration, adoption of better technology and an
overall conducive ambiance play an important part in the productivity of a
company, which clearly has not been hinted at. 

Taking all the above things into consideration, it can be summarized that if the
recommended steps as outlined by the vice president are imposed, it might not
prove to be as effective as it had been forecasted.

The following appeared in a memo from the president of Bower Builders, a


company that constructs new homes.
"A nationwide survey reveals that the two most-desired home features are a
large family room and a large, well-appointed kitchen. A number of homes in
our area built by our competitor Domus Construction have such features and
have sold much faster and at significantly higher prices than the national
average. To boost sales and profits, we should increase the size of the family
rooms and kitchens in all the homes we build and should make state-of-the-art
kitchens a standard feature. Moreover, our larger family rooms and kitchens
can come at the expense of the dining room, since many of our recent buyers
say they do not need a separate dining room for family meals."
Write a response in which you examine the stated and/or unstated
assumptions of the argument. Be sure to explain how the argument depends
on these assumptions and what the implications are for the argument if the
assumptions prove unwarranted.
The following argument claims that families prefer homes that have
larger family room and kitchen. The competitor of Bower Builders,
Domus Construction, answers the needs of its customer, hence its
homes are sold at higher prices than national average. Consequently,
Bower Builders would be able to get the similar results if they expand
their family rooms and kitchens. Stated in this way, the argument fails to
consider several factors which are essential to render reliable
conclusion. Hence, the proposal is not supported adequately for the
following reasons.
First, the nationwide survey that renders the decision might not be a
reliable one. To illustrate, there is no information regarding the scope
and number of states where the survey is conducted. It might be the
case that the survey interviewed only a small percentage of population to
reach this conclusion. Moreover, the questions that are included in the
survey may be biased and it might lead people to give specific answers.
In addition, there are no specific statistic about the percentage of
participants who consider these two qualities as the most important
ones. There may be negligible differences between the other options
and these two option. If this is the case, the survey should not be
considered as a valuable one. Consequently, the argument would be
much stronger if it explicitly stated the number of participants and the
statistics that are obtained as a result of the survey.
Second, even if the nationwide survey is a valid one, the populace of
cities where Bower Builders operates might not consider these two
qualities as the most significant ones. There is a possibility that the
survey is not conducted in these areas. The people who live in these
cities might have other priorities when purchasing a home. For example,
since the families in these cities might have four or five kids , they are
more likely to demand extra children rooms instead of larger family
rooms. The other option is there may be an increasing trend towards the
studio flats, hence the clientele might desire to buy homes in that type.
As a result, the argument would be substantially convincing if a separate
survey is conducted in each city where Bower Builders operates.
Finally, the argument claims that buildings of Domus Construction are
sold at higher prices than the national average and faster because they
consider the two factors that is provided by the survey when constructing
homes. The connection the argument reached is not supported
adequately because it does not eliminate other possibilities that might be
effective on the success of Domus Construction. Primarily,due to thrive
of economy or some other trends, people might invest their money to
real estates and it may be the fuel that boost the speed of sales. In
addition, the area where Domus Construction operates might be
wealthier district of the country, hence the home prices might be higher
than the national average. Other factors such as the reputiton of Domus
Construction could be an determining factor in their accomplishments.
Without eliminating other possibilities that might lead to the success of
Domus Construction, the Bower Builders would be able to achieve
similar results by just expanding family rooms remains much of wishful
thinking rather than substantive evidence.
Finally, the argument is flawed for the above-mentioned reasons and
therefore it fails to provide a convincing case. In order to assess the
merits of a certain situation, it is essential to have full knowledge of all
contributing factors. In this case, the factors that are considered as
important when purchasing a home by the populace of the cities Bower
Builders operates would be beneficial.

The following appeared in a letter from a firm providing investment advice for
a client.
"Most homes in the northeastern United States, where winters are typically
cold, have traditionally used oil as their major fuel for heating. Last heating
season that region experienced 90 days with below-normal temperatures, and
climate forecasters predict that this weather pattern will continue for several
more years. Furthermore, many new homes are being built in the region in
response to recent population growth. Because of these trends, we predict an
increased demand for heating oil and recommend investment in Consolidated
Industries, one of whose major business operations is the retail sale of home
heating oil."
Write a response in which you examine the stated and/or unstated
assumptions of the argument. Be sure to explain how the argument depends
on these assumptions and what the implications are for the argument if the
assumptions prove unwarranted.

The author assumes that the developments like the new homes built and
the cold winters guarantee a rise in demand for heating oil. Based on
this fallacious assumption, he advises his client to invest in Consolidated
Industries. However the author fails to consider other alternative
explanation. He also presents no concrete evidence to bolster his
argument. Thus the conclusion is incredible.
To start with the author mentions that the northeastern United States
have traditional used oil as their major fuel for heating. However the to
ensure that the traditional trend is prevailed the author has to answer
few questions.What percent of population still uses oil as their primarily
fuel? Based on the answer the argument can be evaluated further.
Next the author states theta the climate forecasters has predicted that
the cold weather pattern will persist for several years. The argument
begs the question, how accurate is the forecast? This question has to be
answered in to to validate the author's conclusion. Moreover the
prediction contains vague and ambiguous terms like several years. Thus
the author has to address the crucial questions and cite concrete
evidences to corroborate his claim.
Afterwards the author states that many new homes have been built in
the region during past year. Based on this the author assumes that new
citizens will also use the oil and thus the oil usage will increase. However
in order to make this presupposition, certain questions need to be
answered. Exactly how many new homes are built? Do the new
residents use oil as their fuel? The lack of answers make the author's
assumption unwarranted.
Finally the author recommends that client should invest in Consolidated
industries whose main business operation is the sale of home heating
oil. To draw this conclusion, the author makes several unreasonable
suppositions. Nevertheless to validate his conclusion he should have
mentioned, what precisely is the target market of the Consolidated
industries? Perhaps they operated only in the southern United States.
Even no information regarding their financial status is given. The
company could be in debts and financial crises. All this would then refute
the conclusion.
To recapitulate, the author's argument is full of holes. The lack of
evidences, flawed assumptions and unanswered queries make the
author's assumption that the developments will certainly result in an
increased demand for heating oil specious. Ergo his recommendation is
also dubious.

The following appeared in an article in the Grandview Beacon.


"For many years the city of Grandview has provided annual funding for the
Grandview Symphony. Last year, however, private contributions to the
symphony increased by 200 percent and attendance at the symphony's
concerts-in-the-park series doubled. The symphony has also announced an
increase in ticket prices for next year. Given such developments, some city
commissioners argue that the symphony can now be fully self-supporting, and
they recommend that funding for the symphony be eliminated from next year's
budget."
Write a response in which you discuss what questions would need to be
answered in order to decide whether the recommendation and the argument
on which it is based are reasonable. Be sure to explain how the answers to
these questions would help to evaluate the recommendation.

The argument proposes the funding of the Grandview Symphony should


be scotched starting the following year due to the donations made by the
private sector having doubled and also concurrent with increased
attendance to the symphony. Furthermore it specifies the increased
prices of tickets for the following year would be doubled and attributes
this as additional reason to cease the funding by the city for the following
year’s budget.
The author is unclear about the reasons as to why the Grandview
concert required funding in the first place and how a temporary surge in
contributions could make the symphony self-sustainable. There are no
cogent reasons mentioned that would indicate the need to cease funding
by the government. Simply because the funds raised from the private
sector has doubled does not imply that the accumulated fund is
prodigious, it could be that private contributions at preceding years were
little and so doubling that may not necessarily accrue to a large sum.
Also the attendance having doubled could occur as a result of different
temporary and coincidental occurrences, perhaps as a result of the
symphony taking place during the holiday, which would probe more
attendees since most of the people are less busy and free to jubilate the
season. There should be a fastidious investigation done into the
surrounding factors that lead to the success of the event in the year with
certified professionals providing proper accounts of the symphony and
the city should heed to their advice as to whether or not the event would
be a success the following year if the government decides to pull out.
Furthermore, the city commissioner’s conclusion to stop funding based
on the assumption that the symphony would be self-supporting is not
convincing. There is no definitive evidence providing insight into the
auspicious turnout of the event the following year if the government
stanches funding, instead the increased ticket cost would likely cause a
reduction in the attendance that would be detrimental to the longevity of
the Grandview symphony. There is no estimate regarding the popularity
of the event and the attendance at the event the previous year, if there
was a large turnout and as the author mentioned the attendance doubled
the present year then the future looks bright, but if the turnout was little,
doubling that may not exactly adumbrate a profitable turnout, as such
there should be proper inquiry into these missing details before a final
decision is made.
And if the Grandview symphony is indeed a promising event that has
picked up sales and is flourishing with glimpses of a bright future, the
author should address other options that could lead to better outcomes.
Perhaps it would then be beneficial to the government to aid and invest
in such an event that could help attract tourists and gain popularity that
could act as a source of revenue for the city.
The government should consult the appropriate expertise to comprehend
the current standing of the event and also inquire into the budget to
understand exactly how much progress has been made and whether or
not it is sufficient for the sustenance of the Grandview symphony without
being sponsored by the Grandview city, keeping in mind its potential of
being a source that could generate revenue for the city.

The following appeared in a memo from the director of a large group of


hospitals.
"In a laboratory study of liquid antibacterial hand soaps, a concentrated
solution of UltraClean produced a 40 percent greater reduction in the bacteria
population than did the liquid hand soaps currently used in our hospitals.
During a subsequent test of UltraClean at our hospital in Workby, that hospital
reported significantly fewer cases of patient infection than did any of the other
hospitals in our group. Therefore, to prevent serious patient infections, we
should supply UltraClean at all hand-washing stations throughout our hospital
system."
Write a response in which you examine the stated and/or unstated
assumptions of the argument. Be sure to explain how the argument depends
on these assumptions and what the implications are for the argument if the
assumptions prove unwarranted.

Laboratory studies revealed that using UltraClean could reduce the


bactera population by a whooping 40%. And subsequent tests also
revealed that, upon the use of UltraClean there were a fewer reported
cases of patient infection. Due to the combined positive results of these
tests and studies, the Director suggested using UltraClean in all their
hand-washing stations through out the hospital system. Although, this
argument appears to be apt and adequate, there are some serious
unwarranted flaws which could prove devastating.
Upon scrutinizing the argument, it can be seen that the laboratory
studies account for reduction of only 40% of the bacterial population.
There may be many other liquid hand wash companies out there in the
market, who are producing hand wash or sanitiziers that are proved to
remove more than 40% of the bacteria present. It is subtly assumed that,
there are no other hand wash which is better than the specified group of
hospitals. This assumption if proved correct could be fatal to all the
people that are using the hand wash.
Furthermore, the argument does not specify how healthy the individuals
are, at the time of conducting the test. It could be that most of the
percentage of people chosen for the test were already fit and in good
health. It might be that they were having a healthy diet in their daily life.
And they are already immune to most of the infections. So, the argument
assumes that all the people chosen for the test were normal people,
suffering from few infections.
If this assumption was to prove unwarranted and UltraClean hand wash
was supplied to all the people working in the hospital, it could be
dangerous.
Moreover, only few cases were reported, out of all the patients tested,
some of them could have used other medicines to counteract the side
effects and these were not detected by the testing individuals.
Additionally, statistics for the number of people tested have not been
submitted. It could be that only a meagre 50 people were tested and only
a 10 of them were infected and the other 40 were not infected, this does
not mean that UltraWash could be supplied to the hospital system and
still only a few people would be infected. It is presumed that, because a
few samples of the people from the hospital reported fewer infections
doesn't necessarily mean that when UltraWash is used by many people
in the hospital, still only a few would be infected.
To conclude, the above unwarranted arguments could prove to be an
erroneous mistake.

The following appeared in a letter to the editor of the Parkville


Daily newspaper.
"Throughout the country last year, as more and more children below the age
of nine participated in youth-league sports, over 40,000 of these young
players suffered injuries. When interviewed for a recent study, youth-league
soccer players in several major cities also reported psychological pressure
exerted by coaches and parents to win games. Furthermore, education
experts say that long practice sessions for these sports take away time that
could be used for academic activities. Since the disadvantages outweigh any
advantages, we in Parkville should discontinue organized athletic competition
for children under nine."
Write a response in which you examine the stated and/or unstated
assumptions of the argument. Be sure to explain how the argument depends
on these assumptions and what the implications are for the argument if the
assumptions prove unwarranted.

The author recommends that Parkville discontinnue its athletic


competition for children under nine due to the number of injured children,
the detraction from academic activities and the cited psychological
pressure involved in such athletic activities, as cited in a recent study.
Nevertheless, the authors suggestions are predicated on some yet to be
validated assumptions.
The author mentions that 40,000 children suffer injuries due to athletic
activities assuming that this is a high number of injuries and as a result
all sport should be discontinued for those below nine years. However,
she does not give a figure on the total number of children who
participated in the sport so as to enable an evaluation of the significance
of the number of injured players. If perhaps a million children participated
in the sport and only 40,000 children got injured, then that is a 4% injury
which is not high enough to necessitate discontinuing such sport
activitiy. It sport and the one feature of sport is injury. There can be no
sport without injury and if the percent of injured is low then, the sport
activity is being managed well.
Also the author does not distinguish which kind of sports cause these
injuries. He/she assumes that all sports are dangerous. May be the
injuries were the result of one kind of sport activity which was too risky
for nine year olds; but that cannot become the reason why other sport
activities, which may be safer for such age grade of children, should be
banned.
The author assumes valid the opinions of educational experts who have
an objection to athletic activities due to the time it takes, as they believe
it cuts out of the time spent on academics. However, the author fails to
recognize that other educational experts may argue that sports are
necessity in the education of a child. If such sports are necessary then
discontinuing them, would negatively harm the chiildren of Parkville.
The author also assumes that psychological pressure is harmful and
hence since athletics involve psychological pressure, it should be
discontinued. Nowhere in the above report does the author indicate the
level of psychological pressure involved or any harm caused by this
pressure. In fact, some level of psychological pressure may be helpful to
spur children into performing well at various activities.
The author needs to objectively analyze the issue surrounding children
sport in Parkville – injury rate, the benefits of such sports, the possibilty
of other less harmful sport activities, etc. – before reaching a conclusion
on whether sports should be continued or not.

When Stanley Park first opened, it was the largest, most heavily used public
park in town. It is still the largest park, but it is no longer heavily used. Video
cameras mounted in the park's parking lots last month revealed the park's
drop in popularity: the recordings showed an average of only 50 cars per day.
In contrast, tiny Carlton Park in the heart of the business district is visited by
more than 150 people on a typical weekday. An obvious difference is that
Carlton Park, unlike Stanley Park, provides ample seating. Thus, if Stanley
Park is ever to be as popular with our citizens as Carlton Park, the town will
obviously need to provide more benches, thereby converting some of the
unused open areas into spaces suitable for socializing.
Write a response in which you examine the stated and/or unstated
assumptions of the argument. Be sure to explain how the argument depends
on these assumptions and what the implications are for the argument if the
assumptions prove unwarranted.

Stanley Park is seemingly visited by the fewer people. To increase the


visits, the author of the statement believes that equipping Stanley Park
with more benches would be the only solution. This conclusion is based
on the premise that such a strategy have worked in Carton Park so it
would Work in Stanley Park either. To reach the conclusion, the author
made several unsupported assumptions.
First, the author assumes that the average 50 cars per day, shows a
drastic decline of Stanley Park visitors. As there is no evidence showing
that before last month what number of cars placed in the Park’s parking
lot, we cannot be certain that any notable decline happened. It is
possible that the cars were 51 cars per day and in last month it is 50.
Even if there are significantly fewer cars it might be due to other
reasons. Another parking lot might offer space for the cars that people
place their cars in there rather than using the Park’s parking. If this is the
case, there is no decrease to worry about in the first place.
Secondly, assuming that the decline happened, the author assumes that
as more than 150 people visit Carlton Park in the heart of business
district per day, it shows the success of the park in attracting the people
to come to the park. It is possible that as Carlton Park is located in the
center of the business district, people merely use the park as a passage
way to go to the other parts of the district. It is also possible that the
concentration of the population in that district is far more than the
population of the people around the Stanley Park that Carlton Park has
more visitors.
Finally, even if the benches were the reason why more people go to the
Carlton Park it might not work the allocation of more benches might not
be efficient for the Stanley Park, as the author wrongly assumes
otherwise. We do not know the numbers of the benches, they might be
sufficient and there are other reasons why people do not go the Stanley
Park. The water ponds may stick. There may be a shortage of the
playground for children. The guards might not be polite with the people
and so on. If these are the case, with or without benches visitors decline
will continue.
In short, the author made a weak argument since it is replete with
ambiguous points. As discussed, the answers to the enumerated
questions would illustrate the argument. As long as the questions are left
unaddressed, the conclusion which is based on such an unclear
argument cannot be tenable.

The following appeared in a memo from the owner of a chain of cheese stores
located throughout the United States.
"For many years all the stores in our chain have stocked a wide variety of both
domestic and imported cheeses. Last year, however, all of the five best-selling
cheeses at our newest store were domestic cheddar cheeses from Wisconsin.
Furthermore, a recent survey by Cheeses of the World magazine indicates an
increasing preference for domestic cheeses among its subscribers. Since our
company can reduce expenses by limiting inventory, the best way to improve
profits in all of our stores is to discontinue stocking many of our varieties of
imported cheese and concentrate primarily on domestic cheeses."
Write a response in which you discuss what questions would need to be
answered in order to decide whether the recommendation is likely to have the
predicted result. Be sure to explain how the answers to these questions would
help to evaluate the recommendation

The memo given by the business manager of the chain of cheese stores
proffers to discontinue stocking of the varieties of imported cheeese and
concentrate solely on the sale of domestic cheese. The manager has
arrived at this decision based on the best-selling domestic cheddar
cheese noted to gain popularity in the recently introduced cheese stores.
However, the argument relies on many unstated assumptions and the
argument cannot be taken for granted unless these assumptions are
validated with proper evidence. Thorough analysis of the assumptions is
mandatory to evaluate the concerned argument.
The manager quotes that the stores in his chain have stocked a wide
variety of domestic and imported cheese. It assumes that both of the
varieties are equally significant in number and distributed evenly
thoroughout all its stores. However, the particular distribution of each
type of cheese in each of the stores is mandatory to arrive at a clear-cut
picture of the prevailing scenario in cheese stores. It is almost
impossible to judge the level of distribution of cheeses just by the term
"wide variety". Hence it has to be quantified to get a clear understanding.
Next, the manager assumes that the five-best selling domestic cheddar
cheeses in Wisconsin had no tough competition with other types of
cheese. For instance, if any other kind of imported cheese just falls
second to the sales of this cheddar cheese, then it cannot be neglected
and there is a huge possibility that it will surpass the current popularity of
cheddar cheeses. Thus, a comparison needs to be established to prove
without doubt that the five-best selling domestic cheddar cheeses
deserve their position. If this fact is not proven, then it is impossible to
assume that Cheddar cheeses of this type will ensure high sales.
The manager quotes the results of survey conducted by the World
Magazine. This survey particularly indictates the preference of the
subscribers towards domestic cheese. It does not however guarantee
that the subscribers will buy this type of cheese. Hence the price levels
of the domestic cheese must be proved to be affordable for the
subscribers to opt for this type of cheese. And the group interviewed by
the survey must be established. Only if the survey interviews a wide
variety of population, can we come to a conclusion regarding the validity
of the survey.
Finally, the manager is keen on reducing expenses by bringing to
control, the inventories of the company. Hence he suggests
discontinuing stocking of many varieties of imported cheese. But, the
manager has initially quoted that the imported cheese has been stocked
for many years together. Therfore, to determine whether the imported
cheese can be discontinued, without incurring huge losses is a question.
Moreover, it is not established that the domestic cheese will sell in the
desired manner. Consequently, it is preposterous to come to a
conclusion that profit of the stores will be magnified to a great extent.
Above all, the popularity of some of the domestic varieties of cheese is
noted only in the newly introduced stores. This leads to the doubt
whether the same level of popularity for the domestic variety will be
achieved when introduced in all of the existing cheese stores. The huge
profits in one of the stores is correlated with similar kind of profits in
other stores. It assumes the same nature of stores, the income of
customers, the favorable location of the stores to be applicable for all of
the stores. These have to be established with proper evidence.
Thus the argument put forward by the manager rests on lots of stated
and unstated assumptions like: wide variety of two types of cheese to be
equally distributed in all of the stores, the validity of the survey, the
affordable price levels of the domestic Cheddar cheese, no losses
incurred on discontinuing imported cheese, the same level of popularity
to be achieved in all of the cheese stores as observed in the newly
introduced stores. These assumptions have to be substantiated with
proper evidence. Otherwise, the suggestions put forward by the
manager will turn out to be futile and serve no purpose. Therefore, it is of
utmost importance to delve into the intricacies of the argument and solve
the contradictions before considering the manager's suggestions in the
memo for implementation.
The following appeared as part of a business plan developed by the manager
of the Rialto Movie Theater.
"Despite its downtown location, the Rialto Movie Theater, a local institution for
five decades, must make big changes or close its doors forever. It should
follow the example of the new Apex Theater in the mall outside of town. When
the Apex opened last year, it featured a video arcade, plush carpeting and
seats, and a state-of-the-art sound system. Furthermore, in a recent survey,
over 85 percent of respondents reported that the high price of newly released
movies prevents them from going to the movies more than five times per year.
Thus, if the Rialto intends to hold on to its share of a decreasing pool of
moviegoers, it must offer the same features as Apex."
Write a response in which you discuss what questions would need to be
answered in order to decide whether the recommendation is likely to have the
predicted result. Be sure to explain how the answers to these questions would
help to evaluate the recommendation

The author of the statement believes that in order not to be closed,


Rialto Movie Theater needs to make changes and become up-to-date as
the Apex Theater is. This conclusion is based on an unspoken
assumption that since the Apex features modern apparatuses, the
moviegoers, whose numbers are seemingly in a decline, might prefer to
go to Apex instead of the Rialto Movie theater. The author needs to
answer several vital questions to change the argument from the current
ambiguous form to a clear one.
First, the author needs to ask why people choose Rialto Movie Theater.
It is possible that people mainly go there because of the nostalgic aroma
preserved in that theater. If the change happens, there might be no
nostalgic ambience for the classic movie goers. Furthermore, the author
needs to ask whether the Rialto’s box office is failing. As there are no
statistics of the number of sold movie tickets, the downtown movie
theater may not face a bankruptcy at all.
Secondly, the author needs to ask whether Apex can be as a successful
model for Rialto. It is possible that despite all the expenditure on video
arcade, carpeting, and a state-of-art sound system, the Apex is not
successful in absorbing enough movie goers. The net profit of Apex
might be so low that makes Apex’s investment unjustifiable. If this is the
case, Apex cannot be a successful model to be imitated by Rialto.
Finally, the author needs to ask questions about the survey. Does the
five-time-going-to-theater by each movie goer show a declining trend? It
is possible that even with this frequency of going to the theater, an
increased box office hit is expected for the Rialto rather than a decrease
in the ticket sale. The author, thus, needs to ask how this frequency
would be translated into selling movie tickets. Furthermore, even the
movie goers are in a decline because of the price of new released films,
the author still needs to ask how this fact is consistent with Rialto
Theater. People might prefer watching old movies in the half-century
built Rialto Theater and the price of new movies might even encourage
people to watch old movies in there.
In short, as discussed above, there are several vital questions whose
answers are needed to illustrate the author’s argument. As the trend of
moviegoers is not clear and so are many other crucial points, the
argument cannot be evaluated as persuasive.
A recent study reported that pet owners have longer, healthier lives on
average than do people who own no pets. Specifically, dog owners tend to
have a lower incidence of heart disease. In light of these findings, Sherwood
Hospital should form a partnership with Sherwood Animal Shelter to institute
an adopt-a-dog program. The program would encourage dog ownership for
patients recovering from heart disease, which should reduce these patients'
chance of experiencing continuing heart problems and also reduce their need
for ongoing treatment. As a further benefit, the publicity about the program
would encourage more people to adopt pets from the shelter. And that will
reduce the incidence of heart disease in the general population.
Write a response in which you examine the stated and/or unstated
assumptions of the argument. Be sure to explain how the argument depends
on these assumptions and what the implications are for the argument if the
assumptions prove unwarranted.
The author of the statement above believes that in order to reduce the
incidence of the heart disease in the general population, it is an efficient
way to encourage people to adopt pets. This conclusion is based on a
recent study reporting that pet owners have longer and healthier lives
than do people who have no pets. The author takes several assumptions
for granted to reach such a conclusion. With each assumption being
potentially unwarranted, the argument cannot be tenable.
First, the author assumes that the people in the study are representative
of the people of the Sherwood. As there is no evidence showing the
features of the people in the study, it is highly possible that the
Sherwood people are different with them. The Sherwood people might
be old, fat, so much more prone to develop heart diseases, while the
people in the study might be young and healthy. It is also possible that
the people of the Sherwood are genetically prone to develop heart
diseases, then, with or without pets they would have such incidence of
heart diseases.
Secondly, even if the people of Sherwood would benefit from having the
dogs as the people in the survey do, the author hastily assumes that
those who have pets would lesser need their treatments. In no place of
the argument the author provided any evidence that having dogs would
also have a progressive function on the treatment of those who were
already developed such diseases.
Thirdly, the author assumes that with prompting an adopt-a-dog
program, people would come and adopt pets for themselves. It is
possible that the program fails to encourage people to adopt any dog.
People might have their reason why not to have pets. It is possible that
people have allergies to pets, the cost of caring a pet might be high, and
the time and responsibility demanded to care pets, all can preclude
people from having dogs.
Finally, even if all the assumptions are veritable, when the author
predicts reduction in the heart diseases in the future, he/ she fails to
consider there may be other factors that can overshadow the
advantages of having a dog. It is possible that the lifestyle of the pet
owners makes them vulnerable to heart diseases. The fats, the stress of
work, the pollution in the air, unhealthy diets, all can overshadow the
function of owning pets in the future. Hence the prediction cannot be
valid.
In short, as discussed, the author reaches the conclusion via taking
several unsupported assumptions for granted. As the argument lacks
any supporting evidence, the conclusion which is based on the argument
cannot be tenable

The following appeared in a memo from a vice president of a large, highly


diversified company.
"Ten years ago our company had two new office buildings constructed as
regional headquarters for two regions. The buildings were erected by different
construction companies — Alpha and Zeta. Although the two buildings had
identical floor plans, the building constructed by Zeta cost 30 percent more to
build. However, that building's expenses for maintenance last year were only
half those of Alpha's. In addition, the energy consumption of the Zeta building
has been lower than that of the Alpha building every year since its
construction. Given these data, plus the fact that Zeta has a stable workforce
with little employee turnover, we recommend using Zeta rather than Alpha for
our new building project, even though Alpha's bid promises lower construction
costs."
Write a response in which you discuss what questions would need to be
answered in order to decide whether the recommendation and the argument
on which it is based are reasonable. Be sure to explain how the answers to
these questions would help to evaluate the recommendation.

A vice president in his memorandum expresses his opinion that Alpha


company should be used for the new construction site due to several
reasons. More specifically, when instructed to construct the same office
building as Zeta company, according to the vice president Alpha
company achieved 30% less cost of building the offices and also the
energy consumption of the Alpha building during these years is also less
than Zeta building. Also, the workforce in Alpha is more stable than in
Zeta. However, there are several assumptions made in the proccess of
supporting the argument the most important of which are being
examined in the paragraphs below.
The author in his way of backing up his argument that indeed the
company should use Alpha construction company instead of Zeta is
failing to take into consideration that the two construction sites are
situated in two different regions. This indicates that the climate, the
terrain and the working conditions may have been entirely different
between these two construction sites. For example, consider that Zeta
company had to build the office building in a rocky terrain with a little
road infrastructure and the operations had to seize during winter due to
heavy snowfall. This example, indicates that the conditions in the buiding
construction site by Zeta company may have taken more money to build
and also the costs of maintaining the offices due to poor roads may be
higher. This consideration, undermines the assumption of the vice
president that these two buildings had the same weather and climate
conditions and therefore that Alpha is a more efficient company than
Zeta.
Furthermore, the vice president states that the Zeta building has more
energy consumption than the Alpha building and this is due to
construction malfunctions by Zeta company. However, the energy
consumption may be more not only from construction issues but from
higher work load in Zeta building. For example, if the Zeta building is
situated in a region were it is needed for the employees to work overtime
and stay up late, the energy consumption from the lights, computer
terminals and heating or cooling may be significantly larger than the
Alpha building. Also, as mention in the paragraph above if the Zeta
building is situated in a region with heavy winters then the energy
consumption from heating the entire building may be larger than heating
the Zeta building which may be placed in a region with less severe
climate conditions. The above examples, undermine the argument made
by the vice president and may lead to a change of the conclusion stating
that Alpha construction company is better than Zeta.
Lastly, an unstated assumption the vice president is making is that the
workforce on Zeta company is less or completely unstable when
contrasted with the workforce of Alpha company and thus Alpha
company is a better candidate for constructiong the new office building.
However, we should take into consideration the possibility that the
changing workforce of Zeta company, just because it is changing can
achieve better results in order to stay in the company longer. Also, there
is the issue of time, the vice president fails to give a specific time period
in which the two constructions were finished. Therefore, there is also the
possibility that the more unstable workforce of Zeta company to manage
and finish the new work in shorter time than the Alpha company.
To sum up, in this essay the stated and unstated assumptions used by
the vice president in order to support Alpha company were demonstrated
through examples. The vice president should provide more evidence
regarding the climate conditions, the work load and the time of
construction in order to back up his assumptions.

The following appeared in a memo from a vice president of a large, highly


diversified company.
"Ten years ago our company had two new office buildings constructed as
regional headquarters for two regions. The buildings were erected by different
construction companies — Alpha and Zeta. Although the two buildings had
identical floor plans, the building constructed by Zeta cost 30 percent more to
build. However, that building's expenses for maintenance last year were only
half those of Alpha's. Furthermore, the energy consumption of the Zeta
building has been lower than that of the Alpha building every year since its
construction. Such data indicate that we should use Zeta rather than Alpha for
our contemplated new building project, even though Alpha's bid promises
lower construction costs."
Write a response in which you discuss what specific evidence is needed to
evaluate the argument and explain how the evidence would weaken or
strengthen the argument.
In his memo the vice president suggests that they should employ Alpha
Construction Company in place of Zeta for all future projects. He comes to this
conclusion by comparing the two companies after which he says that Zeta has
proved to be more expensive than Alpha. Moreover, according to him Alpha
has a stable workforce as well. However, the vice president does not make
use of good sense and logic in coming to this conclusion. In order to decide
which company is better he needs to make a proper survey of their works.

First of all, the vice president cites that the two buildings erected by these
construction companies ten years ago were is different regions. It is possible
that in the present times, the cost of construction has undergone a change.
Consequently, it is likely that Zeta is no more as expensive as it was ten years
ago. Moreover, since they were constructed in different places, the vice
president should not expect the same durability of the two buildings. In a span
of ten years, there could be many changes in a place, which would have an
effect over the buildings constructed over it. It is possible that the building
constructed by Zeta construction company has seen many adverse weather
changes and natural calamities than the building constructed by Alpha. It is
evident that there would be more wear and tear of the building that is exposed
more to rains, snow, hail, storms and earthquakes etc. The condition of a
building after ten years should not be a standard to measure the quality of the
construction company.

If the building constructed by Zeta costs 30 percent more than that


constructed by Alpha, it does not indicate that Zeta is more expensive than
Alpha. The cost of constructing the building could have risen due to other
factors like the cost of brick and cement at that place. Moreover, the cost of a
building also depends upon the quality of material used in the building. It is
likely that Zeta Construction Company has installed the best material and has
given high quality interiors to the building. Therefore, even if the basic floor
plan is the same, it is the kind of material used that can make the cost of
construction of one building higher than that of the other.

The vice president gives the account of cost of maintenance of only the past
one year. Firstly, it is possible that in the previous years the building
constructed by Zeta has not demanded a high maintenance cost. Moreover, it
is likely that the building constructed by Alpha constructors has been
renovated a few years back and all the damaged parts have been replaced.
Therefore, their maintenance cost has gone down as compared to the other
buildings’.

Similarly, the energy consumption of one building is different from that of


another building. It is possible that the energy consumption of the building
constructed by Zeta is more due to longer working hours. Moreover, it is also
possible that the people using the building have been careless in consuming
energy, which has resulted in more energy consumption.

If Alpha Construction Company has a stable workforce, it does not mean that
Zeta would not have a stable workforce. Further, the argument only
concentrates on these two construction companies. There could be other
construction companies, which could be better and less expensive than these
companies. Moreover, it is likely that many more good construction
companies come up in future. Therefore, the vice-president should not make a
decision in haste, but should conduct a proper study of all the companies
before deciding which one could be the best in future.

Potrebbero piacerti anche